Suggested Answer Paper CAP II Dec 2019 6

You might also like

Download as pdf or txt
Download as pdf or txt
You are on page 1of 117

CHARTERED ACCOUNTANCY PROFESSIONAL II

(CAP-II)

Suggested Answer Paper


December 2019

Education Department
The Institute of Chartered Accountants of Nepal
Suggested Answer Paper Group I

© The Institute of Chartered Accountants of Nepal


All exam questions and solutions are copyright of The Institute of Chartered Accountants of
Nepal (ICAN) and can only be used by students for preparation of their CA exams. They cannot
be published in any form (paper or soft copy), or sold for profit in any way, without first gaining
the express permission of ICAN. Nor can they be used in examinations, in whole or in part, by
other institutions or universitites.

Published On: February 2020

Disclaimer:
The Suggested Answer Paper are prepared by the Institute with a view to assist the students in
their study. The suggested answers published here are indicative and not exhaustive. They do
not constitute the basis for evaluation of the students' answers in the examination. Students are
expected to apply their knowledge and write the answer in the examinations taking the
suggested answers as guide. Due care has been taken to prepare the suggested answer paper.
In case students need any clarification, creative feedbacks or suggestions for the further
improvement on the material, or any error or omission on the material, they may report to the
email educationdepartment@ican.org.np at Education Department of the Institute.

© The Institute of Chartered Accountants of Nepal 2


Suggested Answer Paper Group I

Table of Contents
Group I

Paper 1: Advanced Accounting .............................................................................................................. 4


Paper 2: Audit and Assurance ............................................................................................................... 21
Paper 3: Corporate and Other Laws ...................................................................................................... 32
Examiner’s Commentary on Students' Performance in December 2019 Examinations .............. 46

© The Institute of Chartered Accountants of Nepal 3


Suggested Answer Paper Group I

Paper 1: Advanced Accounting


Attempt all questions. Working notes should form part of the answer.
1. A and B who carry on partnership business in the name of M/s AB Ltd., closes their firm's
account as on 31st Ashadh each year.
Their partnership agreement provides:
i) Profit loss sharing A : 2/3 and B : 1/3 and
ii) On retirement or admission of Partner :
a) If the change takes place during any accounting year, such partner's share of profit
or losses for the period up to retirement or from admission, is to be arrived at by
apportionment on a time basis except otherwise stated for specific time(s).
b) No account for Goodwill is to be maintained in the firm's books.
c) Any balance due to an outgoing partner is to carry interest @ 9% p.a. from the
date of his retirement to the date of payment.
The Trial Balance of the firm as on Ashadh 31st 2076 was as follows:

Particulars Amount in (Rs.) Amount in (Rs.)


Capital account
A - 24,000
B - 12,000
C (Cash brought in on 30-09-2075) - 9,000
Plant and machinery at cost 22,000 -
Depreciation provision up to 31-03-2075 - 4,400
Motor car at cost 30,000 -
Depreciation provision up to 31-03-2075 - 6,000
Purchases 84,000 -
Stock as 31st Ashadh, 2075 15,500 -
Salaries 18,000 -
Debtors 5,400 -
Sales - 1,20,000
Travelling expenses 800 -
Office maintenance 1,200 -
Conveyance 500 -
Trade expenses 1,000 -
Creditors - 10,100
Rent and rates 3,000 -
Bad debts 900 -
Cash in hand and at Bank 3,200 -
1,85,500 1,85,000
'A' retired from the firm on 30th Poush, 2075 and on the same day 'C' an employee of the
firm was admitted as partner. Further Profit or Losses shall be shared - B: 3/5 and C: 2/5.
Necessary Accounting Entries for adjustments were pending up to 31-03-2076. You are
given the following further information:
i) The value of firm's goodwill as on 30th Poush, 2075 was agreed to Rs. 15,000.
ii) The stock as on 31st Ashadh, 2076 was valued at Rs. 18,550.
iii) Partners' drawings which are included in Salaries: A – Rs. 2,000, B- Rs. 3,000 and C-
Rs. 1,000.
© The Institute of Chartered Accountants of Nepal 4
Suggested Answer Paper Group I

iv) Salaries also includes Rs. 1,500 paid to C prior to his being admitted as a partner.
v) Bad-debts of Rs. 500 related to period up to 30th Poush, 2075.
vi) As on 31st Ashadh, 2076 rent paid in advance amounted to Rs. 600 and trade expenses
accrued amounted to Rs. 250.
vii) Provision is to be made for depreciation on Plant and machinery and on Motor car at
the rate of 10% p.a. on cost.
viii) A bad-debts provision, specially attributable to the second half of the year, is to be
made @ 5% on debtors as on 31st Ashadh 2076.
ix) Amount payable to A on retirement remained unpaid till 31st Ashadh 2076.
Required: (20 marks)
i) Prepare Trading and Profit & Loss Account for the year ended 31st Ashadh 2076.
ii) Prepare Partners' Capital Accounts for the year ended 31st Ashadh 2076.
iii) Prepare Balance Sheet as on that date.
Answer:
Trading and Profit and loss Account
For the year ended 31st Ashadh 2076
Particulars Rs. Rs.
Sales 1,20,000
Less: Cost of Goods Sold:
Opening Stock 15,500
Purchases 84,000
99,500
Less: Closing Stock 18,550 80,950
Gross Profit 39,050

Particulars Half year Half year


to 30th Poush 2075 to 31st Ashadh 2076
Rs. Rs. Rs. Rs.
Gross profit allocated on time basis 19,525 19,525
Less : Expenses
Salaries (W.N.1) 6,750 5,250
Travelling Expenses 400 400
Office Maintenance 600 600
Conveyance 250 250
Trade Expense (W.N.2) 625 625
Rent and rates ( W.N. 3) 1,200 1,200
Bad debts 500 400
Provision for doubtful debts - 270
Depreciation:
Plant and machinery 1,100 1,100
Motor vehicles 1,500 1,500
Interest on Loan ( W.N. 4) - (12,925) 1,638 (13,233)
Net profit 6,600 6,292
Apportionment of Profit:
Remaining profit
A and B (2:1) 4,400
2,200 6,600 3,775

© The Institute of Chartered Accountants of Nepal 5


Suggested Answer Paper Group I

B and C (3:2) 2,517 6,292

Partners' Capital Accounts


Dr. Cr.
Particulars A B C Particulars A B C
Rs. Rs. Rs. Rs. Rs. Rs.
To A (goodwill) - 4,000 6,000 By balance b/d 24,000 12,000
To Drawings 2,000 3,000 1,000 By Cash - - 9,000
To A's Loan A/c 36,400 - - By B 4,000
(Goodwill)
To Balance c/d - 10,975 4,517 By C 6,000
(Goodwill)
By Profit 4,400 5,975 2,517
38,400 17,975 11,517 38,400 17,975 11,517

Balance Sheet
as on 31st Ashadh 2076
Equity & Liabilities Amount Assets Amount
(Rs.) ( Rs.)
Capital A/c Property, Plant & Equipment:
B 10,975 Plant and Machinery 22,000
C 4,517 15,492 Less: Depreciation 6,600 15,400
A's Loan 36,400 Motor Car 30,000
Less : Depreciation 9,000 21,000
Current Liabilities: Current Assets:
Creditors 10,100 Stock 18,550
Outstanding Trade Exp. 250 Debtors 5,400 5,130
Less: Provision 270
Interest outstanding on A's 1,638 Prepaid Rent 600
Loan
Balance at bank 3,200
Total 63,880 Total 63,880

Working Notes:

S.N. Particulars Rs. Rs.


1. Salaries
Total as per trial balance 18,000
Less : Partners' Drawing 2,000
A
B 3,000
C 1,000 (6,000)
12,000
Less: C's Salary up to 30.09.2075 1,500
10,500
Upto 30.09.2075 Upto 31.03.2076
Allocation on time basis 5,250 5,250
Add: C's Salary upto 30.09.2075 1,500 0
6,750 5,250
2. Trade Expenses
Total as per trial balance 1,000
Add : Accrual 250

© The Institute of Chartered Accountants of Nepal 6


Suggested Answer Paper Group I

1,250
Allocation : on time basis (50:50) 625 625

3. Rent and rates


Total as per trial balance 3,000
Add : Rent paid in advance (600)
2,400
Allocation : on time basis (50:50) 1,200 1,200

4. Interest on loan account of "A'


Balance in Capital a/c as per trial 24,000
balance
Less : Drawings (2,000)
Add: Share of Goodwill 10,000
Share in Profit 4,400 14,400
36,400
Interest payable @9% p.a. from 36,400˟6/12˟9/100 1,638
01.10.2075 to 31.03.2076 (6 months)

Adjustment for A's share of Goodwill


Value of goodwill Rs. 15,000
Net entry for Goodwill:
B's Capital account Dr. Rs. 4,000
C's Capital account Dr. Rs. 6,000
To A's Capital account Rs. 10,000
(A's share in goodwill adjusted to existing partners in their gaining ratio)
2.
a) Nirman Limited commenced a contract on 1.7.2075. The total contract price was Rs.
5,00,000 but Nirman Limited accepted the same for Rs. 4,50,000. It was decided to
estimate the total profit and take to the credit of Profit & Loss Account, the proportion
of estimated profit on cash basis which the work completed bears to total contract
price. Actual Expenditure till 31.12.2075 and estimated expenditure in 2076 are given
below.
Expenses Actual till Estimated for
31.12.2075 (Rs.) 2076 (Rs.)
Materials 75,000 1,30,000
Labour 55,000 60,000
Plant purchased (original cost) 40,000 -
Miscellaneous expenses 20,000 35,000
Plant returned to stores at original cost On 31.12.2075 On 30.09.2076
10,000 25,000
Material at site 5,000 Nil
Work certified 2,00,000 Full
Work uncertified 7,500 Nil
Cash received 1,80,000 Full

© The Institute of Chartered Accountants of Nepal 7


Suggested Answer Paper Group I

The plant is subject to a annual depreciation @ 20% of original cost. The contract is
likely to be completed on 30.09.2076.
Required: (10 marks)
Prepare the contract account for the year ended 31.12.2075. It was the policy of
company to charge depreciation on time basis.
b) The following is the Balance Sheet of Mr. S as on Ashadh 32, 2075.
Liabilities Rs. Assets Rs.
Capital Account 480,000 Buildings 325,000
Loan 150,000 Furniture 50,000
Trade Creditors 310,000 Motor car 90,000
Stock 200,000
Trade Debtors 170,000
Cash in hand 20,000
Cash at bank 85,000
940,000 940,000
A fire occurred on the night of 31st Ashadh, 2076 in which all books and records were
lost. The cashier had escaped with the available cash. Mr. S gives you the following
information:
i) His sales for the year ended Ashadh 31, 2076 were 20% higher than the previous
years. He always sells his goods at cost plus 25%. 20% of the total sales for the
year ended Ashadh 31, 2076 was for cash. There were no cash purchases.
ii) On Shrawan 1, 2075 the stock level was raised to 300,000 and the stock was
maintained at this level throughout the year.
iii) Collection from Debtors amounted to Rs. 14 lakh of which Rs. 3.50 lakh was
received in cash. Business expenses amounted to Rs. 200,000 of which Rs. 50,000
was outstanding on Ashadh 31, 2076 and Rs. 60,000 was paid by cheques.
iv) Analysis of the Bank statements revealed on the following: Payment creditors Rs.
13.75 lakh, Personal drawings Rs. 75,000. Cash deposited in bank Rs. 7.15 lakh.
Cash withdrawn from bank Rs. 120,000.
v) Gross profit as per last year’s audited accounts was Rs. 300,000.
vi) Provide depreciation on building and furniture at 5% and on motor car at 20%.
vii) The amount defalcated by the cashier may be treated as recoverable from him.
Required: (10 marks)
i) Prepare Trading and Profit and Loss Account for the year ended Ashadh 31,
2076.
ii) Prepare Balance Sheet as on 31.03.2076.
Answer:

a)
Nirman Limited
Contract Account
For the year ending 31.12.2075
Particulars Amount Particulars Amount
To Materials 75,000 By Plant returned to stores 9,000

© The Institute of Chartered Accountants of Nepal 8


Suggested Answer Paper Group I

(Cost less Depreciation


W.N-3)
To Labour 55,000 By Plant at site (W.N-3) 27,000
To Plant 40,000 By Material at site 5,000
To Misc. Expenses 20,000 By Work in progress:
To P & L A/c (W.N-2) 26,600 Work certified 2,00,000
To Work in progress (Reserve) 31,900 Work uncertified 7,500
2,48,500 2,48,500

Working Note-1
Memorandum Contract Account
(1.7.2075 to 30.9.2076)
Particulars Amount Particulars Amount
To Materials (75,000+1,30,000) 2,05,000 By Plant returned to stores 27,750
(9,000+18,750)
To Labour (55,000+60,000) 1,15,000 By Plant at site (5,000- 3,750
1,250)
To Plant 40,000 By Contractee's A/c 4,50,000
To Misc. Expenses 55,000
(20,000+35,000)
To Estimated profit 66,500
4,81,500 4,81,500

Working Note-2
Profit to be transferred to Profit or Loss A/c of the contract ending on 31st Chaitra,
2075.
Cash Received Work certified
=𝐸𝐸𝐸𝐸𝐸𝐸𝐸𝐸𝐸𝐸𝐸𝐸𝐸𝐸𝐸𝐸𝐸𝐸 𝑝𝑝𝑝𝑝𝑝𝑝𝑝𝑝𝑝𝑝𝑝𝑝 × ×
𝑊𝑊𝑊𝑊𝑊𝑊𝑊𝑊 𝐶𝐶𝐶𝐶𝐶𝐶𝐶𝐶𝐶𝐶𝐶𝐶𝐶𝐶𝐶𝐶𝐶𝐶 𝑇𝑇𝑇𝑇𝑇𝑇𝑇𝑇𝑇𝑇 𝐶𝐶𝐶𝐶𝐶𝐶𝐶𝐶𝐶𝐶𝐶𝐶𝐶𝐶𝐶𝐶 𝑃𝑃𝑃𝑃𝑃𝑃𝑃𝑃𝑃𝑃

Rs.1,80,000 Rs.2,00,000
=66,500 × × = Rs. 26,600
𝑅𝑅𝑅𝑅.2,00,000 𝑅𝑅𝑅𝑅.4,50,000

Working Note-3
i) Computation of value of Plant returned to stores on 31.12.2075
Original cost Rs.10,000
Less: Depreciation @ 20% for six months Rs.1,000
Rs.9,000

ii) Plant at site on 31.12.2075


= Original cost of Plant- Plant returned – Depreciation
=Rs.40,000-Rs.10,000- Rs.3,000 = Rs27,000
iii) Plant returned to stores on 30.09.2076
Original cost Rs.25,000
Less: Depreciation for 15 months (Rs.25,000×20%×15/12) Rs.6,250
Rs.18,750

iv) Plant at site on 30.09.2076


Original cost Rs.5,000
Less: Depreciation for 15 months (Rs. 5,000×20%×15/12) Rs.1,250
Rs.3,750

© The Institute of Chartered Accountants of Nepal 9


Suggested Answer Paper Group I

b)
(i)

MR. S
Trading and Profit and Loss Account
for the year ended Ashadh 31, 2076

Particulars Rs. Particulars Rs.


To Opening Stock A/c 200,000 By Sales A/c (Note 4)
To Purchase A/c 1,540,000 Credit (80%) 14,40,000
(Balancing figure)
To Gross Profit c/d 360,000 Cash (20%) 360,000
(20% of Rs. 18,00,000)
By Closing Stock 300,000
21,00,000 21,00,000

To Business Expenses 200,000 By Gross Profit b/d 360,000


To Depreciation on:
Building 16,250
Furniture 2,500
Motor car 18,000 36,750
To Net Profit (transferred to 123,250
capital)
3,60,000 3,60,000

(ii)

Mr. S
Balance Sheet
As at Ashadh 31, 2076
Equity and Liabilities Rs. Assets Rs.
Capital:
Opening Balance 480,000 Buildings 325,000
Add: Net Profit 123,250 Less: Depreciation 16,250 308,750
Less: Drawings 75,000 528,250 Furniture 50,000
Loan 150,000 Less: Depreciation 2,500 47,500
Trade Payables 475,000 Motor Car 90,000
O/s business expenses 50,000 Less: Depreciation 18,000 72,000
Stock-in-trade 300,000
Trade Receivables 210,000
Cash at Bank 220,000
Amount due from 45,000
employee
1,203,250 1,203,250

Working notes:
(1)
Cash Book

© The Institute of Chartered Accountants of Nepal 10


Suggested Answer Paper Group I

Particulars Cash Bank Particulars Cash Bank


To Balance b/d 20,000 85,000 By Business 90,000 60,000
Expenses
To Sales (Note 4) 360,000 - By Drawings - 75,000
To Trade 350,000 1,050,000 By Trade payables - 1,375,000
Receivables
To Cash (C) - 715,000 By Bank (C) 715,000 -
To Bank (C) 120,000 - By Cash (C) - 120,000
By Balance c/d 45,000* 220,000
8,50,000 18,50,000 8,50,000 18,50,000
*Recoverable from Cashier
(2)
Trade Receivables Account
Particulars Rs. Particulars Rs.
To Balance b/d 170,000 By Bank (14,00,000 - 1,050,000
3,50,000)
To Sales (Note 4) 14,40,000 By Cash 350,000
By Balance c/d 2,10,000
1,610,000 1,610,000
(3)
Trade Payables Account
Particulars Rs. Particulars Rs.
To Bank A/c 1,375,000 By Balance b/d 310,000
To Balance c/d 475,000 By Purchase A/c(Note 5) 1,540,000
1,850,000 1,850,000

(4) Computation of Total Sales: Rs.

Last year’s gross profit @ 20% on sales (cost +25%) 300,000


Last year’s sales (300,000 x 5) 1,500,000
Current year’s sales (1,500,000 x 120%) 1,800,000
Gross Profit: 20% of Sales 360,000
Cash Sales: 20% of Total Sales 360,000
Credit Sales: 80% of Total Sales 1440,000
(5) Calculation of Purchase:
Purchase= (Sales + Closing Stock) – (Opening Stock + Gross Profit)
= (1,800,000+300,000) – (200,000+20% of 1,800,000)
= (2,100,000-560,000) = 1,540,000
3.
a) The summarized Balance Sheet of A Ltd. and B Ltd. as at 01.04.2076 are as follows:
Liabilities A Ltd. B Ltd. Assets A Ltd. B Ltd.

© The Institute of Chartered Accountants of Nepal 11


Suggested Answer Paper Group I

Equity shares of 3,55,000 3,05,000 Plant and 4,05,000 4,55,000


Rs. 10 each machinery
11% Preference - 2,05,000 Stock 1,55,000 85,000
shares of Rs. 10
each
12% Debenture 1,05,000 1,55,000 Sundry 1,80,000 1,25,000
debtors
Profit and loss 1,00,000 - Cash and 80,000 1,05,000
A/c bank
Reserve 1,75,000 -
Sundry creditors 85,000 1,05,000
8,20,000 7,70,000 8,20,000 7,70,000
The above two companies agree to amalgamate and form a new company AB Limited
on the following conditions.
For A Ltd.:
i) For every 5 equity shares, 6 share of AB Ltd. of Rs. 10 each will be issued at par.
ii) Debenture holders will be issued 12% debentures of AB Ltd. of same amount and
denominations.
For B Ltd.:
i) The holders of 11% preference share will be allotted 4, 13% preference shares of
Rs. 10 each of AB Ltd. for every 5 preference shares held.
ii) For every 5 equity shares, 6 shares of AB Ltd. of Rs. 10 each will be issued at
par.
iii) Debenture holder will be issued 12% debentures of AB Ltd. of same amount and
denomination.
Required: (10 marks)
i) Calculation of purchase consideration.
ii) Prepare Ledger accounts to close the books of A Ltd. and B Ltd.
b) Moon Ltd. was incorporated on 1st Kartik, 2075 to acquire running business of Feel
goods with effect from 1st Shrawan, 2075. During the year 2075-76, the total sales
were Rs. 48,00,000 of which Rs. 9,60,000 were for the first six months. The Gross
profit of the company Rs. 7,81,600. The expenses debited to the Profit & Loss Account
included:
i) Director's fees Rs. 60,000.
ii) Bad debts Rs. 14,400.
iii) Advertising Rs. 48,000 (under a contract amounting to Rs. 4,000 per month).
iv) Salaries and General Expenses Rs. 2,56,000.
v) Preliminary Expenses written off Rs. 20,000.
vi) Donation to political party given by the company Rs. 20,000.
Required: (5 marks)
Prepare a statement showing pre-incorporation and post-incorporation profit for the
year ended 31st Ashadh, 2076.

© The Institute of Chartered Accountants of Nepal 12


Suggested Answer Paper Group I

Answer:
a)
Calculation of Purchase Consideration
S.N Particulars Calculation Rs.
(i) A Ltd
Equity Shareholders 35,500/5×6×Rs. 10 Rs.4,26,000
(ii) B Ltd
a 11% Preference shareholders 20,500/5×4×Rs.10 Rs.1,64,000
b Equity Shareholders 30,500/5×6×Rs.10 Rs.3,66,000
Total Rs.5,30,000

Dr In the books of A Ltd Cr


Realization Account
Particulars Rs. Particulars Rs.
To Plant and Machinery 4,05,000 By 12% Debenture 1,05,000
To Stock 1,55,000 By Sundry Creditors 85,000
To Sundry Debtors 1,80,000 By AB Ltd 4,26,000
(Purchase consideration)
To Cash and Bank 80,000 By Sundry Equity 2,04,000
shareholders A/c
Total 8,20,000 Total 8,20,000
Dr AB Ltd Cr
Particulars Rs. Particulars Rs.
To Realization 4,26,000 By Equity Share in AB Ltd 4,26,000
Total 4,26,000 Total 4,26,000
Dr Equity share in AB Ltd Cr
Particulars Rs. Particulars Rs.
To AB Ltd 4,26,000 By Sundry Equity shareholders 4,26,000
Total 4,26,000 Total 4,26,000
Dr Sundry equity shareholders A/c Cr
Particulars Rs. Particulars Rs.
To Realization 2,04,000 By Equity Share Capital 3,55,000
To Equity shares in AB Ltd 4,26,000 By Profit and Loss 1,00,000
By Reserve A/c 1,75,000
Total 6,30,000 Total 6,30,000
Dr In the books of B Ltd, Cr
Realization Account
Particulars Rs. Particulars Rs.
To Plant and Machinery 4,55,000 By 12% Debenture 1,55,000
To Stock 85,000 By Sundry Creditors 1,05,000
To Sundry Debtors 1,25,000 By AB Ltd 5,30,000
(Purchase consideration)
To Cash and Bank 1,05,000 By Preference shareholders 41,000
A/c ( Discount on payment)
To Sundry equity shareholders ( 61,000
Profit)
Total 8,31,000 Total 8,31,000

Dr AB Ltd Cr
Particulars Rs. Particulars Rs.
To Realization 5,30,000 By Equity Share in AB Ltd 3,66,000
By Preference shares in AB Ltd
1,64,000
Total 5,30,000 Total 5,30,000

Dr Preference share in AB Ltd Cr


Particulars Rs. Particulars Rs.

© The Institute of Chartered Accountants of Nepal 13


Suggested Answer Paper Group I

To Preference shares in AB Ltd 1,64,000 By 11% Preference share capital 2,05,000


To Realization
41,000
Total 2,05,000 Total 2,05,000

Dr Sundry Equity shareholders A/c Cr


Particulars Rs. Particulars Rs.
To Equity shares in AB Ltd 3,66,000 By Equity Share Capital 3,05,000
By Realization (Profit on 61,000
realization)
Total 3,66,000 Total 3,66,000

b) Statement showing the calculation of Profits for the pre-incorporation and post-
incorporation periods for the year ended 31st Ashadh, 2076
Particulars Total Basis of Pre- Post-
Amount Allocation incorporation incorporation
Gross Profit 7,81,600 Sales 78,160 7,03,440
Less: Directors' fee 60,000 Post 60,000
Bad debts 14,400 Sales 1,440 12,960
Advertising 48,000 Time 12,000 36,000
Salaries & general expenses 2,56,000 Time 64,000 1,92,000
Preliminary expenses 20,000 Post 20,000
Donation to Political party 20,000 Post 20,000
Net Profit 3,63,200 720 3,62,480

Working Notes:
1. Sales Ratio
Particulars Rs.
Sales for period up to 30.06.2075 ( 9,60,000˟ 3/6) 4,80,000
Sales for period from 01.07.2075 to 31.03.2076 ( 48,00,000- 4,80,000) 43,20,000

Thus, Sales Ratio = 1: 9


2. Time Ratio
1st Shrawan, 2075 to 30 Ashoj , 2075 : 1st Kartik 2075 to 31st Ashadh , 2076
= 3 months : 9 months = 1: 3
Thus Time Ratio is 1: 3

4.
a) The following information furnished to you by Lumbani Insurance Co. Ltd.;
i) On 1.04.2075, it had reserve for unexpired risk amounting to Rs. 55 crores. It
comprised of Rs. 21 crores in respect of marine insurance business, Rs. 28 crores
in respect of fire insurance business and Rs. 6 crores in respect of miscellaneous
insurance business.
ii) Lumbani Insurance Co. Ltd. creates reserves at 100 % of net premium income in
respect of marine insurance policies and at 50% of net premium income in respect
of fire and miscellaneous income policies.
iii) During 2075/76, the following business was conducted:
Particulars (Rs. in crores)
Marine Fire Miscellaneous

© The Institute of Chartered Accountants of Nepal 14


Suggested Answer Paper Group I

Premium collected from:


a) Insured in respect of policies issued 22.00 46.00 13.00
(Direct Business)
b) Other insurance companies in
respect of risks undertaken:
Received during the year 11.5 9.2 5.5
Receivable- 01.04.75 7.0 3.0 1.5
Receivable- 31.03.76 4.0 1.0 1.0
Premium paid/payable to other 7.5 5.3 8.0
insurance companies on business ceded

Required: (10 marks)


Pass Journal entries relating to unexpired risk reserve and show in columnar form
"Unexpired Risk Reserve Account" for 2075/2076.
b) ABC Ltd. has provided following information to you:
Current Ratio=2.8
Acid-test Ratio=1.5
Working Capital=Rs. 1,62,000
Required to calculate: (5 marks)
i) Current Assets
ii) Current Liabilities
iii) Liquid Assets
Answer:
a)
Date Particulars Rs. in crores
Dr. Cr.
1.4.2075 Unexpired Risk Reserve (Fire) A/c Dr 28
Unexpired Risk Reserve (Marine) A/c Dr 21
Unexpired Risk Reserve (Miscellaneous) A/c 6
Dr
To Fire Revenue Account 28
To Marine Revenue Account 21
To Miscellaneous Revenue Account 6
(Being unexpired risk reserve brought forward from last
year)
31.03.2076 Marine Revenue A/c Dr. 23
To Unexpired Risk Reserve A/c 23
(Being closing reserve for Unexpired Risk created at 100%
of net premium income amounting to Rs. 23 crores i.e.
22+8.5-7.5)
Fire Revenue A/c Dr. 23.95
To Unexpired Risk Reserve A/c 23.95
(Being closing reserve for Unexpired Risk created at 50%
of net premium income amounting to Rs. 47.90 crores i.e.
46+7.2-5.3)
Miscellaneous Revenue A/c Dr. 5
To Unexpired Risk Reserve A/c 5

© The Institute of Chartered Accountants of Nepal 15


Suggested Answer Paper Group I

(Being closing reserve for Unexpired Risk created at 100%


of net premium income amounting to Rs. 10 crores i.e.
13+5-8)

Unexpired Risk Reserve Account


Date Particulars Marine Fire Misc Date Particulars Marin Fire Misc.
. e
1.4.2075 To Revenue A/c 21 28 6 1.4.2075 By Balance b/d 21 28 6
31.3.207 To balance c/d 23 23.95 5 31.3.207 By Revenue 23 23.95 5
6 6 A/c
44 51.95 11 44 51.95 11

Working Notes:
Premium from other insurance companies in respect of risk undertaken
Received during the year 11.5 9.2 5.5
Less : Receivable – 01.04.75 (7.0) (3.0) (1.5)
Add : Receivable – 31.03.76 4.0 1.0 1.0
8.5 7.2 5.0
b) Current Ratio=2.8, it means if current assets are 280, current liabilities are 100.
Current Ratio=Current Assets /Current Liabilities
=280/100=2.8
Working Capital=Current Assets – Current Liabilities=2.8 – 1=1.8
Working capital=1,62,000
In such a case, if 1.8 is Rs. 1,62,000 current Assets (2.8) would be 2,52,000 and
current liabilities (1) would be Rs. 90,000.
Acid Test Ratio or Liquid Ratio =Liquid assets/Current liabilities
Liquid assets = current assets – stock
Liquid ratio is 1.5. It means if current liabilities are 1, liquid assets are 1.5.
Current liabilities are Rs. 90,000, so liquid assets are Rs. 1,35,000.
Acid Test Ratio or Liquid Ratio =135,000/90,000 = 1.5
i) Current Assets=Rs. 2,52,000
ii) Current Liabilities=Rs. 90,000
iii) Liquid Assets=Rs 1,35,000
5.
a) A Ltd. manufactures plastic water tanks for the farming industry. On 31st Ashadh
2076, its closing inventory consisted of 950 kg of plastic resin raw material, and also
250 finished units (plastic water tanks). Further information is provided as follows:
i) Plastic: The purchase price of plastic resin was Rs. 3 per kg throughout the year
to 31st Ashadh 2076, plus an additional Rs. 0.50 per kg of delivery cost. A Ltd.
has a policy of always keeping plenty of plastic resin in inventory, as its supply
can be unreliable. However, close to the year-end, the price of plastic resin
reduced due to supply exceeding demand. The purchase price of A’s raw material

© The Institute of Chartered Accountants of Nepal 16


Suggested Answer Paper Group I

is now Rs. 2.10 per kg plus the Rs. 0.50 per kg delivery charge. The existing
inventory of plastic resin can be sold in the market for Rs. 1.80 per kg net of all
costs.
ii) Tanks: Each tank requires 10 kg of plastic to manufacture, and each unit incurs
Rs. 25 in conversion costs (labour and overhead). A Ltd. sells the tanks for Rs.
100. It is expected that this price will drop to Rs. 90 as a result of the fall in the
market price of plastic. All completed units sold by A Ltd. incur a Rs. 6 selling
and distribution cost.
Required: (5 marks)
Calculate the value of closing inventory in the books of A Ltd. at 31st Ashadh 2076
applying the principles of NAS 2: Inventories.
b) M/s Dhurmus & Suntali Co. noted in 2073-74 that in 2072-73 it had omitted to record
a depreciation expense on an asset amounting to Rs. 600. Its accounts before the
correction of the error are;
2073-74 Rs. '000 2072-73 Rs. '000
Gross profit 6,000 6,900
Distribution costs (600) (600)
Administration expenses (1,800) (1,800)
Depreciation (600) Nil
Profit from operations 3,000 4,500
Income tax (600) (900)
Net profit 2,400 3,600
The company’s retained earnings (income surplus) for the two years before the
correction of the error were:
Retained earnings carried forward 6,900 4,500
Retained earnings brought forward 4,500 900
Required: (5 marks)
Describe how the above error should be corrected in accordance with Nepal
Accounting Standard.
c) ABC Ltd. has entered into a contract for the provision of services over a two-year
period. The total contract price is Rs. 150,000. In the first year costs of Rs. 60,000
have been incurred and 50% of the work has been completed. The contract has not
progressed as expected and ABC Ltd. is not sure of the ultimate outcome, but believes
that the costs incurred to date will be recovered from the customer. ABC Ltd. initially
expected to earn a profit of Rs. 20,000 on the contract. Company is struggling with
the amount of revenue to be recognized in the first year of contract. Suggest the right
amount of revenue to be recognized, clearly indicating the provision of relevant
standard. (5 marks)
Answer:
a) The inventory of A Ltd should be valued as follows:
Finished goods:
Cost per unit: Rs.
Material – 10kg *Rs. 3 30

© The Institute of Chartered Accountants of Nepal 17


Suggested Answer Paper Group I

Conversion 25
Total 55
Net realisable value:
Expected selling price 90
Less selling costs estimate (6)
Net realisable value 84
As the Net Realisable Value exceeds the cost, the finished goods are valued in the
books at cost. Hence a value of 250 *Rs. 55 =Rs. 13,750 will be entered into the books
as closing inventory of finished goods.
Raw Material: Rs.
Cost per unit:
Purchase price 3.00
Delivery costs 0.50
Total cost 3.50
Net realisable value:
Expected sale proceeds if sold as inventory 1.80
Expected sale proceeds if sold as finished units 90
Less selling cost (6)
Less completion costs: Conversion (25)
Net realisable value 59
NRV per kg of raw material Rs.5.90
The NRV of the raw material if sold as raw material is lower than the cost. However
the NRV if processed into finished units is higher than cost. Therefore the inventory
should not be written down, and should be recorded in the books at cost. Hence a value
of 950 *Rs.3.50 =Rs.3,325 will be entered into the books as closing inventory.
Total closing inventory = 3,325 + 13,750 =Rs.17,075.
b) NAS 8, Accounting Policies, Changes in Accounting Estimates and Errors states that
the correction of an error that relates to prior periods should be shown as an adjustment
in the opening balance of retained earnings. As a result, in 2072/73 accounts (ignoring
all tax implications):
Dr. retained earnings brought forward 600
Cr. Accumulated depreciation 600
It is important to notice that this will have no impact on the current income statement
but is shown as a prior period adjustment in the statement of changes in equity in
2073/74.
Retained earnings brought forward as reported previously 4,500
Prior year adjustment to correct error (600)
Retained earnings, beginnings as restated 3,900

© The Institute of Chartered Accountants of Nepal 18


Suggested Answer Paper Group I

Profit for the year 2,400


Retained earnings carried forward 6,300
c) As per NAS 18, Revenue, the revenue from rendering of services shall be recognized
on the basis of stage of completion. However, when outcome of the transaction
involving the rendering of services cannot be estimated reliably, revenue shall be
recognized only to the extent of the cost incurred that are recoverable.
In the given case, it is clear that the company is not in position to estimate the outcome
of service contract and the company expects that the costs incurred to date will be
recovered. So, as per NAS 18, Revenue, the company shall recognize revenue to the
extent of cost incurred i.e. Rs. 60,000.
6. Write short notes on: (5×3=15 marks)
a) Reportable segment
b) Substance over form
c) Sinking fund
d) Non-banking asset and its treatment
e) Prepackaged accounting software
Answer:
a) Reportable segment is a business segment or a geographical segment identified as
reportable segment for which segment information is required to be disclosed.
A business or geographical segment shall be identified as a reportable segment if a
majority of its revenue is earned from sale to external customers and:
• its revenue from sales to external customers and from transactions with other
segments is 10 percent or more of the total revenue, external or internal, of all
segments; or
• its segment result, whether profit or loss, is 10 percent or more of the combined
result of all segments in profit or loss, whichever is the greater in absolute amount;
or
• its assets are 10 percent or more of the total assets of all segments.
b) Substance over form is one of the qualitative characteristics of financial statements
which has been mentioned in the Framework for the preparation and presentation of
financial statements. If information is to represent faithfully the transactions and other
events that it purports to represent, it is necessary that they are accounted for and
presented in accordance with their substance and economic reality and not merely their
legal form. The substance of transactions or other events is not always consistent with
that which is apparent from their legal or contrived form. For example, an entity may
dispose of an asset to another party in such a way that the documentation purports to
pass legal ownership to that party; nevertheless, agreements may exist that ensure that
the entity continues to enjoy the future economic benefits embodied in the asset. In
such circumstances, the reporting of a sale would not represent faithfully the
transaction entered into.
c) It refers to a fund created for redemption of a liability or replacement of an asset. The
money of the fund is invested in securities outside the business. Such securities are
known as ‘sinking fund investments’. The objective of creating such fund is to avoid
© The Institute of Chartered Accountants of Nepal 19
Suggested Answer Paper Group I

any financial hardship to the company in the event of these eventualities. The sinking
fund may be created for any of the following purposes:
i) For redemption of a liability
ii) For replacement of an asset
d) Bank & financial institutions may, in the case of non-realization of loan, recover the
outstanding principal and interest by way of disposing the collateral property. Such
property may be assumed by the institution itself if the same could not be sold in
auction, known as non-banking asset. Such asset shall be accounted at the prevailing
market value of the property or total receivable amount from borrower (i.e.
outstanding principal, interest, penal interest and any other receivable amount from
borrower), whichever is lower.
In case, if the market value of such property is lower than the total receivable amount,
the difference amount shall be immediately charged to Profit & Loss Account.
As per the directives issued by Nepal Rastra Bank, 100 percent provision shall be
made for such asset in the year in which such property has been transferred in the
name of bank/financial institutions.
e) There are several prepackaged accounting software which are available in the market
and are used extensively for small and medium sized organizations. These software
are easy to use, relatively inexpensive and readily available.
The installation of these software is very simple. An installation diskette or CD is
provided with the software which can be used to install the software on a personal
computer. A network version of this software is also generally available which needs
to be installed on the server and work can to perform from the various workstations or
nodes connected to the server. Along with the software a user manual is provided
which guides the user on how to use the software.
After installation of the software, the user should check the version of the software to
ensure that they have been provided with the latest. The vendor normally provides
regular updates to take care of the changes of law as well as add features to existing
software.

© The Institute of Chartered Accountants of Nepal 20


Suggested Answer Paper Group I

Paper 2: Audit and Assurance


Attempt all questions.
1. As an auditor, give your opinion with explanations on the following cases: (4×5=20 marks)
a) Ax Ltd. sold the flats to Ke Ltd. for Rs. 30 lakhs on 30/10/2075 and gave possession of
the property to Ke Ltd. However, Malpot documentation and legal formalities are
pending. Due to this, the company has not recorded the sale and has shown the amount
received as an advance. The book value of the building is Rs. 25 lakhs as on Ashadh
end 2076.
b) Agri Nepal Pvt. Ltd. has a business of agricultural farm. The Government of Nepal has
provided grant of Rs. 10 lakhs against the bank guarantee with a condition that the
company has to export its product worth of Rs. 50 lakhs per year in next two fiscal
year. Guide the accountant of the company for the accounting of the grant.
c) TCIN Pvt. Ltd. has been fighting a legal case since last three years. The legal team of
the company has assured that company will win the case. During the year some new
facts have developed and these facts are not in favor of the company. If the company
loses the case it has to pay Rs. 25 lakhs to other party. The company has approached
to you for dealing with the situation.
d) You are the auditor of Nepal Fashion Pvt. Ltd. (the company) for the FY 2075/076.
Total number of shareholder of the company since financial year 2074/075 are 115
which includes 11 employees.
Answer:
a) Principles of prudence, substance over form and materiality should be looked into, to
ensure true and fair consideration in a transaction. In the given case, the economic
reality and substance of the transaction is that the rights and beneficial interest in the
property has been transferred although legal title has not been transferred. Hence, Ax
Ltd. should record the sale and recognize the profit in its financial statements for the
year ended Ashadh 2076; value of building should be removed from the balance sheet.
Therefore, the treatment given by the company is not correct.
b) As per NAS 20, Accounting for Government Grants and Disclosure of Government
Assistance, government grants shall not be recognized until there is reasonable
assurance that:
• The entity will comply with the condition attaching to them and
• The grants shall be received.
Receipt of grant does not itself provide conclusive evidence that the conditions
attaching to the grant have been or will be fulfilled.
In the given case, the Government of Nepal has provided grant of Rs. 10 lakhs with the
condition that the company need to export its produce worth of Rs. 50 lakhs in next two
years. The grant received shall only be recognized as income if the company can export
Rs. 50 lakh in next two years. In this case, the government grant shall be recognised in
profit and loss on systematic basis over the periods in which the entity recognises as
expenses the related costs for which the grants are intended. So the company shall book
Rs. 5 lakhs as income in first year if it has exported Rs. 50 lakhs. If it has reasonable

© The Institute of Chartered Accountants of Nepal 21


Suggested Answer Paper Group I

assurance that it will be able to export Rs. 50 lakhs in second year it can book income
of next Rs. 5 lakhs in second year.
c) As per NAS 37, Provisions, Contingent Liabilities and Contingent Assets, a provision
shall be recognized when:
• An entity has a present obligation as a result of past obligation;
• It is probable that outflow of resources embodying economic benefits will be
required to settle the obligation and
• A reliable estimate can be made of the amount of the obligation.
If these conditions are not met, no provision shall be recognized.
In the given case, the company was fighting a legal case since last three years. During
the year, new facts were developed against the company. Since the new facts are not in
favour of the company, it has to make provision of Rs. 25 lakh in profit and loss account
because it has met all the condition of recognition of provision.
d) Pursuant to Section 9 of the Companies Act (with amendment 2074) a private company
cannot have more than One Hundred and One (101) shareholders. A private company
can now have a maximum of 101 shareholders. It is tradition to define the companies
as public companies or private companies in terms of the minimum or maximum
number of promoters or shareholders. Though the Companies Act and the Amendment
still retain this conventional concept, the Amendment has provided flexibility to the
private companies in terms of the maximum number of the shareholders. The threshold
of maximum number of the shareholders does not apply in case of the employee
shareholders.
In the light of foresaid changes of Companies Act, 2063 (amended 2074). The
maximum shareholders to be maintained for FY 2074/075 & 2075/076 is 101
(excluding employees shareholders). The shareholders of the company excluding
employees shareholders for FY 2074/075 and 2075/076 are 104. Accordingly the
company has breached the provision of amended Company's Act 2063. As the auditor,
I would like to suggest the company to regularize it without any further delay.
2. Give your comments on the following cases: (4× 5=20 marks)
a) Inland Expeditions Ltd. is a travel company engaged in the trekking of tourists. Dara
and Associates has been appointed as the auditors to carry out its audit. Auditor thinks
that planning an audit would involve establishing the overall audit strategy for the
engagement and developing an audit plan. Also, adequate planning benefits the audit
of financial statements in several ways.
b) Mr. HKA, Chartered Accountants has used the expert’s work on the audit of Biotech
Company Ltd. In the audit report he wants to use the reference of the expert's work.
c) The statutory auditor of Nara Pvt. Ltd. was not able to get the confirmation about the
existence and value of certain machineries. However, the management gave him a
certificate proving the existence and value of the machinery as appeared in the books
of account. The auditor accepted it without carrying out any further procedure and
signed the audit report.
d) Mrs. Y, a statutory auditor of MNP Ltd., wants to verify cash on hand on 31st Ashadh,
2076. The management informs auditor that it is not possible to co-operate, as cashier
has been hospitalized.

© The Institute of Chartered Accountants of Nepal 22


Suggested Answer Paper Group I

Answer:
a) Planning an audit involves establishing the overall audit strategy for the engagement
and developing an audit plan. Adequate planning benefits the audit of financial
statements in several ways, including the following:
a. Helping the auditor to devote appropriate attention to important areas of the audit.
b. Helping the auditor identify and resolve potential problems on a timely basis.
c. Helping the auditor properly organize and manage the audit engagement so that it
is performed in an effective and efficient manner.
d. Assisting in the selection of engagement team members with appropriate levels of
capabilities and competence to respond to anticipated risks, and the proper
assignment of work to them.
e. Facilitating the direction and supervision of engagement team members and the
review of their work.
f. Assisting, where applicable, in coordination of work done by auditors of
components and experts.
b) As per NSA 620, Using the Work of an Auditor's Expert, when issuing an unmodified
auditor’s report, the auditor should not refer the work of an expert. Further as a result
of the work of expert, the auditor decides to issue a modified auditor’s report in some
circumstances it may be appropriate in explaining the nature of the modification, to
refer to or describe the work of the expert (including the identity of the expert and the
extent of the expert’s involvement). In these circumstances, the auditor would obtain
the permission of the expert before making such a reference. If permission is refused
and the auditor believes a reference is necessary, the auditors may need to seek legal
advices.
In the given case, if unmodified report is being issued then the reference of the expert
work cannot be used. If modified audit report is being issued as a result of the work of
the expert, then auditor may use the reference of expert work only after the permission
from the expert. If the permission is denied by the expert then the auditor believes a
reference is necessary, the auditors may need to seek legal advices and do accordingly.
c) The physical verification of fixed assets is the primary responsibility of the
management. The auditor, however, is required to examine the verification programme
adopted by the management. He must satisfy himself about the existence, ownership
and valuation of fixed assets. In the case of Nara Ltd., the auditor has not been able to
verify the existence and value of some machinery despite the verification procedure
followed in routine audit. He accepted the certificate given to him by the management
without making any further enquiry. As per NSA 580, when representation relate to
matters which are material to the financial information, then the auditor should seek
corroborative audit evidence for other sources inside or outside the entity. He should
evaluate whether such representations are reasonable and consistent with other
evidences and should consider whether individuals making such representations can be
expected to be well informed on the matter. “Written Representations” cannot be a
substitute for other audit evidence that the auditor could reasonably expect to be
available. If the auditor is unable to obtain sufficient appropriate audit evidence that he
believes would be available regarding a matter, which has or may have a material effect
on the financial information, this will constitute a limitation on the scope of his

© The Institute of Chartered Accountants of Nepal 23


Suggested Answer Paper Group I

examination even if he has obtained a representation from management on the matter.


Therefore, the approach adopted by the auditor is not correct.
d) Limitation on the scope of Audit
The scope of audit may be limited for varied reasons:
i) The entity may impose restriction on scope of audit.
ii) The limitation may be imposed by circumstances.
When the audit is carried out as per law, the auditor should not accept the assignment
when his duties are curtailed by agreement, unless required by any law.
When audit is carried out in accordance with the entity’s terms voluntarily, the audit
may indicate his scope in his audit report.
Sometimes, the circumstances may impose restrictions on audit scope. For example, if
the auditor is appointed after the year end, he may not be able to participate in inventory
checking. Likewise, the records required may not be available so that the auditor may
not be able to check the details in the manner he liked. Such limitations in scope may
warrant an auditor to express disclaimer of opinion or qualified opinion in his audit
report depending upon the circumstances. In the given case, since cashier is
hospitalized, management says its inability to cooperate with auditor.
In general, non-cooperation of MNP limited will amount to limitation on scope of
auditors. However, the auditor should try to employ other alternative audit procedures
to verify cash. Auditor further should consider materiality of the fact and conclude
accordingly.
3. Answer the following: (3×5=15 marks)
a) Explain the relationship between Materiality and Audit Risk.
b) What are the factors that may assist the auditor in determining whether external
confirmation procedures are to be performed as substantive audit procedures while
responding the assessed risks?
c) Explain the basic elements of an assurance engagement.
Answer:
a) In conducting an audit of financial statements, the overall objectives of the auditor are
to obtain reasonable assurance about whether the financial statements as a whole are
free from material misstatement, whether due to fraud or error, thereby enabling the
auditor to express an opinion on whether the financial statements are prepared, in all
material respects, in accordance with an applicable financial reporting framework; and
to report on the financial statements, and communicate as required by the NSAs, in
accordance with the auditor’s findings.
The auditor obtains reasonable assurance by obtaining sufficient appropriate audit
evidence to reduce audit risk to an acceptably low level. Audit risk is the risk that the
auditor expresses an inappropriate audit opinion when the financial statements are
materially misstated. Audit risk is a function of the risks of material misstatement and
detection risk. Materiality and audit risk are considered throughout the audit when:
1. Identifying and assessing the risks of material misstatement;
© The Institute of Chartered Accountants of Nepal 24
Suggested Answer Paper Group I

2. Determining the nature, timing and extent of further audit procedures; and
3. Evaluating the effect of uncorrected misstatements, if any, on the financial
Statements and in forming the opinion in the auditor’s report.
b) Para A51 of NSA 330, The Auditor’s Responses to Assessed Risks deals with the factors
that may assist the auditor in determining whether external confirmation procedures are
to be performed as substantive audit procedures while responding to the assessed risks.
Such factors include:
a. The confirming party’s knowledge of the subject matter – responses may be more
reliable if provided by a person at the confirming party who has the requisite
knowledge about the information being confirmed.
b. The ability or willingness of the intended confirming party to respond – for
example, the confirming party:
• May not accept responsibility for responding to a confirmation request;
• May consider responding too costly or time consuming;
• May have concerns about the potential legal liability resulting from responding;
• May account for transactions in different currencies; or
• May operate in an environment where responding to confirmation requests is
not a significant aspect of day-to-day operations.
In such situations, confirming parties may not respond, may respond in a casual manner
or may attempt to restrict the reliance placed on the response.
The objectivity of the intended confirming party – if the confirming party is a related
party of the entity, responses to confirmation requests may be less reliable.
c) The assurance engagement has the following basic elements:
i) Three party relationship:
Assurance engagements involve three separate parties: a practitioner, a responsible
party and intended users.
ii) An appropriate subject matter:
Subject matter information may be the reorganization, measurement, presentation
and disclosers represented in financial statements, key indicators, special
documents, assertion about effectiveness or statement of compliance depending on
the nature of assurance engagements.
iii) Suitable criteria:
Criteria can be formal or informal depending on the nature of assurance
engagement. It may be Nepal Standards on Auditing or an established internal
control framework or individual control objectives specifically designed for the
engagement or applicable law, regulation or contract.
iv) Sufficient appropriate evidence:
The practitioner plans and performs an assurance engagement with an attitude of
professional scepticism to obtain sufficient appropriate evidence about whether the
subject matter information is free of material misstatement. The practitioner

© The Institute of Chartered Accountants of Nepal 25


Suggested Answer Paper Group I

considers materiality, assurance engagement risk, and the quantity and quality of
available evidence when planning and performing the engagement.
v) Assurance report:
The practitioner provides a written report containing a conclusion that conveys the
assurance obtained about the subject matter information.
4. Answer/Comment on the following: (3× 5=15 marks)
a) Gayatri Associates, Chartered Accountants has been appointed by Gandaki Bank
Limited to provide merger consulting service. The Bank has agreed to pay fee of Rs. 1
crore if consultant can fix merger with any top 10 banks. Comment on this in the light
of ICAN Code of Ethics.
b) CA. Ram is the practicing chartered accountant, proprietor of Ram & Associates.
Rajdhani Ltd., his ex-audit client asked his opinion regarding application of Nepal
Financial Reporting Standards in certain financial components. What will be the duty
of CA. Ram in this situation?
c) What are the factors that could impair the firm's independence for the engagements?
Answer:
a) In the given case, Bank is offering contingent fees which are calculated on a
predetermined basis relating to the outcome of a transaction or the result of the services
performed by the firm.
A contingent fee charged directly or indirectly, for example through an intermediary,
by a firm in respect of an audit engagement creates a self-interest threat that is so
significant that no safeguards could reduce the threat to an acceptable level.
Accordingly, a firm shall not enter into any such fee arrangement.
A contingent fee charged directly or indirectly, for example through an intermediary,
by a firm in respect of a non-assurance service provided to an audit client may also
create a self-interest threat. The threat created would be so significant that no safeguards
could reduce the threat to an acceptable level if:
(a) The fee is charged by the firm expressing the opinion on the financial statements
and the fee is material or expected to be material to that firm;
(b) The fee is charged by a network firm that participates in a significant part of the
audit and the fee is material or expected to be material to that firm; or
(c) The outcome of the non-assurance service, and therefore the amount of the fee, is
dependent on a future or contemporary judgment related to the audit of a material
amount in the financial statements.
Accordingly, such arrangements shall not be accepted.
b) Section 230 (1 to 3) of ICAN Code of Ethics deals with the matter. The duty of CA.
Ram on such situation would be :
i) Ensuring the existence and significance of any threat will depend on the
circumstances of the request and all the other available facts and assumptions
relevant to the expression of a professional judgment.
ii) Evaluate the significance of any threats and apply safeguards when necessary to
eliminate them or reduce them to an acceptable level.

© The Institute of Chartered Accountants of Nepal 26


Suggested Answer Paper Group I

iii) Seeking client permission to contact the existing accountant, describing the
limitations surrounding any opinion in communications with the client and
providing the existing accountant with a copy of the opinion.
iv) If the company or entity seeking the opinion will not permit communication with
the existing accountant, a professional accountant in public practice shall determine
whether, taking all the circumstances into account, it is appropriate to provide the
opinion sought.
c) Auditor while accepting the audit engagement should consider all factors that could
impair the firm's independence for his engagement including ICAN code of ethics.
Determine if any of the following challenges are present:
(a) Other services provided to the entity and its related entities in the previous years.
(b) Proposed or prospective services
(c) Firm financial or business relationships with the client
(d) Ex-firm staff working for the client in a position of influence
(e) Firm professionals who were formerly employed by the client with involvement
in the audit engagement
(f) Audit team's and other covered persons' family members employed by the client
(g) Long association of a senior team member with the client
(h) Acceptance of hospitality that is not clearly insignificant
(i) Threat of replacement over a disagreement with an accounting policy.
5. Answer/Comment on the following: (2× 5=10 marks)
a) Explain the procedures for appointment of auditor of corporate bodies wholly owned
by Government of Nepal, Provincial Government and Local Level.
b) CA. Ram Sharma is a partner of Ram & Shyam Associate, Chartered Accountants. CA.
Ram is nominated in the Board of Sagarmatha Securities Ltd. as professional director
with effect from 30th Jestha 2074 for three years. Subsequently his firm has been
appointed as auditor for the financial year 2075/76. Give your opinion on the validity
of appointing his firm as auditor in line with the provision of Securities Act.
Answer:
a) Section 10 of Audit Act 2075, deals with the provision for appointing Auditor of
Corporate Bodies Wholly Owned by Government of Nepal, Provincial government and
local level. The related provisions are:
1. As per Section 10 (1), the Auditor General may appoint license holder auditors
under the prevailing laws his assistant as required to conduct the audit of corporate
bodies wholly owned by as Government of Nepal, Provincial government and local
level.
2. The auditor appointed pursuant to this Section shall act under the direction,
supervision and control of the Auditor General.
3. As per section 10 (3), the powers, functions, duties and responsibilities of the
auditors appointed pursuant this Section and the procedures to be followed by them
in course audit and provisions relating to their report shall be as prescribed by the
Audit Act and the Auditor General.
© The Institute of Chartered Accountants of Nepal 27
Suggested Answer Paper Group I

4. As per section 10 (4), the remuneration to be paid by the concerned organization to


the auditors appointed pursuant to this Section shall be fixed by the Auditor General
keeping in view the volume of financial transactions, status of accounts, number of
branches and sub-branches, work load and work progress of the concerned
organization.
b) As per section 80 of the Securities Act 2063, following provision has been mentioned
regarding appointment of the auditor of the security company:
(1) A securities business person shall appoint an auditor from amongst the auditors
enlisted by the Board.
(2) Any director, shareholder, officer employee or partner of a securities business
company or body shall be deemed disqualified to be appointed as the auditor of
that securities business company or body.
(3) A securities business person shall give information, indicating the name and
address also of the auditor appointed by it, to the Board within seven days from
the date of such appointment.
(4) If a securities business person removes any auditor from office or such an auditor
resigns from office, the securities business person shall give information thereof
to the Board within seven days from the date of such removal.
(5) Notwithstanding anything contained elsewhere in this Act, no employee of a stock
exchange shall be eligible to be appointed as an auditor of any securities business
company or body.
In the light of the aforesaid provision of Securities Act 2063, Section 80 (2), since CA.
Ram (Partner of Audit Firm) is the professional director of Sagarmatha Securities Ltd.,
appointment of Ram & Shyam Associates; Chartered Accountants as auditor for FY
2075/76 is invalid.
6. Write short notes on the following: (4×2.5=10 marks)
a) Permanent Audit File
b) Statistical Sampling in Auditing
c) Uses of Negative External Confirmation Requests
d) Objectivity in an Audit.
Answer:
a) In a recurring audit, the file of working papers that are relevant to more than one audit
engagement or core documents are often kept separately in a file known as permanent
audit file. Permanent audit file is updated regularly with information of continuing
importance to succeeding audit. As per NSA 230, Audit Documentation a permanent
audit file normally includes:
i) Information concerning the legal and organizational structure of the client. In the
case of a company, this includes the memorandum and articles of association. In
the case of a statutory corporation, this includes the Act and Regulations under
which the corporation operates.

© The Institute of Chartered Accountants of Nepal 28


Suggested Answer Paper Group I

ii) Extracts or copies of important legal documents, agreements and minutes relevant
to the audit.
iii) A record of the study and evaluation of the internal controls related to the
accounting system.
iv) Copies of audited financial statements of previous years.
v) Analysis of significant ratios and trends.
vi) Copies of management letters issued by the auditor if any.
vii) Record of communication with the retiring auditor, if any before acceptance of
the appointment as the auditor and
viii) Notes regarding significant accounting policies.
b) Statistical Sampling in auditing stands for the technique of forming an opinion about a
group of items on the basis of an examination of a few of the items. On the basis of
audit carried out, an auditor is required to give a report containing his opinion about the
truth and fairness of the accounting statements. Thus, audit sampling involves the
application of audit procedures to less than 100% of the items within an account balance
or class of transactions to enable the auditor to obtain and evaluate audit evidence about
some characteristics of the items selected in order to form or assist in forming a
conclusion concerning the population.
c) A negative external confirmation request asks the respondent to reply only in the event
of disagreement with the information provided in the request. However, when no
response has been received to a negative confirmation request, the auditor remains
aware that there will be no explicit evidence that intended third parties have received
the confirmation requests and verified that the information contained therein is correct.
Accordingly, the use of negative confirmation requests ordinarily provides less reliable
evidence than the use of positive confirmation requests, and the auditor considers
performing other substantive procedures to supplement the use of negative
confirmations.
Negative confirmation requests may be used to reduce audit risk to an acceptable level
when:
i) The assessed level of inherent and control risk is low;
ii) A large number of small balances is involved;
iii) A substantial number of errors is not expected; and
iv) The auditor has no reason to believe that respondents will disregard these requests.
d) The principle of objectivity imposes the obligation on all professional accountants not
to compromise their professional or business judgement because of bias, conflict of
interest or the undue influence of others.
Professional accountants may be exposed to situations that may impair objectivity
which involve the possibility of pressures being exerted on them. These pressures may
impair their objectivity.
It is impracticable to define and prescribe all such situations where these possible
pressures exist. Reasonableness should prevail in establishing standards for identifying
relationships that are likely to, or appear to, impair a professional accountant’s
objectivity.

© The Institute of Chartered Accountants of Nepal 29


Suggested Answer Paper Group I

7. Distinguish between: (2×5=10 marks)


a) Audit and Investigation
b) Test of Controls and Substantive Procedure
Answer:
a) Audit and Investigation
Bases Audit Investigation

Description Auditing is the independent Investigation means an inquiry, or


examination of financial is the act of detail examination of
information of any entity, whether activities so as to achieve certain
profit oriented or not, and objectives.
irrespective of its size or legal form,
when such an examination is
conducted with a view to expressing
an opinion thereon

Owners Audit is conducted on behalf of Investigation may be conducted


owners only and they make the either by owner of the undertaking
appointment. or by an outsider.

Purpose To determine the true and fair view. Varies from business to business

Process Routine process Investigation is not a regular


process

Scope It includes only an examination of It covers an examination of the


the accounts of a business accounts bur also covers an inquiry
into other matter that are connected
with the purpose for which it
is undertaken

Employees Does not examine personally May examine personally

Sequence Usually conducted before Usually conducted after the audit of


investigation of accounts accounts

Person Audit is to be conducted by a Investigation may be take on even


performing chartered accountant or person by a non-chartered accountant
work recognized by regulatory body.

Legal Audit is mandatory under law There is no such legal obligations


Obligations with regard to investigation

b)
Auditor should obtain sufficient and appropriate audit evidences and test them before
framing an opinion about the assertions the financial statements reveal. For this, the
auditor checks evidences through:

© The Institute of Chartered Accountants of Nepal 30


Suggested Answer Paper Group I

• Test of controls and


• Substantive procedure.
Test of controls are procedures designed to evaluate the operating effectiveness of
controls in preventing, or detecting and correcting material misstatements at the
assertion level. It seeks to test that:
• there exists internal control,
• the existing internal control is effective and
• the internal control is working without break or lacunae during the period under
review.
When internal control is found to be to an acceptable level, the accounting entries
generated in such a system is more reliable than in one where the control is weak.
Mere satisfaction about the existence of internal control may not be sufficient for
auditors to express opinion about the assertions the financial data in the form of account
balances and class of transactions.
These transactions and balances need to be tested in detail. This is done by audit
\

procedure called substantive procedure. Substantive procedures are designed to obtain


audit evidence as to the completeness, accuracy and validity of the data produced by
the accounting system.
The substantive procedures involve.
 Test of detail of classes of transactions, account balances and disclosures, and
 Substantive analytical procedure.
The checking of transaction and balances involves vouching of sales, purchases,
payments, receipts and scrutiny of ledgers. The analytical procedure involves critically
examining the accounts in an overall manner and it may entail computation of ratios,
trend analysis so as to dwell in length for examination of unusual or unexplained
deviations.

© The Institute of Chartered Accountants of Nepal 31


Suggested Answer Paper Group I

Paper 3: Corporate and Other Laws


Attempt all questions.
1. Answer the following questions: (5×5=25 marks)
a) Uttam K. C., a director of Diyalo Palace Ltd. was involved in preparing prospectus and
with his some objections he signed on it. The prospectus is approved and registered to
the Office of Company Registrar and accordingly published. When he knew that the
director signing on the prospectus with false contents shall liable personally for loss
caused to the investors, therefore, asked you how he can be freed from such obligation.
Advise him, regarding his approach.
b) As against the interest of Freight Nepal Pvt. Ltd., some of its shareholders were
carrying on a competing business. The majority of the shares were held by the directors,
who passed a special resolution altering the company's articles and introducing a
power for the directors to require any shareholders who competed with the company's
business to transfer their shares at full value to the other directors. The shareholders
carrying on the competing business file a case challenging the validity of the resolution.
Give your opinion on the following issues:
a. Is the special resolution passed valid?
b. Is this amount to oppression to minority shareholders?
c) Kaligandaki Development Bank Ltd. has amended its objective clause of Memorandum
of Association (MoA). As a shareholder, Mr. Gurudatta objected the amendment
thereof. He wanted to make the amendment void. How this is possible? As a corporate
consultant suggest him the legal process in this respect quoting the provisions of the
Companies Act, 2063.
d) Alpine Apple Limited held meeting of its Board of Director (BoD), having 8 members
in the board, on October 25, 2019. It is fourth meetings with the presence of seven
members and signed by five thereof. One of the directors caste his dissenting opinion
in the minute book. Other one has no signature thereon. However, the question of
validity of meeting is raised by the members out of signatory. Justify the validity of the
meeting with reference to the existing provision of the Companies Act, 2063.
e) Mr. Ram Chandra Shah, who was newly appointed CEO of the Gorkha Herbal
Company Ltd. wants to know about the provision of appointment of auditor of the
company. Briefly explain the provision of appointment of auditor as per the Companies
Act, 2063.
Answer:
a) As per Section 24 (1) of the Companies Act, 2063 it shall be the duty and obligation of
the concerned company to be abide by the matters contained in the prospectus
published. Any directors who have signed the prospectus as referred to in Subsection
(1) shall be liable for the matters mentioned in that prospectus.
Similarly, Subsection (3) states that if any published prospectus contains false
statements made maliciously or deliberately and any person sustains any loss or
damage byreason of his/her subscription of securities on the faith of that prospectus,
thedirectors who have signed that prospectus shall be personally liable to pay
compensation for the actual loss or damage so sustained.

© The Institute of Chartered Accountants of Nepal 32


Suggested Answer Paper Group I

A Director who resigns before the decision made bythe company to publish the
prospectus or whom on becoming aware of anyfalse statement in the prospectus,
publishes a notice of that matter to theinformation of the general public prior to the sale
or allotment of securities or who proves that he/she did not know that the prospectus
contained any false statement shall not be liable to bear such compensation.
Hence, in the given case, Uttam by publishing a notice as stated above can be freed
from his obligation.
b)
i.
The basic principle relating to the administration of the affairs of a company is that the
court will not, in general, intervene at the instance of the shareholders in matters of
internal administration where the majority shareholders have been exercised powers in
good faith for the benefit of the company as a whole. Section 74(2) of the Companies
Act, 2063 has laid down that the decision taken in the meeting by the majority of
shareholders, it becomes the decision of the meeting i.e., the decision of the company
and all the shareholders are bound by it. It follows that the court will interfere to protect
the minority only where the majority shareholders of a company propose to be
benefited themselves at the expense of the minority. The resolution passed is valid and
binding.
ii.

Under section 139 and 140 of the Companies Act, 2063 the court can intervene in the
administration of the affairs of a company to protect the minority shareholders from
some improper conducts in the company in the following cases:
• Mala fide: - capriciously, oppressively or corruptly, or in some way mala fide.
• Inadequacy of reasons,
• Irrelevant considerations: - A refusal based on extraneous considerations will be
wrong.
In this case, it is very much for the benefit of the company to get rid of the members
who were in competing business; as such members have the unique opportunity of
exploiting the company's business secrets against its very interest. Hence, the resolution
does not amount to oppression to the minority shareholders.
c) Section 21 of the Companies Act, 2063 has mentioned the process of the amendment
to the memorandum of association and articles of association. The general meeting of
a company may, subject to Section (6), amend the memorandum of association or
articles of association, by adopting a special resolution to that effect.
If a shareholder of a public company who is not satisfied with an amendment made to
the objectives of the company may, on fulfilling the following requirements, file a
petition, setting out the reasons therefore, in the court to have that amendment declared
null and void:
(a) A shareholder or shareholders holding at least five percent shares of the paid-up
capital of the company, except the shareholders who consent to or vote for the
amendment or alteration, has to make a petition,

© The Institute of Chartered Accountants of Nepal 33


Suggested Answer Paper Group I

(b) A petition has to be filed within twenty one days after the adoption of the
resolution to amend the objectives of the company,
(c) Where any one is to file a petition on behalf of one or more than one shareholder
entitled to make petition, the petition has to be filed by a person who is authorized
in writing for that purpose.
On a petition as referred to in sub- section (4), the court may issue an appropriate order,
specifying the following terms and conditions:
(a) Declaring the amendment made to the objectives of the company to be fully or
partly valid or void,
(b) Requiring the company to subscribe for a reasonable value, the shares and other
rights held by the shareholders making a petition under Sub section(4),upon
being disagreed with the making so alteration in the main objectives of the
company,
(c) The shares have to be subscribed under Clause (b) from the moneys as referred
to in Sub-section(2) of Section 61; and in the case of a company which has no
such moneys, issuing an order to decrease the capital of the company as if the
share capital were decreased to the extent of such subscription by adopting a
special resolution by the company; and where such order is issued, the company
shall amend its memorandum of association and articles of association, subject
to the provisions of this Act.
d) The Companies Act, 2063 has provided the meeting procedure of Board of Directors
(BoD). Section 97 of the Act prescribes the procedure about meetings of BoD.
According to Section 97(7) minutes regarding the names of directors present in the
meeting of board of directors, the subjects discussed and the decisions taken thereon
shall be recorded in a separate book, and such minute book shall be signed by at least
fifty one per cent of the total directors present in the meeting.
If any director puts forward any opinion opposed too or differing from the decision in
the course of discussions on any subject in a meeting, he/she may mention the same in
the minute book. Any decision shall not be deemed invalid merely for the reason that
there is no signature of any member.
Regarding the decision of the BoD it is valid. It is because that the Fifty One percent
directors has signed the minute. If a director does not sign the minute, even though he
may or may not have a differing opinion, his opinion will not be sustained. Therefore,
the decision taken by the BoD is valid and sustained for the legal implementation.
e) Section 111 of the Companies Act, 2063 prescribes provision regarding appointment
of the auditor of the company as follows.
1. In the case of a public company an auditor of a company shall be appointed from
amongst the auditors licensed to carry out audit under the prevailing law by the
general meeting.
2. In the case of a private company an auditor of a company shall be appointed from
amongst the auditors licensed to carry out audit under the prevailing law according
to the provisions of Memorandum of Association, Articles of Association or
Consensus Agreement. If there is no such provision, the auditor will be appointed
by the general meeting of the Company.

© The Institute of Chartered Accountants of Nepal 34


Suggested Answer Paper Group I

3. Prior to first general meeting of the company, Board of Directors of the company
will appoint auditor of the company.
4. The auditor so appointed will hold office only until the next annual general meeting.
2. Answer the following questions: (3×5=15 marks)
a) Barleys Bank, London decided to incorporate of its fully owned subsidiary in Nepal.
The bank has appointed you to give advice them as to the legal provisions for the
incorporation of a bank as its fully owned subsidiary and appointed you for the further
process of obtaining a prior approval for the incorporation of a bank fully owned by a
foreign banks and financial institution. Referring the relevant provisions of the Bank
and Financial Institution Act, 2073, give your advice as to the grounds where Nepal
Rastra Bank may refuse to grant prior approval for the incorporation.
b) Civil Bank proposed to buy back of its 20% of shares to consolidate the shares as its
shares were scattered in thousands of shareholder with small denominations.
Advice with the relevant legal provisions of the Bank and Financial Institution Act,
2073 as to the conditions whereby the bank is allowed to buy back its shares.
c) You are appointed as an auditor of Butwal Micro Finance. Its your first auditing in that
Financial Institution (FI). As an auditor, what powers can be exercised over the FI and
list out the matters to be included in audit report relying on the provisions of the Banks
and Financial Institution Act, 2073.
Answer:
a) Section 5 of the Banks and Financial Institutions Act, 2073 has provided for the prior
approval for the incorporation of bank or financial institution with foreign investment
either in joint venture or fully owned subsidiary. Under Sub-section (1) the foreign
bank or financial institution has to make an application for the incorporation of a bank
or financial institution under this Act in joint investment with a corporate body
incorporated in the State of Nepal or with a citizen, or as a subsidiary company
subscribing shares of the foreign bank or financial institution as fixed by Rastra Bank.
Under Section 7 of the Act, the Rastra Bank may deny to grant prior approval for
incorporation of any bank or financial institution and for open branch office of the
foreign bank or financial institution in following circumstances:
 If the name or banking and financial transaction to be carried out by the proposed
bank or financial institution is not found to be appropriate from the point of view of
public interests, religion, ethnicity or traditional belief, etc.,
 If the objective of the proposed bank or financial institution is contrary to the
prevailing laws,
 If incorporation of the proposed bank or financial institution does not seem to be
technically appropriate,
 If study of the feasibility study report, details and documents other infrastructures
submitted by the proposed bank or financial institution does not provide a ground to
believe that it may carry on financial transactions in a healthy and competitive
manner,
 If not all promoters of the proposed bank or financial institution have signed the
Memorandum of Association and Articles of Association, also stating their names,

© The Institute of Chartered Accountants of Nepal 35


Suggested Answer Paper Group I

address and number of shares subscribed by them, in the presence witness and the
name and address of the witnesses have not been mentioned,
 If per person share investment limit and share ownership ratio has not been found to
have been maintained as specified by the Rastra Bank from time to time,
 If it is found to be inconsistent with the policy relating to incorporation of bank or
financial institution and licensing policy issued by the Rastra Bank,
 If any condition as prescribed by the Rastra Bank has not been found to be fulfilled.
b) Buyback of shares means the act of buying or purchase of shares by a bank or financial
institution issued by it. The Banks and Financial Institutions Act, 2073 has prohibited
the Bank or Financial Institution to purchase or buyback of its’ own Shares, Sub-section
(1) of Section 13 of the Act reads, thus, no bank or financial institution shall purchase
its own shares or lend loan against security of its own shares.
However, under Sub-section (2), in the following circumstances, a bank or financial
institution may, with the approval of the Rastra Bank, purchase its shares out of its free
reserves available for being distributed as dividends not exceeding the percentage
prescribed by the Rastra Bank: -
 If the shares issued by the bank or financial institution are fully paid up,
 If the shares issued by the bank or financial institution have already been listed in
the securities market,
 If the buy-back of own shares is authorized by the Articles of Association of the
concerned bank or financial institution,
 If a special resolution has been adopted at the General Meeting of the concerned
bank or financial institution authorizing the buy-back of own share,
 If the ratio of the debt owed by the bank or financial institution is not more than
double of the capital and general reserve fund after such buy-back of shares,
 If the value of shares to be bought back by a bank or financial institution is not more
than 20% of the total paid up capital and general reserve fund of that bank or
financial institution,
 If the buy-back of shares comply with the directives relating to capital fund issued
by the Rastra Bank to the bank or financial institution,
 If it is not against the directives issued by the Rastra Bank from time to time with
regard to buy back of shares.
Therefore, the Civil Bank by complying with the above mentioned conditions upon
receipt of the approval from the Rastra Bank may buy back its shares in any of the
following manners, within 6 months from the date of receipt of such approval or within
12 months of the adoption of a special resolution at the General Meeting, whichever
occurs later:
 Purchasing through the securities market,
 Purchasing from the existing shareholders on a proportional basis.
c) The person who audits the financial statements is called an auditor. He/she is
independent of the audit client. Banks and Financial Institution Act, 2073 has also
provided for the role of the auditor of a bank or financial institution.

© The Institute of Chartered Accountants of Nepal 36


Suggested Answer Paper Group I

The auditor has power to inspect all documents concerning accounts including ledger,
books, account, voucher, and goods at any time and the auditor may seek answers or
explanations from a Director or official of a bank or financial institution on matters
required by him/her in the course of performing his/her duties and carrying out his/her
functions in appropriate manner.
After auditing the accounts of the Bank and financial institution-Butwal Micro Finance,
he/she has to submit a report. Such Report should include the following matters as
required in the sub section (3) of section 66.
(a) Whether or not replies to the queries as per the demand were provided,
(b) Whether or not the balance sheet, off-balance sheet transactions, profit and loss
account, cash flow statement and other financial statements, as well, have been
prepared in such format and in accordance with such procedures as prescribed
by the Rastra Bank, and whether or not they actually matched with the accounts,
records, books and ledgers maintained by the bank and financial institution,
(c) Whether or not the accounts, records, books and ledgers have been
maintained accurately in accordance with prevailing laws,
(d) Whether or not any official of the bank or financial institution has committed
any act contrary to the prevailing laws or committed any irregularity or
caused any loss or damage to the bank or financial institution,
(e) Whether or not credits have been written off as per the Credit Write-off Byelaws
or directives of the Rastra Bank,
(f) Whether or not the transactions of the bank or financial institution have been
carried on in a satisfactory manner as prescribed by the Rastra Bank,
(g) Matters to be informed to the shareholders.
(h) Matters prescribed by the prevailing laws and other matters as prescribed by the
Rastra Bank to be mentioned in audit report by the auditor,
(i) Other suggestions which the auditor deems necessary to be furnished.
3. Answer the following questions: (2×5=10 marks)
a) An insured person is interested to know the adjudication process of the insurance
disputes. State the insurance disputes adjudication process as prescribed by the
Insurance Act, 2049, to make it clear understanding.
b) There is a 'Nepal Securities Board' (Board) in order to regulate and manage the
activities of the securities markets and persons involved in securities business by
regulating the issue, purchase, sale and exchange of securities in order to develop
capital market and protect the interests of investors in securities. This board is headed
by a chairperson appointed by Government of Nepal. He/she is an administrative chief
of the Board. State the functions, duties and powers of the chairperson of the Board
provided by the Securities Act, 2063.
Answer:
a)

© The Institute of Chartered Accountants of Nepal 37


Suggested Answer Paper Group I

Section 8 of the Insurance Act 2049, empowers to the insurance board to decide the case
filed by the insured under the insurance policy against insurer. At the same time it entitles
to the board to mediate the disputes between insurer and insured and all types of insurance
claim are to be heard by insurance board as court of first instance.
As determined by law that the insurance companies are responsible for any action
prejudicing the interest of policyholder or causing harm or loss to the policy holder either
by insurer or by employee of the insurer, agent and surveyor. Rules 31 and 32 of insurance
regulation entitle to the policyholder or the person competent for claim to file an
application in case of not determining liabilities upon claim within stipulated time or in
case of inconvenience of insured on determination.
Adjudication procedure
Upon receiving an application by claimant, adjudication proceeds by issuing a written show
cause notice in form of letter asking to furnish all the details of evidences to defend the
case. At the first time fifteen days time is given. In case of not furnishing documents
/evidences or any inconvenience or any concealment or misrepresentation are found upon
scrutinizing the produced documents board shall further demand all policy dacoit and other
evidences to correspond the case by giving seven days notice.
In case of non cooperation or ignorance by the insurer case will be decided in favour of
insured disclosing reason there under. Board will pass an order under section 17(4) of the
Insurance Act and Rule 33(1Ga) to compensate the insured in the name of insurer
disclosing reasons and giving an opportunity for appeal.
Provision of appeal
Appeal against the decision of the insurance board lies to the concerned HighCourt under
section 37 of the insurance Act. The decision of the High Court shall further be appealed
in the Supreme Court in case of supersede by High Court and will be reviewed in case of
ratification at the supreme court and the decision of Supreme Court will be final.
b)
The functions, duties and powers of the Chairperson of Nepal Securities Board (the Board)
as prescribed by Section 8 of the Securities Act, 2063 are as follows:
(a) To perform such functions as may be necessary for the protection of the interests of
investors in securities for the development of capital market,
(b) To regulate and monitor, or cause to be regulated and monitored, stock exchanges
and transactions of securities business persons in order to make transactions
insecurities Strengthened, effective and reliable,
(c) To act as the chief executive of the Board,
(d) To submit such long-term and short-term plans and policies as be necessary to be
adopted by the Board for the management of stock exchanges and development of
capital market to the Board for its approval,
(e) To call or cause to be called the meeting of the Board and preside over the same,
(f) To implement or cause to be implemented the decisions made by the Board,

© The Institute of Chartered Accountants of Nepal 38


Suggested Answer Paper Group I

(g) To inspect and supervise day-to-day business of the Board and perform the functions
in accordance with the objectives of the Board,
(h) To appoint the advisers and employees required for the Board as prescribed,
(i) To perform or cause to be performed such other functions as may been trusted to
him or her by the Board,
4. Answer the following questions: (2×5=10 marks)
a) Employees of Shankar Motors Ltd. demanded for bonus in advance which was refused by
the manager and asked the board of directors to handle the matter. As no balance sheet
and loss and profit accounts have been prepared, the board became unable to decide the
matter. Give your advice stating the provisions of the Bonus Act, 2030.
b) Mr. Arun has been charged for being absent from entity for more than a consecutive period
of 30 days without getting the leave approved. Management intends to terminate Arun.
State the grounds for termination from work in accordance with the Labour Act, 2074. If
Arun can be terminated as per the given case?
Answer:
a) Section 11 of the Bonus Act, 2030 prescribes the provisions regarding the distribution
of bonus in Advance. Subsection (1) states that if any enterprise fails not submit the
balance-sheet and statement of profit and loss of such enterprise within the time limit
as provided in Sub-section (1) of Section 4, or if bonus could not be distributed within
the time limit of Section 9 by the reasons of inquiry or examination conducted by the
concerned Labour Office on the balance sheet and statement of profit and loss
submitted pursuant to Sub-section (2) of Section 4, the management of such enterprise,
after making tentative computation, shall have to distribute at least five percent of the
net, income as bonus.
Similarly, Subsection (2) provides that after preparation of balance-sheet and statement
of profit and loss or after final assessment is made in this matter. If the amount of bonus
distributed pursuant to the Sub-section (1) is found less than the amount to be
distributed as bonus, the difference amount shall have to be redistributed to the
employees proportionately having considered previously distributed amount.
Finally, Subsection (3) has provide further that if the amount, distributed as bonus is
found excess to the amount assessed for distribution of bonus under Sub-section (2),
the excess amount, whatever may be, shall be deducted for recovery it, while assessing
the net income of the enterprise in the next fiscal year.
b) The allegation is the matter of misconduct on the part of Arun. An employee can be
terminated in the following conditions as per section 131 (4) of the Labour Act, 2074.
Termination upon misconduct (Section 131(4)):
• Causing bodily harm or injury to the Proprietor, Manager or Employee of the entity.
• Accepting or offering bribes. Stealing property of the entity. Embezzlement of
property of the entity.
• Theft of property of anyone in the place of job.
• Causing damage knowingly to the property of the entity.

© The Institute of Chartered Accountants of Nepal 39


Suggested Answer Paper Group I

• Causing damages knowingly to the property of the job provider or his/her


protection or in use thereof.
• Unapproved absence from entity for more than 30 consecutive days.
• Causing damage to secrecy relating to special technology of the entity.
• Doing any activities accompanying with competitive job provider
• Convicted of a criminal offense involving moral turpitude.
• Presenting false documents for the appointment.
• Consuming psychotropic drugs or alcoholic drinks.
• Having been punished twice for other misconducts within 3 years.
• Other similar misconducts as prescribed in the Bylaws.
As discussed above, one of the grounds of termination may be unapproved, absence
from entity for more than 30 consecutive days. Hence, Arun may be terminated from
his work.
Termination on other grounds:
• Voluntary resignation (Section 141)
• The employee can terminate the employment voluntarily by providing the employer
a resignation letter and serving a notice period as prescribed in the employment
agreement.
• Compulsory retirement (Section 147) The employment is terminated by
compulsory retirement when the employee reaches the age of 58.
• Time bound employee (Section 140) The employment is terminated upon expiry of
the time period prescribed in the employment agreement.
• Work based employee (Section 140) The employment terminates upon completion
of the work specified in the employment agreement.
• Termination due to poor performance (Section 142) The employment may be
terminated if performance of the employee is found to be unsatisfactory or below
standard in 3 consecutive performance appraisals.
• Termination due to bad health (Section 143) The employment may be terminated
on the recommendation of a medical practitioner, if the employee is physically or
mentally disabled or injured, rendering him/her unable to work, or requiring
him/her a long period of medical treatment, therefore affecting the work of the
entity.
• Termination due to sexual harassment: Section 132. As per sec 132 of the act, the
service may be terminated on the basis of seriousness of offence.
5. Answer the following questions: (2×5=10 marks)
a) State the circumstances where a councilor may cease from his post and how such vacant
post is fulfilled pursuant to the Nepal Chartered Accountants Act, 2053.
b) An Executive Director of the ICAN plays an important role pertaining to the operation of
the administrative business of the Institute. Mention the related provision of the Nepal

© The Institute of Chartered Accountants of Nepal 40


Suggested Answer Paper Group I

Chartered Accountant Act, 2053 about Executive Director and also explain the functions,
duties and authorities of the Executive Director.
Answer:
a)
Pursuant to Section 8 (1) of the Act, a councilor shall cease to hold his/her office in any
of the following circumstances:
(a) If the councilor ceases to be a member of the Institute.
(b) If resignation tendered by the councilor from his/her office is accepted by the
Council.
(c) If the councilor absents himself or herself from three consecutive meetings of the
Council without giving a notice with reason to the Council.
(d) If the term of office of the councilor expires.
(e) If it is proved that the councilor has not abided by the conduct referred to in Section
34.
(f) If he dies.
Where the remaining term of office of any councilor elected whose office has become
vacant due to his/her death or resignation or being disqualified to be a member of the
Institute pursuant to the provisions of this Act, is less than One year, the Council shall
designate as the council and where such term is more than One year, the vacancy shall
be filled by way of election.
b)
Section 38 of Nepal Chartered Accountants Act, 2053 prescribes the provision
regarding the Executive Director of the ICAN. According to the Section:
(1) The Council shall, for the operation of the administrative business of the Institute,
appoint any person having experience in accounting profession as the Executive
Director.
(2) The term of office of the Executive Director shall be four years, and the Council
may, if it so desires, reappoint him.
(3) The Council, in the absence of the Executive Director, may designate any officer
level employee of the Institute to act as the Executive Director. 39.
As per section 39 of Nepal Chartered Accountants Act, 1997:
(1) The functions, duties and authorities of the Executive Director shall be as
follows:-
(a) To act as chief executive of the Institute being accountable to the Council,
(b) To carry out day-to-day administrative business of the Institute,
(c) To submit annual budget of the Institute to the Council,
(d) To maintain and cause to be maintained the books of accounts of the
Institute,
(e) To keep in custody of the Members' Register and Register of the Members
holding Certificate of Practice and keep it up-to date.

© The Institute of Chartered Accountants of Nepal 41


Suggested Answer Paper Group I

(2) The Executive Director shall discharge his duties under the control and
supervision of the Council.
(3) The functions, duties and powers of the Executive Director, other than those set
forth in this Act, shall be as prescribed.
6. Answer the following questions: (5×4=20 marks)

a) 'A' finds a lost blank stamped Promissory Note signed by B. He fills it up for NRs.
1,00,000 and delivered it to C against the payment of furniture who has received it in
good faith. When the Note was presented B refused to pay on it as it was the lost
instrument and he had already informed to the police. Can C recover NRs. 1,00,000
from B? Decide.
b) State the legal provisions regarding the account and auditing of the Social Welfare
Council.
c) Discuss about the provision of Quorum of the Companies Act, 2063.
d) Define the term‘specific performance of contract’.State it quoting the concern section
of the National Civil Code, 2074.
e) State in brief regarding audit of Insurance Companies as prescribed by the Insurance
Act, 2049.
Answer:
a. A person who receives negotiable instrument for consideration, before its maturity and
in good faith is known as the holder in due course. Certain defenses which can set up
as against a holder cannot set up as against the holder in due course. The holder in due
course can get good title in the instrument though he has received it from the transferor
whose tile in the instrument is defective.
In this case, C is the holder in due course as he had received the Promissory Note which
was blank stamped instrument and filled up by A, the finder of lost goods. Therefore,
B cannot refuse to pay the amount NRs. 1,00,000 stated therein to C as he is the holder
in due course.
b. According to Section 18 of the Social Welfare Act, 2049, the account of the Council
shall be maintained in accordance with the procedures accepted by Government of
Nepal. Pursuant to Subsection (2), the Audit of the Council shall be carried by the
Auditor General.
As per Subsection (4),the Government of Nepal, if it so wishes, may inspect or cause
to inspect the accounts document along with cash and kind of the council at any time.
Further, Subsection (5) empowers the Council to inspect or cause to inspect the
accounts document along with cash and kind of the social organization and institutions
affiliated with the Council at any time if it so wishes. Account and auditing of the social
organizations, affiliated with the council shall be as prescribed.
c. Quorum means minimum numbers of the shareholders to be present in the general
meeting of the company. Meeting cannot be held unless and until the shareholders are
not present in the specified minimum number for conducting general meeting.
Section 73 of the Companies Act, 2063 is related with the quorum. It provides as
follows:

© The Institute of Chartered Accountants of Nepal 42


Suggested Answer Paper Group I

(1) A quorum for the general meeting of a private company shall be as specified in
the articles of association of such company.
(2) Unless the articles of association of a public company provides for a larger
number for the quorum, no proceedings of the meeting of the public company
shall be conducted unless at least three shareholders of the total shareholders,
representing more than fifty per cent of the total number of allotted shares of that
company, are present either in person or by proxy.
(3) Where a meeting cannot be held because of quorum as referred to in Sub-section
(2), and the meeting is called next time by giving a notice of at least seven days,
nothing shall prevent the holding of such a meeting if at least three shareholders,
representing twenty five percent of the total number of allotted shares of the
company, are present either in person or by proxy.
(4) Notwithstanding anything contained elsewhere in this Section , in the case of a
company incorporated under the proviso to Sub-section (2), of Section 3 or a
company incorporated under Sub section(1) of Section 173, the presence of three
shareholders as mentioned in Sub-section(2) or (3) shall not be mandatory.
d. The aggrieved party is entitled to sue for specified performance of the contract. It
means an order issued by the court upon the party breaching the contract directing
him to perform the obligation as agreed upon previously. When only the
compensation is not adequate remedy for the aggrieved party can demand such
performance.
Section 540 of the National Civil Code, 2074 has provided about the specific
performance as follows:
(1) If monetary compensation is not reasonable and adequate for the actual loss
or damage suffered by the aggrieved party because of the breach of contract,
the aggrieved party may claim for the specific performance of the contract
instead of compensation.
(2) Notwithstanding anything contained in sub-section (1), specific performance
may not be claimed in any of the following circumstances:
(a) If monetary compensation is an adequate remedy for the breach of
contract,
(b) If the court cannot supervise the performance of the act under the
contract,
(c) If the contract is for providing services of personal ability, skill or
knowledge,
(d) If the contract cannot specifically be performed,
(e) If the party in breach of the contract claims for its specific performance.
e. Insurance Act, 2049 specifically prescribes the provision regarding the audit of the
insurance companies. According to Section 25 of the Insurance Act, 2049, the audit
of an insurance companies will be done as follows:
(1) The Insurer shall have the accounts and records of its Insurance Business
audited by an auditor recognized in accordance with the prevailing law and
submit a report thereof to the Board no later than ten months from the date
expiration of each fiscal year.

© The Institute of Chartered Accountants of Nepal 43


Suggested Answer Paper Group I

(2) Comments (inconsistencies and irregularity) made while performing the audit
and the clarification of the management relating to it also shall be mentioned in
the report pursuant to Sub-section (1).
7. As per the objective of the Industrial Enterprises Act, 2073 it has prescribed the various
classification of the industries. State the classification of those industries specifying
their nature thereof. (10 marks)
Answer:
The Industrial Enterprise Act, 2073 has classified industries on the basis of fixed assets
and on the basis of the nature. This provision has been prescribed in Section 15 for the
fulfillment of the objective of the Act. According to it, the classification of the
industries are as follows:
i. Classification on the basis of the fixed assets:
a. Micro entrepreneur/Industries: following industries will be considered as
micro entrepreneur/industries:
 With fixed capital up to five lakh except house and land,
 Industries run by the proprietor or entrepreneur himself,
 Not exceeding nine employees including proprietor,
 Having less than fifty lakh transaction annually,
 Machines, equipment or applying electrical energy having capacity less
than 20 kilowatts. However, Industries mentioned in Schedule I required to
obtain permission will not be considered as Micro Industries.
b. Cottage Industries: Following industries will be considered as Cottage
industries:
 Traditional skill and technology based industries,
 Labour intensive and specific skill or local raw materials as well as or
local technology, art and culture related industries,
 Industries which have been using machines equipments or up to 10
kilowatts,
 Industries prescribed in Schedule II.
c. Small Industries: It refers to the industries other than micro and small industries
with fixed capital up to ten crores.
d. Medium Industries: It refers to the industries with fixed capital more than ten
crores not exceeding 25 crores.
e. Large Industries: It refers to the industries having fixed capital more than
twenty five crores.
ii. Classification of above industries, on the basis of the nature as follows:
a. Energy based Industry: Industries mentioned in Schedule III
© The Institute of Chartered Accountants of Nepal 44
Suggested Answer Paper Group I

b. Manufacturing Industry: Industries producing goods using raw materials or


semi or processed raw materials (Not prescribed schedule, but provided more
facilities)
c. Agro Forest based Industry: Industries based on agriculture or forest products:
Industries prescribed in Schedule IV
d. Mineral Industry: Industries other than Metal or Metallic mineral production
through excavation or processing minerals thereof.
e. Construction Industry: Industries prescribed in Schedule V operated with
Physical infrastructure production based
f. Tourism Industry: Industries related to tourism services as prescribed in
Schedule VI.
g. Industries based on Information, Transmission and Communication
Technology: Industries prescribed in Schedule VII
h. Service Oriented Industry: Service production or service provider based
industries as prescribed in Schedule VIII

© The Institute of Chartered Accountants of Nepal 45


Suggested Answer Paper Group I

Examiner’s Commentary on Students' Performance in December


2019 Examinations
This commentary has been written to accompany the published questions and answers and is
written based on the observations of evaluators. The aim is to provide constructive guidance
for future candidates, giving insight into what the evaluating team is looking for, and flagging
difficulties encountered by candidates who attempted these questions.
Subject: Advanced Accounting
Question No. 1
Many students were confused in preparing separate financial statements for first half and
second half periods. Student did not understand the question properly. Only few student had
allocated the profit into pre- and post-retirement portions.
Question No. 2
(a) Computation of estimated profit was not solved properly. Students were not aware about
the calculation of Memo Contract Account. Most of the students had not answered the site
plant's valuation correctly. Also working notes were lacking.
(b) The answer to this question was satisfactorily.
Question No. 3
(a) Some students were confused in computing purchase consideration. Question asked for
preparation of ledger accounts, however, most of the students had prepared journals.
(b) Almost all students did not allocate the items properly. Only statement was to be prepared.
Ratio calculation was not appropriate in most of the cases.
Question No. 4
(a) Students were confused in proper treatment of reserve and unable to pass journal entries.
Very few students were aware about the Insurance business accounting. Understanding of
revenue recognition of insurance business was lacking.
(b) Some students were confused in acid test ratio.

Question No. 5
(a) Almost all students were confused in deriving value of Raw Material and finished goods.
No clarity on Inventory valuation and adjustment of prior period item, revenue recognition.
Students lacked conceptual clarity about accounting standards. Answer should be based on
standards.
(b) Students did not have proper understanding of restatement.
(c) No conceptual and practical knowledge.
Question No. 6
Language was poor. No conceptual clarity. Students had no knowledge of reportable segment
and substance over form. Rest of the question had been properly answered by most of the
students. The theory notes should be to the point.

Subject: Audit and Assurance


Question No. 1

© The Institute of Chartered Accountants of Nepal 46


Suggested Answer Paper Group I

Students were unable to answer as per the requirement of question and students lacked the
conceptual knowledge regarding standards. Overall performance was satisfactory.
Question No. 2
There was lack of conceptual knowledge. However, in case of answer 2(b) students were
unable to understand the requirement of question. The provision/standards related with using
the work of an expert was not up to the mark.
Question No. 3
Most of the students did not answer three parts of the question. They had not answered as per
the requirement of the question. Most of the students were unable to write the proper
consideration. They were unable to test for reliance on written representations under NSA 580.
The concepts of materiality, elements of the assurance engagement were not understood by
majority of the candidates.
Question No. 4
Although most of the students attempted all three parts, they lacked the proper understanding
of the requirement of laws. Consideration to reporting to audit committee was not explained.
Assessment of significance of breach was not explained.
Question No. 5
Most of the students answered both part, though majority of them did not write the contents of
Section 64 of BAFIA. Few students were confused between 51% or more than 51% holding.
Question No. 6
Some tagged it to negative opinion. Few were confused with contents of permanent audit file.
Question No. 7
Concept of test check and internal check was not clear in majorities of cases.

Subject: Corporate & Other Laws


Question No. 1
Student's performance was satisfactory. Most of them had understood the facts and the law.
Question No. 2
Students had prepared well in banking laws, their answer was satisfactory.
Question No. 3
Students had not gone through the insurance law and they are not clear on the provisions of the
Securities Act. Not properly aware regarding the dispute settlement in insurance. Adjudication
process/procedure had not been answer clearly and final jurisdiction of Court not mentioned in
3(a) and the functions, duties and powers correctly written by few students.
Question No. 4
Students were not clear about the conditions provided under the Labour Act. Most answers
written mode of distribution of bonus in 4(a), whereas, half of answer of 4(a) mention almost
answer only termination upon misconduct has not written termination on other grounds.
Question No. 5
Students had applied their common sense. They were not aware of the provision of the Act.

© The Institute of Chartered Accountants of Nepal 47


Suggested Answer Paper Group I

Question No. 6
Students had not given focus to the provisions of these laws believing they are not as important
as other laws.
Question No. 7
They were aware of the provisions and had mentioned them. The performance was satisfactory.

© The Institute of Chartered Accountants of Nepal 48


Table of Contents
Group II

Paper 4: Financial Management.............................................................................................................. 2


Paper 5: Cost and Management Accounting ......................................................................................... 20
Paper 6: Business Communication and Marketing ............................................................................... 38
Paper 7: Income Tax and VAT ............................................................................................................. 48
Examiner’s Commentary on Students' Performance in December 2019 Examinations .............. 66
Suggested Answer Paper Group II

Paper 4: Financial Management


Attempt all questions.

Working notes should form part of the answer. Make assumptions wherever necessary.

1. A manufacturing company is considering the installation of a machine to process the waste


produced by one of its existing manufacturing process to be converted into a marketable
product. At present, the waste is removed by a contractor for disposal on payment by the
company of Rs. 5 million per annum for the next 4 years. The contract can be terminated
upon installation of the aforesaid machine on payment of a compensation of Rs. 3 million
before the processing operation starts. This compensation is not allowed as deduction for
tax purpose.
The machine required for carrying out the processing will cost Rs. 20 million to be financed
by a loan repayable in 4 equal annual installments of principal plus accrued interest
commencing from the end of year 1. The interest rate is 16% per annum. At the end of the
4th year, the machine can be sold for Rs. 2 million. The machine will fall in its existing
block of machines on its tax books.
Sales and direct costs of the product emerging from waste processing for 4 years are
estimated as under:
Amount in Rs. million
Description Year 1 Year 2 Year 3 Year 4
Sales 32.2 32.2 41.8 41.8
Material consumption 3 4 8.5 8.5
Wages 7.5 7.5 8.5 10
Other expenses 4 4.5 5.4 7
Factory overheads 5.5 6 11 14.5
Depreciation (as per income tax provisions) 5 3.8 2.8 2.1
Initial stock of materials required before commencement of the processing operations is
Rs. 2 million at the start of year 1. The stock levels of materials to be maintained at the end
of year 1, 2 and 3 will be Rs. 5.5 million and the stocks at the end of year 4 will be nil. The
storage of materials will utilize space which would otherwise have been rented out for Rs.
1 million per annum. Labour costs include wages of 40 workers, whose transfer to this
process will reduce idle time payments of Rs. 1.5 million in year 1 and Rs. 1 million in year
2. Factory overheads include apportionment of general factory overheads except to the
extent of insurance charges of Rs. 3 million per annum payable on this venture. The
company’s tax rate is 30% and its cost of capital is considered to be 15%.
Required: (20 marks)
Advise the management on the desirability of installing the machine for processing the
waste.
Answer:
Calculation of net present value of the venture Amount in Rs. million

© The Institute of Chartered Accountants of Nepal 2


Suggested Answer Paper Group II

Description Year 0 Year 1 Year 2 Year 3 Year 4 Total


Annual cash flow after tax (WN1) 10.60 9.40 11.20 9.38
Less: Post tax cost of rent forgone (0.7) (0.7) (0.7) (0.7)
Add: Release of working capital 5.5
Add: Proceeds from sale of machine (WN3) 2.0
Add: Post tax cost of removal of waste 3.50 3.50 3.50 3.50
Total cash inflows 13.40 12.20 14.00 19.68 59.28
PVF @ 15% 1 0.870 0.756 0.658 0.572
Present value of cash inflows (A) 0 11.66 9.22 9.21 11.26 41.35

Compensation on termination (3)


Add: Payment of machine loan installment (5.00) (5.00) (5.00) (5.00)
Add: Initial working capital (2)
Add: Subsequent working capital 3.5
Total cash outflows (5) (8.5) (5) (5) (5) (28.5)
Present value of cash outflows (B) (5) (7.40) (3.78) (3.29) (2.86) (22.33)

Net present value (A-B) 19.03


Advice: Since the proposed machine produces an overall net present value of Rs. 19.03 million
which adds to the maximization of shareholders wealth, it is highly recommended to install the
machine.
Working notes:
1. Calculation of annual cash flow after tax (Rs. Million)
Description Year 1 Year 2 Year 3 Year 4
Sales 32.2 32.2 41.8 41.8
Less: Materials consumption 3 4 8.5 8.5
Less: Wages (net of idle time savings) 6 6.5 8.5 10
Less: Other expenses 4 4.5 5.4 7
Less: Factory overheads (except apportioned overheads) 3 3 3 3
Less: Depreciation (1) 5 3.8 2.8 2.1
Less: Interests (WN2) 3.2 2.4 1.6 0.8
Profit before tax 8.0 8.0 12.0 10.40
Less: Income tax @ 30% 2.4 2.4 3.6 3.12
Profit after tax (2) 5.6 5.6 8.4 7.28
Cash flow after tax (2+1) 10.6 9.4 11.2 9.38
2. Calculation of interest expense (Rs. Million)
Year Opening Principal Interest expense Principal Repayment
1 20 3.2 5
2 15 2.4 5
3 10 1.6 5
4 5 0.8 5

3. Since the proposed machine falls in existing block of machines in tax books, the entire sale
proceeds of machine is added in cash flows of fourth year.
2.

© The Institute of Chartered Accountants of Nepal 3


Suggested Answer Paper Group II

a. The Rocket Ltd. has target capital structure of 60% equity and 40% debt. The schedule of
financing cost for the Rocket is shown in the table below:

Amount of New Debt (Rs.) After tax Amount of New Equity Cost of Equity
Cost of Debt (Rs.)
0-1,000,000 5% 0-1,000,000 9%
1,000,001-2,000,000 6% 1,000,001-3,000,000 10%
2,000,001-3,000,000 7% 3,000,001- 5,000,000 11%
3,000,001- 4,000,000 8% 5,000,001- 8,000,000 12%
4,000,001- 5,000,000 9%

Required: (5+5=10 marks)


i. Calculate the break point for The Rocket Ltd.
ii. Calculate Weighted Average Cost of Capital (WACC) for alternate level of financing.
b.The following information relates to Mangal Limited:
Earnings of the company (Rs.) 1,000,000
Dividend payout ratio 60%
No. of share outstanding 200,000
Return on investment 15%
Equity capitalization rate 12%
Required: (2.5+2.5=5 marks)
i. What would be the market value per share of the company as per Walter’s model?
ii. What is the optimum dividend payout ratio according to Walter’s model and the market
value of the company’s share at that payout ratio?
Answer:
Solution 2 (a):
Working for Break Point
Break Point Working Size of Investment/Capital (Rs.)

Debt > 1,000,000 1,000,000 ÷ 0.40 > 2,500,000


Debt > 2,000,000 2,000,000 ÷ 0.40 > 5,000,000

Debt > 3,000,000 3,000,000 ÷ 0.40 > 7,500,000


Debt > 4,000,000 4,000,000 ÷ 0.40 > 10,000,000

Debt > 5,000,000 5,000,000 ÷ 0.40 >12,500,000

Equity > 1,000,000 1,000,000 ÷ 0.60 > 1,666,667


Equity > 3,000,000 3,000,000 ÷ 0.60 > 5,000,000

Equity > 5,000,000 5,000,000 ÷ 0.60 > 8,333,333


Equity > 8,000,000 5,000,000 ÷ 0.60 > 13,333,333

Rocket will have a break point each time a component cost of capital changes, for a total of six
break points;

© The Institute of Chartered Accountants of Nepal 4


Suggested Answer Paper Group II

Break Point Size of Reason for Change in Cost of Capital


Investment/Capital (Rs.)

1 > 1,666,667 Equity

2 > 2,500,000 Debt

3 > 5,000,000 Debt as well as Equity


4 > 7,500,000 Debt

5 > 8,333,333 Equity

6 > 10,000,000 Debt

7 >12,500,000 Debt

8 >13,333,333 Equity

WACC for Alternative level of Financing


Investment/Capital (Rs.) Equity Cost of Debt 40% Cost of WACC
60% Equity Debt
Up to 1,666,667 60% 9% 40% 5% 7.40%
1,666,668 - 2,500,000 60% 10% 40% 5% 8.00%
2,500,001 - 5,000,000 60% 10% 40% 6% 8.40%
5,000,001 - 7,500,000 60% 11% 40% 7% 9.40%
7,500,001 - 8,333,333 60% 11% 40% 8% 9.80%
8,333,334 - 10,000,000 60% 12% 40% 8% 10.40%
10,000,001 - 12,500,000 60% 12% 40% 9% 10.80%
Solution 2 (b) :
i. Walter’s Model is given by
𝑟𝑟
𝐷𝐷+(𝐸𝐸−𝐷𝐷)𝑘𝑘𝑘𝑘
P=
𝐾𝐾𝐾𝐾

Where, P = Market Price Per Share


E = Earnings Per Share = Rs. 1,000,000/200,000 = Rs. 5
D = Dividend Per Share = Rs. 5×0.60 = Rs. 3
r = Return on Investment = 15%
Ke = Cost of Equity = 12%
Therefore,
0.15
3+(5−3)0.12
P= = Rs. 45.83
0.12
ii. According to Walter’s Model, when the return on investment is more than the cost of equity
capital, the price per share increases as the dividend pay-out ratio decreases. Hence the

© The Institute of Chartered Accountants of Nepal 5


Suggested Answer Paper Group II

optimum dividend pay-out ratio in this case is Nil. So, at a pay-out ratio of zero, the market
value of the company’s share will be:
0.15
0+(5−0)0.12
P= = Rs. 52.08
0.12
3.
a. The following figures are made available to you:

Rs.
Net profit for the year 1,800,000
Less: Interest on secured debentures at 15% p.a. 112,500
(Debentures were issued 3 months after the commencement
of the year)
Earnings before tax 1,687,500
Less: Income tax at 35% and Dividend Tax 843,750
Profit after tax 843,750
Number of equity shares (Rs. 10 each) 100,000
Market quotation of equity share Rs. 109.70
The company has accumulated revenue reserves of Rs. 1.2 million. It is examining a project
calling for an investment obligation of Rs. 1 million. This investment is expected to earn
the same rate of return as funds already employed.
You are informed that a debt equity ratio (Debt divided by debt plus equity) higher than
60% will cause the price earnings ratio to come down by 25% and the interest rate on
additional borrowings will cost company 300 basic points more than on the current
borrowings on secured debentures.
Required: (8 marks)
Advise the company on the probable price of the equity share, if
i. the additional investment were to be raised by way of loans; or

ii. the additional investment were to be raised by way of equity. Make valid assumption
wherever necessary.

b. Star Ltd. is a manufacturer of various electronic gadgets. The annual turnover for the
year 2075/76 was Rs. 73 million. The company has a wide network of sales outlets all
over the country. All sales are for credit and are spread evenly throughout the year. All
invoicing of credit sales is carried out at the head office in Kathmandu. Sales
documentation is sent by post, daily from each location to the head office. Delays in
preparing and dispatching invoices have come to the notice of management.

An analysis of the delay in invoicing, being the interval between the date of sale and the
date of dispatch of the invoice, indicated the following pattern:
No. of days of delay in invoicing 3 4 5 6
% of weeks’ sales 20 10 40 30

© The Institute of Chartered Accountants of Nepal 6


Suggested Answer Paper Group II

A further analysis indicated that the debtors take on an average 36 days of credit before
paying. This period is measured from the day of dispatch of the invoice rather than the
date of sale.
It is proposed to hire an agency for undertaking the invoicing work at various locations.
The agency has assured that the maximum delay would be reduced to three days under the
following pattern:
No. of days of delay in invoicing 0 1 3
% of week’s sales 40 40 20
The agency has also offered, additionally, to monitor the collections which will reduce the
credit period to 30 days. Star Ltd. expects to save Rs. 4,000 per month in postage costs. All
working funds are borrowed from a local bank at simple interest rate of 20 % p.a.
The agency has quoted a fee of Rs. 200,000 p.a. for the invoicing work and Rs. 250,000
p.a. for monitoring collections and is willing to offer a discount of Rs. 50,000 provided
both the works are given. (7 marks)
Required:
Advise Star Ltd. about the acceptance of agency’s proposal.
Answer:
a.

Working Note:
Present earnings per share:
Rs.
Profit before taxes 1,687,500
Less: Taxes at 35% (590,625)
Profit after tax 1,096,875
No. of equity shares 100,000

E.P.S. = 1,096,875/100,000
= Rs. 10.97
Market Price = Rs. 109.70

Hence, P/E ratio = 109.70/10.97 = 10 times


(i) Probable Price/share, if the additional investment were to be raised by way of loans
Present capital employed:
Rs.
Equity 1,000,000
Debenture (Long term) 1,000,000 [(112,500×12/9)÷0.15]
Revenue reserves 1,200,000 Rs. 3,200,000

© The Institute of Chartered Accountants of Nepal 7


Suggested Answer Paper Group II

Pre-interest and pre-tax profits given Rs. 1,800,000


Rate of return = 1,800,000/3,200,000×100 = 56.25%
Debt equity ratio, if Rs. 1 million (additional investment) were to be borrowed (Debt Rs.
1.0+1.00 2
20 lacs and equity Rs.× 100 = ×100 = 47.62%
3.2+1.00 4.2
Since, the debt equity ratio will not exceed 60% P/E will remain same.
If Rs. 1 million is to be borrowed, the earning will be as under:
Rs. Rs.
Return of 56.25% on Rs. 4.2 million 23,62,500
Less: Interest at 15% on existing Rs. 1 million debentures 150,000
Interest on fresh borrowed amount of Rs. 1 million at 18% 180,000 330,000
Profit after interest before tax 2,032,500
Less: Tax at 35% 711,375
Profit after tax 1,321,125
No. of equity shares 100,000
Rs. 13,21,125
E.P.S. = 1,00,000 = Rs. 13.21

Probable price of equity share = Rs. 13.21 x 10


= Rs. 132.10
(ii) Probable Price/share, if additional investment were to be raised by way of equity
If Rs. 1 million were to be raised by way of equity shares to be raised at market rates.
The existing market price of Rs. 109.70 may come down a little and may possibly settle
at Rs. 100. Hence, new equity shares to be raised will be:
Rs. 1,000,000 /Rs. 100 = 10,000 shares
If Rs. 1 million is to be raised by way of equity shares, the earning will be as under:-
Rs.
Profit before interest and tax 2,362,500
Less: Interest on debentures 150,000
Profit after interest before tax 2,212,500
Less: Tax @ 35% 774,375
Profit after tax 1,438,125
No. of equity shares 110,000

Rs. 14,38,125
E.P.S.= = Rs. 13.07
1,10,000
Probable price of equity share = Rs. 13.07 x 10
= Rs. 130.70
b.

© The Institute of Chartered Accountants of Nepal 8


Suggested Answer Paper Group II

Average Annual Sales (Rs in Million) 73


Number of Days in a Year 365
Average Daily Sales (Rs.) (73,000,000÷365) 200,000

Invoicing Function:
Average Days delay in Invoicing (Days) 4.8
(3*0.20+4*0.10+5*0.40+6*0.30)
Average Delay after Agency's Service (Days) 1
(0*0.40+1*0.40+3*0.20)
Annual Saving in Invoicing Days 3.8
Release of Fund (Rs.) [3.8 Days * Rs 200,000] 760,000
Annual Saving in Opportunity Cost [Rs 760,000 *20%] 152,000
Additional Saving on Collection Cost (Postage) (12*Rs 4,000) 48,000
Total Gross Saving under Invoicing Function (Rs) 200,000

Collection Function:
Annual Saving in Collection Days (36 Days - 30 Days) 6
Release of Fund (Rs.) [ 6 Days * Rs 200,000] 1,200,000
Annual Saving in Opportunity Cost [ Rs 1,200,000*20%] 240,000

Analysis of Offer of Agency


Work Saving (Rs) Cost of Agency (Rs) Decision
Invoicing 200,000 200,000 Indifference
Collection 240,000 250,000 Reject
440,000 450,000
Discount (50,000)
Total Work 440,000 400,000 Accept

4.
a) Naxal Devi Ltd. is commencing a new project for manufacture of a plastic component.
The following cost information has been ascertained for annual production of 12,000
units which is the full capacity:
Particulars Cost per unit (Rs.)
Materials 40
Direct labour and variable expenses 20
Fixed manufacturing expenses 6
Depreciation 10
Fixed administration expenses 4
Total 80
The selling price per unit is expected to be Rs. 96 and the selling expenses are Rs. 5 per
unit, 80% of which is variable.
In the first two years of operations, production and sales are expected to be as follows:
Year Production (units) Sales (units)

© The Institute of Chartered Accountants of Nepal 9


Suggested Answer Paper Group II

1 6,000 5,000
2 9,000 8,500
To assess the working capital requirements, the following additional information is
available:
Stock of materials : 2.25 months’ average consumption
Work in process : Nil
Debtors : 1 month's average cost of sales
Cash Balance : Rs. 10,000
Creditors for supply of materials: 1 month's average purchase during the year
Creditors for expenses : 1 month's average of all expenses during the year
Valuation of finished goods : At average cost
Required: (4+4=8 marks)
i) Prepare projected statement of profit/loss (ignoring taxation) for the two years.
ii) Prepare projected statement of working capital requirement for the two years on cash
cost basis.
b) An investor is considering to buy a share of the face value of Rs. 100 which has current
market price of Rs. 480. Annual expected dividend from the share is 30%. During the 5th
year, the investor is expecting a bonus in the ratio of 1:5. Dividend rate is expected to be
maintained on the expanded capital base. The investor intends to retain the share till the
end of 8th year. At the time, the value of share is expected to be Rs. 1,000. Incidental
expenses at the time of purchase and sale are estimated at 5% on the market price. There
is no tax on dividend income and capital gain. The investor expects a minimum return of
15% per annum.
Required: (7 marks)
Should the investor buy the share? What is the maximum price he can pay for the share?

Answer:
a.
Projected Statement of Profit/ (Loss)
Particulars Year I Year II

Units Per Unit Total Units Per Unit Total (Rs)


(Rs) (Rs) (Rs)

Normal Production 12,000 12,000

Actual Production 6,000 9,000


Sales 5,000 8,500

A. Sales Revenue 96 480,000 96 816,000


B. Less : Cost of Sales

a) Direct Material Cost 40 240,000 40 360,000

© The Institute of Chartered Accountants of Nepal 10


Suggested Answer Paper Group II

b) Direct Labour & 20 120,000 20 180,000


Variable Expenses

c) Fixed Manufacturing 12 72,000 8 72,000


Expenses

d) Depreciation 20 120,000 13.33 120,000

e) Total Cost of Goods 6000 92 552,000 9000 81.33 732,000


produced

f) Add: Opening Stock of 0 0 1,000 92.00 92,000


Finished Goods
g) Total cost of goods 6000 92 552,000 10,000 82.4 824,000
available

h) Less: Closing stock of 1,000 92 (92,000) 1,500 82.4 (123,600)


finished goods

i) Total cost of Goods sold 5,000 92 460,000 8,500 82.4 700,400


j) Add: Fixed 9.6 48,000 5.65 48,000
Administration expenses

k) Add: Variable selling 4 20,000 4 34,000


Expenses

l) Add: Fixed Selling 2.4 12,000 1.41 12,000


Expenses
Total Cost of Sales 540,000 794,400
(i+j+k+l)

C. Profit/(Loss) (A-B) (60,000) 21,600

Statement showing the requirement of working capital (On cash cost basis)
Particulars Year I Year II

Computation Rs. Computation Rs.

A. Current Assets

Stock of Raw Material 240,000Х2.25/12 45,000 360,000Х2.25/12 67,500

Stock of Finished Goods 72,000 102,600

Debtors 440,000Х1/12 36,667 675,400Х1/12 56,283

Cash in Hand 10,000 10,000

Total Current Assets 163,667 236,383


B. Current Liabilities

Creditors for Raw Materials 285,000Х1/12 23,750 382,500Х1/12 31,875

© The Institute of Chartered Accountants of Nepal 11


Suggested Answer Paper Group II

Creditors for wages and 120,000Х1/12 10,000 180,000Х1/12 15,000


variable expenses

Creditors for Manufacturing 72,000Х1/12 6,000 72,000Х1/12 6,000


Expenses

Creditors for Administrative 48,000Х1/12 4,000 48,000Х1/12 4,000


Expenses
Creditors for Selling Expenses 32,000Х1/12 2,667 46,000Х1/12 3,833

Total Current Liabilities 46,417 60,708

C. Net Working Capital (A-B) 117,250 175,675

Working Notes:
I) Calculation of cash cost of closing stock and cash cost of sales
Particulars Year I Year II

Units Per Unit Total (Rs) Units Per Unit Total (Rs)
(Rs) (Rs)

Normal Production (In units) 12,000 12,000

Actual Production (In units) 6,000 9,000

Sales (In Units) 5,000 8,500

Cost of Sales
a) Direct Material Cost 40 240,000 40 360,000

b) Direct Labour & Variable 20 120,000 20 180,000


Expenses
c) Fixed Manufacturing 12 72,000 8 72,000
Expenses (Excluding
Depreciation)

d) Total Cost of Goods 6,000 72 432,000 68 612,000


produced

e) Add: Opening Stock of 0 0 1,000 72 72,000


Finished Goods

f) Total cost of goods 6,000 72 432,000 10,000 68.4 684,000


available

g) Less: Closing stock of 1,000 72 (72,000) 1,500 68.4 (102,600)


finished goods

h) Total cost of Goods sold 5,000 72 360,000 8,500 68.4 581,400

i) Add: Fixed Administration 9.6 48,000 5.65 48,000


Expenses

© The Institute of Chartered Accountants of Nepal 12


Suggested Answer Paper Group II

j) Add: Variable selling 4 20,000 4 34,000


Expenses

k) Add: Fixed Selling 2.4 12,000 1.41 12,000


Expenses

Total Cost of Sales (h+i+j+k) 440,000 675,400

II) Calculation of credit purchase


Particulars Year I Year II

A. Raw Material Consumed 240,000 360,000

B. Add: Closing Stock 45,000 67,500

C. Less: Opening Stock 0 (45,000)


D. Purchases(A+B-C) 285,000 382,500

(b)
Face value of share = Rs. 100
Current market price = Rs. 480
Annual expected dividend = 30% (1-4 years)
During 5th year bonus issue made in the ratio of 1:5.
After bonus issue the face value becomes (equivalent) Rs. 120.
Dividend rate of 30% is maintainable on expanded capital base of Rs. 120.
Then, dividend is Rs. 120 X 30/100 =Rs. 36 (5-8 years)

Particulars Rs.
Value of share at the end of 8th Year 1000
Add: 20% addition for bonus issue 200
1200
Less: Incidental expenses on sale @ 5% 60
Net Sales Realization 1140

Calculation of present value of the net benefit @ 15% DCF (Rs.)

Dividend for 1 to 4 years (Rs 30X2.855) 85.65

Dividend for 5 to 8 years (Rs 36X1.632) 58.75

Sales realization after the end of 8th year (1140X0.327) 372.78

517.18

Less: Cost of shares (Rs 480+ 5% incidental expenses) 504.00

Present value of the net benefit 13.18

© The Institute of Chartered Accountants of Nepal 13


Suggested Answer Paper Group II

Yes, the investor should buy the share because of positive PV of net benefit. The maximum
price can be Rs. 517.18 including incidental expenses.

5.
a. Following is the data regarding six securities:

Securities A B C D E F

Return (%) 8 8 12 4 9 8

Risk (%) 4 5 12 4 5 6

Required: (2+3=5 marks)

i. Which of the securities will be selected for investment?

ii. Assuming perfect positive correlation, analyse whether it is preferable to invest


75% of fund in security A and 25% in security C.

b. Company A has following details:

Sales (30,000 units @ Rs. 100 per unit) Rs. 3,000,000


Variable Cost Rs. 40 per unit
Fixed Cost Rs. 900,000
It has borrowed Rs. 1,500,000 @ 10% p.a. and its equity share capital is Rs. 1,000,000 of
Rs. 100 each shares. It is taxed @ 30% for income tax.
Required: (3+2=5 marks)
i. Calculate operating, financial and combined leverage, and also the return on
investment.

ii. If the sales increases by Rs. 500,000 what will be the new EBIT?

c. A Mutual Fund having 500 units has shown its Net Assets value (NAV) per unit of Rs. 10.25
and Rs. 11.50 at the beginning and at the end of the year respectively. The Mutual Fund
has decided to distribute Re. 1 per unit as dividend and Re. 0.50 per unit as capital gain.
Further, it is offering to reinvest these distributions at an average NAV of Rs. 10 per unit.
Required: (5 marks)
Suggest on the attractiveness of the offer.
Answer:
a. .

i. Security A has a return of 8% for a risk of 4%, whereas securities B and F have a
higher risk for the same rate of return. Hence security A dominates securities B and
F. For the same degree of risk of 4% security D has only a return of 4%. Hence, this
security is also dominated by A. Securities C and E has a higher return as well as a
higher degree of risk. Hence the securities which will be selected are A, C and E.

© The Institute of Chartered Accountants of Nepal 14


Suggested Answer Paper Group II

ii. When perfect positive correlation exists between two securities, their risk and return
can be averaged with the proportion. Hence the average value of A and C together
for a proportion of 3 : 1 for risk and return will be as follows:
Risk = 0.75×4+0.25×12 = 6%
Return 0.75×8+0.25×12 = 9%
Comparing the above average risk and return with security E, it is better to invest in
E as it has lesser risk (5%) for the same return of 9%.
b. Calculation of EBIT and EPS:
Particulars Amount in Rs.
Sales revenue 3,000,000
Less: Variable cost 1,200,000
Contribution margin 1,800,000
Less: Fixed cost 900,000
EBIT 900,000
Less: Interest 150,000
EBT 750,000
Less: Tax @ 30% 225,000
EAT 525,000
No. of equity shares 10,000
EPS 52.50
𝐶𝐶𝐶𝐶𝐶𝐶𝐶𝐶𝐶𝐶𝐶𝐶𝐶𝐶𝐶𝐶𝐶𝐶𝐶𝐶𝐶𝐶𝐶𝐶 𝑀𝑀𝑀𝑀𝑀𝑀𝑀𝑀𝑀𝑀𝑀𝑀 1,800,000
Operating leverage (OL) = 𝐸𝐸𝐸𝐸𝐸𝐸𝐸𝐸
= 9000,000
= 2 times

𝐸𝐸𝐸𝐸𝐸𝐸𝐸𝐸 900,000
Financial leverage (FL) = 𝐸𝐸𝐸𝐸𝐸𝐸 = 750,000 = 1.2 times

Combined leverage (CL) = OL × FL = 2×1.2 = 2.4 times


Return on investment = (EAT+I)/ (Equity share capital + Debt)
= (525,000+150,000)/ (1,000,000+1,500,000)
= 675,000/2,500,000 = 27%
(Note: Return on investment can alternatively be calculated by using EBIT/(Equity share
capital + debt) and in this case it will be 36%
If sales increases by Rs. 500,000 i.e. by 500,000/3,000,000 = 16.67% the new
EBIT will be given by the degree of operating leverage as under:
% 𝐶𝐶ℎ𝑎𝑎𝑎𝑎𝑎𝑎𝑎𝑎 𝑖𝑖𝑖𝑖 𝐸𝐸𝐸𝐸𝐸𝐸𝐸𝐸
Degree of operating leverage =% 𝑜𝑜𝑜𝑜 𝐶𝐶ℎ𝑎𝑎𝑎𝑎𝑎𝑎𝑎𝑎 𝑖𝑖𝑖𝑖 𝑆𝑆𝑆𝑆𝑆𝑆𝑆𝑆𝑆𝑆
% 𝑜𝑜𝑜𝑜 𝐶𝐶ℎ𝑎𝑎𝑎𝑎𝑎𝑎𝑎𝑎 𝑖𝑖𝑖𝑖 𝐸𝐸𝐸𝐸𝐸𝐸𝐸𝐸
Or, 2 = 0.167

Or, % of change in EBIT = 0.334


So, increase in EBIT = Rs. 900,000 × 0.334 = Rs. 300,000
Therefore, new EBIT = 900,000 + 300,000 = Rs. 12,00,000

© The Institute of Chartered Accountants of Nepal 15


Suggested Answer Paper Group II

c. Change in value over the year = Rs. 11.50 - Rs. 10.25 = Rs. 1.25
Dividend received Rs. 1.00
Capital gain received Rs. 0.50
Total returns Rs. 2.75
(𝑁𝑁𝑁𝑁𝑁𝑁1 −𝑁𝑁𝑁𝑁𝑁𝑁0 )+𝐷𝐷𝐷𝐷𝐷𝐷𝐷𝐷𝐷𝐷𝐷𝐷𝐷𝐷𝐷𝐷+𝐶𝐶𝐶𝐶𝐶𝐶𝐶𝐶𝐶𝐶𝐶𝐶𝐶𝐶 𝐺𝐺𝐺𝐺𝐺𝐺𝐺𝐺
Holding period return = 𝑁𝑁𝑁𝑁𝑁𝑁0

(11.5−10.25)+1+0.50
= 10.25
2.75
= 10.25 = 0.2683 or 26.83%

Per unit cash proceeds as return = Dividend + Capital Gain = 1 + 0.5 = Rs. 1.5
Total proceeds = Total Number of Units × Per unit cash proceeds = 500×1.5 = Rs.750
750
Total units available as reinvestment from cash proceeds = 10
= 75 units

Total units upon reinvestment = 500 + 75 = 575 units


Value of units at the end of year = 575 × 11.50 = Rs. 6,612.50
Value of units at the beginning of year = 500 × 10.25 = Rs. 5,125
Total gain upon reinvestment = 6,612.50 – 5,125 = Rs. 1,487.50
Holding period return
𝑇𝑇𝑇𝑇𝑇𝑇𝑇𝑇𝑇𝑇 𝑔𝑔𝑔𝑔𝑔𝑔𝑔𝑔 1487.5
=𝑉𝑉𝑉𝑉𝑉𝑉𝑉𝑉𝑉𝑉 𝑜𝑜𝑜𝑜 𝑢𝑢𝑢𝑢𝑢𝑢𝑢𝑢𝑢𝑢 𝑎𝑎𝑎𝑎 𝑡𝑡ℎ𝑒𝑒 𝑏𝑏𝑏𝑏𝑏𝑏𝑏𝑏𝑏𝑏𝑏𝑏𝑏𝑏𝑏𝑏𝑏𝑏 = 5125
= 0.2902 or 29.02%

Suggestion:
Since the holding period return is more i.e. 29.02% by making reinvestment of the
proceeds from distribution against retaining the proceeds that would provide holding
period return of 26.83%, reinvestment of distribution proceeds as offered by the Mutual
Fund is suggested.
6. Write short note/ answer on: (4×2.5=10 marks)
a) Bridge finance
b) Trading on equity
c) Crowd funding
d) Trade credit
Answer:
a) Bridge finance refers, normally, to loans taken by the business, usually from commercial
banks for a short period, pending disbursement of term loans by financial institutions.
Normally it takes time for the financial institution to finalize procedures of creation of
security, tie-up participation with other institutions etc. even though a positive appraisal of
the project has been made. However, once the loans are approved in principle, firms in order
not to lose further time in starting their projects arrange for bridge finance. Such temporary

© The Institute of Chartered Accountants of Nepal 16


Suggested Answer Paper Group II

loan is normally repaid out of the proceeds of the principal term loans. It is secured by
hypothecation of moveable assets, personal guarantees and demand promissory notes.
Generally rate of interest on bridge finance is higher as compared with that on term loans.
b) The term trading on equity means debts are contracted and loans are raised mainly on the
basis of equity capital. Those who provide debt have a limited share in the firm’s earnings
and hence want to be protected in terms of earnings and values represented by equity capital.
Since fixed charges on debts do not vary with firm’s earnings before interest and tax, a
magnified effect is produced on earnings per share called financial leverage. Whether the
leverage is favourable, in that increase in earnings per share more proportionately to the
increased earnings before interest and tax, depends on the profitability of the investment
proposal. If the rate of returns on investment exceeds their explicit cost, financial leverage
is said to be positive.
c) Crowd funding is the practice of raising money from a large number of individuals for the
purposes of financing a project, venture, business or cause. Traditionally, crowd funding
has been carried out via subscriptions, benefit events and door-to-door fundraising.
However, today the term is typically associated with raising money through website
platforms, which allows crowdfunding to reach a larger pool of potential funders.
Crowdfunding usually takes place on a light-touch online platform rather than through
banks, charities or stock exchanges. The business or individual seeking finance will
typically produce a pitch for their business, project or venture, which is then uploaded to
the online platform with the aim of attracting as many loans, contributions and investments
as possible. Websites such as Kickstarter, Seedrs and Crowdcube are examples of the
available online platforms, which enable project initiators to reach a pool of thousands, if
not millions, of potential funders.
d) Trade credit refers to an arrangement whereby the supplier of the raw materials,
components, stores and spare parts, finished goods, allow the customers to pay their
outstanding balances within the credit period allowed by them. Generally, suppliers grant
credit for a period of three to six months and thus provide short term funds to finance current
assets. The availability of trade credit depends upon various factors such as nature and size
of the firm, status of the firm (i.e. credit worthiness), activity level of the firm, policy of
trade credit suppliers, prevailing economic conditions etc. The major limitations of trade
credit include ready availability, absence of issue formalities etc. The major limitation of
trade credit is that it involves loss of cash discount, which could be earned if payments were
made within seven to ten days from the date of purchase. This loss is regarded as the cost
of trade credit.
7. Distinguish between: (4×2.5=10 marks)
a) Foreign direct investment (FDI) and Foreign portfolio investment (FPI)
b) Credit standard and Credit analysis
c) IRR and MIRR
d) Gearing and Leverage
Answer:
a) Foreign direct investment (FDI) and Foreign portfolio investment (FPI)
Foreign direct investment (FDI) Foreign portfolio investment (FPI)
Investment involves creation of physical Investment is only in financial assets

© The Institute of Chartered Accountants of Nepal 17


Suggested Answer Paper Group II

assets

Has a long term interest and therefore Only short term interest and generally remain
remain invested for long invested for short periods
Relatively difficult to withdraw Relatively easy to withdraw
Not inclined to be speculative Speculative in nature
Often accompanied by technology transfer Not accompanied by technology transfer

Direct impact on employment of labour and No direct impact on employment of labour and
wages. wages
Enduring interest in management and No abiding interest in management and
control control
Securities are held with significant degree Securities are held purely as a financial
of influence by the investor on the investment and no significant degree of
management of the enterprise influence on the management of the enterprise

b) Both of these are related to credit policy a part of credit management or receivables
management of a firm. However, the difference among them can be inferred from
following discussion:
Credit standard is the basis or the criteria based on which credits are flown by a firm. The
strength of credit standard determines the extent of credit flown by the firm. A tight credit
standard results in the low receivable balance whereas loose credit standard leads to higher
receivable balance. The basis or criteria of credit standard are credit eligibility, credit
rating, credit references, credit segment, credit period etc.
Credit analysis is the evaluation of a potential debtor for the purpose of decision on granting
credit. Internal and external quantitative and qualitative information on debtor and their
credit histories are collected and analyzed. If they fulfill the credit granting criteria, they
are assigned certain credit limits based on the quality of credit found.
c) IRR and MIRR
The points given below are substantial so far as the difference between IRR and MIRR is
concerned:
 Internal Rate of Return or IRR implies a method of reckoning the discount rate
considering internal factors, i.e. excluding the cost of capital and inflation. On the other
hand, MIRR alludes to the method of capital budgeting, which calculates the rate of
return taking into account cost of capital. It is used to rank various investments of the
same size.
 The internal rate of return is an interest rate at which NPV is equal to zero. Conversely,
MIRR is the rate of return at which NPV of terminal inflows is equal to the outflow,
i.e. investment.
 IRR is based on the principle that interim cash flows are reinvested at the project’s IRR.
Unlike, under MIRR, cash flows apart from initial cash flows are reinvested at firm’s
rate of return.
 The accuracy of MIRR is more than IRR, as MIRR measures the true rate of return.
d) Gearing and leverage are terms associated with the utilization of debt for the purpose of
employing those funds in business operations.

© The Institute of Chartered Accountants of Nepal 18


Suggested Answer Paper Group II

Leverage refers to the amount of funds that are borrowed by a business and directed
towards investments with the aim of obtaining a high return.
Gearing is the measurement of the level of debt alongside the amount of equity held within
a firm. The higher the levels of debt that is utilized higher the gearing of the firm.
The main similarity between leverage and gearing is that they the gearing ratio is derived
from evaluating the levels of debt within the firm. The higher the leverage the higher the
gearing ratio, and higher the risk faced by the firm.

© The Institute of Chartered Accountants of Nepal 19


Suggested Answer Paper Group II

Paper 5: Cost and Management Accounting


All questions are compulsory. Working notes should form part of the answer.
Make assumptions wherever necessary.
1. A company within the chemical industry mixes powdered ingredients in two different
processes to produce one product. The output of Process 1 becomes the input of Process 2
and the output of Process 2 is transferred to the packing department.
From the information given below, you are required to open accounts for Process 1,
Process 2, abnormal loss and packing department and to record the transactions for the
week ended 11th June 2019. (20 marks)
Process 1
Input:
Material A 6,000 kilograms@ Rs. 1 per kilogram
Material B 4,000 kilograms@ Rs. 2 per kilogram
Mixing labour 430 hours@ Rs. 4 per hour
Normal loss 5% of weight input, disposed off @ 32 paise per kilogram
Output 9,200 kilograms.
No work in progress at the beginning or end of the week.
Process 2
Input 2:
Material C 6,600 kilograms@ Rs. 2.50 per kilogram
Material D 4,200 kilograms@ Rs. 1.50 per kilogram
Flavouring essence Rs. 600
Mixing labour 370 hours@ Rs. 4 per hour
Normal wastage 5% of weight input with no disposal value
Output 18,000 kilograms.
There was no work in process at the beginning of the week but 1,000 kilograms were in
process at the end of the week that was estimated to be only 50% complete so far as labour
and overhead were concerned.
Overhead of Rs. 6,400 incurred by the two processes to be absorbed on the basis of mixing
labour hours.
Answer:
Process 1 Account
Particulars Kg Per Kg Amount Particulars Kg. Per Kg Rs. Amount
Rs. Rs. Rs.
To Material A 6,000 1.00 6,000 By Normal loss 500 0.32 160
To Material B 4,000 2.00 8,000 By Abnormal Loss (See 300 2.00 600
note 2)
To Mixing Labour (430 hours @ Rs. 4 1,720 By transfer to Process 2 9,200 2.00 18,400
per hour)
To Overhead (See Note 1) 3,440

© The Institute of Chartered Accountants of Nepal 20


Suggested Answer Paper Group II

10,000 19,160 10,00 19,160


0

Process 2 Account
Particulars Kg Per Kg Amount Particulars Kg. Per Kg Rs. Amount
Rs. Rs. Rs.
To Process 1 9,200 2.00 18,400 By Normal waste 1,000 -
To Material C 6,600 2.50 16,500 By Work in process 1,000 2,320
(Note 3)
To Material D 4,200 1.50 6,300
To flavouring Essence 600
To Mixing Labour (370 hours @ Rs. 4 per 1,480 By Packing Deptt. 18,000 2.44 43,920
hour)
To Overhead (See Note 1) 2,960
20,000 46,240 20,000 46,240

Abnormal Loss Account


Particulars Kg Per Kg Amount Particulars Kg. Per Kg Rs. Amount
Rs. Rs. Rs.
To Process 1 300 2.00 600 By Sales A/c 300 0.32 96
By Balance to P/L 504
A/c

Packing Department Account


Particulars Kg Per Kg Amount Particulars Kg. Per Kg Amount
Rs. Rs. Rs. Rs.
To Process 2 A/c 18,000 2.44 43,920 By Balance c/d 18,000 2.44 43,920
18,000 43,920 18,000 43,920

Notes:
1. Total Overhead expenses= Rs. 6,400
Total labour hours in Process 1 and 2=800
Overhead absorption rate=Rs.6,400/800 hours= Rs. 8 per labour hour
Overhead under process 1=430* Rs. 8=Rs. 3,440
Overhead under process 2=370* Rs. 8=Rs. 2,960
2. Cost of 9,500 kg of output is = (Rs.19,160-Rs. 160) i.e. Rs. 19,000
Hence, cost per kg. of output is Rs. 2.00
3. Equivalent Units Statement of Output
Particulars Output units Equivalent Units
Material Labour Overhead
Completed 18,000 18,000 18,000 18,000
WIP(100% Material, 50% 1,000 1,000 500 500
Labour & Overhead)
Normal Waste 1,000
20,000 19,000 18,500 18,500

Cost statement for the week ending 11th June 2019

© The Institute of Chartered Accountants of Nepal 21


Suggested Answer Paper Group II

Particulars Amount Rs
Material (Process 1) 18,400
Material C 16,500
Material D 6,300
Flavouring Essence 600
Total Material Cost 41,800
Total Mixing labour cost 1,480
Total Overhead cost 2,960
Cost per Equivalent Unit
Material=Rs. 41,800/19,000=Rs. 2.20
Labour=Rs. 1,480/18,500=0.08 P
Overhead = Rs. 2,960/18,500=0.16 P
Work-in-Progress (WIP)
Material =1,000*2.20 =Rs. 2,200
Labour =500*0.08 =Rs. 40
Overhead =500*0.16 =Rs.80
=Rs. 2,320
2.
a) Gemini Furnitures produces and markets three product; Chairs, Tables and Benches,
intended to be used by schools and colleges. The company is interested in presenting
its budget for the next quarter ending 31st December 2019. It expects to sell 4,200
chairs, 800 tables and 500 benches during the said period at the selling price of Rs.
500, Rs. 850, and Rs. 1,580 per unit respectively. The following information are made
available for the purpose:
i) Material and labour requirements:
Chairs Tables Benches
Timber per unit (in cubic feet) 0.5 1.2 2.5
Upholstery per unit (in square yards) 0.25 -- --
Carpenter's time (minutes per unit) 45 60 75
Fixer and finisher's time (minutes per unit) 15 15 30
Timber costs Rs. 500 per cubic feet and upholstery costs Rs. 200 per square yard.
Fixing and finishing material costs 5% of the cost of timber and upholstery.
Carpenter gets Rs. 60 per hour while the fixer and finisher get Rs. 48 per hour.
ii) Inventory levels planned:
Timber Upholstery Chairs Tables Benches
(cu. ft.) (Sq. yds.) (nos.) (nos.) (nos.)
Opening 600 400 400 100 50
Closing 650 260 200 300 50
iii) Fixed overheads would be Rs. 80,000 per month.
You are required to: (10 marks)

© The Institute of Chartered Accountants of Nepal 22


Suggested Answer Paper Group II

i) Prepare a production budget showing quantities to be manufactured.


ii) Prepare raw materials purchase budget in quantities as well as in rupees.
iii) Draw a direct wage cost budget.
iv) Present a statement showing variable cost of manufacture per unit of all three
products.
v) Find out the budgeted net income for the said quarter.
b) The financial books of a company reveal the following data for the year ended, 31st
March, 2018

Particulars Amount (Rs.)


Opening stock: Finished goods 875 units 74,375
Work-in-process 1.4.2017 32,000
Raw materials consumed 780,000
Direct labour 450,000
Factory overheads 300,000
Goodwill 100,000
Administration overheads 295,000
Dividend paid 85,000
Bad debts 12,000
Selling and distribution overheads 61,000
Interest received 45,000
Rent received 18,000
Sales 14,500 units 2,080,000
Closing stock: Finished goods 375 units 41,250
Work-in-process 31.03.2018 38,667
The cost records provide as under:
• Factory overheads are absorbed at 60% of direct wages.
• Administration overheads are recovered at 20% of factory cost.
• Selling and distribution overheads are charged at Rs. 4 per unit sold.
• Opening stock of finished goods is valued at Rs. 104 per unit.
• The company values work-in-process at factory cost for both financial and cost
profit reporting.
Required: (10 marks)
st
i) Prepare statements for the year ended 31 March, 2018 show
• The profit as per financial records
• The profit as per costing records.
ii) Present a statement reconciling the profit as per costing records with the profit as
per financial records.
Answer:
a)
i) Production Budget

© The Institute of Chartered Accountants of Nepal 23


Suggested Answer Paper Group II

(Showing quantities to be manufactured)


Chairs Tables Benches
Units to be sold 4,200 800 500
Add: Planned closing inventory 200 300 50
4,400 1,100 550
Less: Planned opening inventory 400 100 50
Units to be manufactured 4,000 1,000 500
ii) Material Purchase Budget
(in quantities)
Timber Upholstery
(cu. ft.) (Sq. yds.)
Materials required for production 4,450 1,000
(Working Note 1)
Add: Planned closing raw materials 650 260
5,100 1,260
Less: Planned opening raw materials 600 400
Raw- materials to be purchased 4,500 860
Material Purchase Budget
(in Rupees)
Quantity Rate (Rs.) Amount (Rs.)
Timber (cu. ft.) 4,500 500 2,250,000
Upholstery (Sq. yds.) 860 200 172,000
2,422,000
iii) Direct Wages Cost Budget
Total Hrs. Rate p.h. (Rs.) Amount (Rs.)
Carpenter (Refer WN 1) 4,625 60 277,500
Fixer and Finisher (Refer WN 1) 1,500 48 72,000
Total Direct wages 349,500
iv) Statement Showing Variable Cost per unit
Rs.
Chairs Tables Benches
Raw Materials:
Timber 250.00 600.00 1,250.00
(0.5 ×Rs. 500) (1.2 × Rs. 500) (2.5 × Rs.500)
Upholstery 50.00 ----------- -----------
(0.25 × Rs.200)
Fixing and Finishing 15.00 30.00 62.50
Rs. (250+50) × 5% Rs. (600+0) × 5% Rs. (1,250+0) × 5%
Wages:
Carpenter 45.00 60.00 75.00
(45/60 × Rs.60) (60/60 × Rs.60) (75/60 × Rs.60)
Fixer and Finisher 12.00 12.00 24.00
(15/60 × Rs.48) (15/60 × Rs.48) (30/60 × Rs.48)
Total variable cost 372.00 702.00 1,411.50
v) Budgeted Net Income Statement
(For the quarter)
Chairs Tables Benches Total
Rs. Rs. Rs. Rs.
Selling price per unit 500.00 850.00 1,580.00

© The Institute of Chartered Accountants of Nepal 24


Suggested Answer Paper Group II

Less: Variable cost per unit 372.00 702.00 1,411.50


Contribution per unit (A) 128.00 148.00 168.50
Units to be sold (B) 4,200 800 500
Total Contribution (A) × (B) 537,600 118,400 84,250 740,250
Less: Fixed Cost for quarter 240,000
(80,000×3)
Budgeted net income 500,250

Working Notes:
1. Raw materials, Carpenter's time and Finisher and Fixer's time required for production
Chairs Tables Benches Total
Units to be manufactured 4,000 1,000 500
Timber (cu. ft.) 2,000 1,200 1,250 4,450
(4,000 × 0.5) (1,000 × 1.2) (500 × 2.5)
Upholstery (Sq. yds.) 1,000 -------- -------- 1,000
(4,000 × 0.25)
Carpenter's time (Hrs.) 3,000 1,000 625 4,625
(4,000 × 45/60) (1,000 × 60/60) (500 × 75/60)
Fixer and Finisher's time 1,000 250 250 1,500
(Hrs.) (4,000 × 15/60) (1,000 × 15/60) (500 × 30/60)

b) Profit & Loss Account


for the year ended 31st March, 2018
Dr. Cr.
` `
To Opening stock of Finished 74,375 By Sales 2,080,000
goods
To Work-in-process 32,000 By Closing stock of finished 41,250
goods
To Raw materials consumed 780,000 By Work-in-Process 38,667
To Direct labour 450,000 By Rent received 18,000
To Factory overheads 300,000 By Interest received 45,000
To Goodwill 100,000
To Administration overheads 295,000
To Selling & distribution 61,000
overheads
To Dividend paid 85,000
To Bad debts 12,000
To Profit 33,542
2,222,917 2,222,917

Statement of Profit as per costing records


for the year ended March 31,2018
Sales revenue (A) (14,500 units) ` 2,080,000
Cost of sales:
Opening stock (875 units x Rs. 104) 91,000
Add: Cost of production of 14,000 units (Refer to WN 2) 1,792,000

Less: Closing stock = Rs. 1,792,000×375 units/14,000 units 48,000

© The Institute of Chartered Accountants of Nepal 25


Suggested Answer Paper Group II

Production cost of goods sold (14,500 units) 1,835,000


Selling & distribution overheads(14,500 units x ` 4) 58,000
Cost of sales: (B) 1,893,000
Profit: {(A) – (B)} 187,000
ii) Statement of Reconciliation
(Reconciling the profit as per costing records with the profit as per financial records)
` `
Profit as per Cost Accounts 187,000
Add: Administration overheads over absorbed 3,667
(Rs. 298,667 – Rs. 295,000)
Opening stock overvalued (Rs. 91,000 – Rs. 74,375) 16,625
Interest received 45,000
Rent received 18,000 83,292
270,292
Less: Factory overheads under recovery ( Rs.. 300,000 – Rs. 30,000
270,000)
Selling & distribution overheads under recovery ( Rs. 61,000 – Rs. 3,000
58,000)
Closing stock overvalued (Rs. 48,000 – Rs. 41,250) 6,750
Goodwill 100,000
Dividend 85,000
Bad debts 12,000 236,750
Profit as per financial accounts 33,542

Working notes :
1. Number of units produced Units
Sales 14,500
Add: Closing stock 375
Total 14,875
Less: Opening stock 875
Number of units produced 14,000
2. Cost Sheet `
Raw materials consumed 780,000
Direct labour 450,000
Prime cost 1,230,000
Factory overheads(60% of direct wages) 270,000
Factory cost 1,500,000
Add: Opening work-in-process 32,000
Less: Closing work-in-process 38,667
Factory cost of goods produced 1,493,333
Administration overheads (20% of factory cost) 298,667
Cost of production of 14,000 units (Refer to WN 1) 1,792,000

Cost of production per unit :


Total Cost of Production/ No. of units produced
= Rs. 1,792,000/14,000 units = Rs. 128

© The Institute of Chartered Accountants of Nepal 26


Suggested Answer Paper Group II

3.
a) The existing Incentive system of Max Ltd. is as under:
Normal working week 5 days of 8 hours each plus 3 late shifts of 3 hours each
Rate of payment Day work: Rs. 160 per hour
Late shift: Rs. 225 per hour
Average output per operator for 49 hours i.e. including 3 late shifts=120 articles
In order to increase output and eliminate overtime, it was decided to switch on to a
system of payment by results. The following information is obtained:
Time rate (as usual): Rs. 160 per hour.
Basic time allowed for 15 articles: 5 hours
Piece work rate: Add 20% to basic piece rate
Premium bonus: Add 50% to time.
Required:
Prepare a statement showing hours worked, weekly earnings, number of articles
produced and labour cost per article for one operator under the following system:
(8 marks)
i) Existing time rate
ii) Straight piece work
iii) Rowan system
iv) Halsey premium system
Assume that 135 articles are produced in a 40 hour/week under straight piece work,
Rowan Premium system and Halsey premium system above and worker earns half the
time saved under Halsey premium system.
b) Speed Transport Company Ltd. has a fleet of three trucks of 10 tonnes capacity each
plying in different directions for transport of customer's goods. The trucks run loaded
with goods and return empty. The distance travelled, number of trips made and the
load carried per day by each truck are as under:
Truck No. One way distance No. of trips per day Load carried per
(Km) trip / day (tonnes)
1 16 4 6
2 40 2 9
3 30 3 8
The analysis of maintenance cost and the total distance travelled during the last two
years are as under:
Year Total distance travelled (Km) Maintenance cost
1 160,200 Rs. 46,050
2 156,700 Rs. 45,175

The following are the details of expenses for the year under review:
Diesel: Rs. 100 per litre. Each litre gives 4 km per litre of diesel on an average.
Driver's salary: Rs. 20,000 per month
Licence and taxes: Rs. 5,000 per annum per truck

© The Institute of Chartered Accountants of Nepal 27


Suggested Answer Paper Group II

Insurance: Rs. 5,000 per annum for all the three vehicles.
Purchase price per truck: Rs. 3,000,000 Life 10 years. Scrap value at the end of life is
Rs. 100,000.
Oil and sundries: Rs. 250 per 100 Km. run.
General overhead: Rs. 110,840 per annum
The vehicles operate 24 days per month on an average.
Required: (8 marks)
i) Prepare an Annual Cost Statement covering the fleet of three vehicles.
ii) Calculate the cost per km run.
iii) Determine the freight rate per tonne km to yield a profit of 10% on freight.
c) What do you mean by time and motions study? Why is it so important to management?
(4 marks)

Answer:
a) Statement showing hours worked, weekly earnings, number of articles produced and
labour cost per article under various wage system
Particulars (Scheme) Hours Weekly earning Rs. No. of articles Labour cost
worked produced per article Rs.
Existing time rate 49 8,425 (W.N.-1) 120 70.21
Straight Piece work 40 8,640 (W.N.-2) 135 64.00
Rowan system 40 9,007.41 (W.N.-3) 135 66.72
Halsey system 40 8,600 (W.N.-4) 135 63.70
Working notes:
Existing time rate: Weekly wages 40 hrs @Rs. 160 per hour =Rs.6,400
9 hours @Rs. 225 per hour =Rs. 2,025
Weekly earnings =Rs. 8,425
Piece rate system:
Basic time: 5 hours for 15 articles
Cost of 15 articles at hourly rate of Rs. 160 per hour =Rs. 800
Add: 20% =Rs.160
=Rs. 960
Therefore, Rate per article (Rs. 960/15) =Rs. 64
And earnings for the week=135*Rs.64 = Rs. 8,640
Rowan premium system:
Basic time=5 hours for 15 articles
Add: 50% to time i.e. 2.5 hours for 15 articles
Total time=7.5 hours for 15 articles or, 30 mins per article
Therefore, time allowed for 135 articles=67.5 hours
Actual time taken for 135 articles= 40 hours
Time taken × time rate + time taken/time allowed × time saved × rate per hour
=40×160+27.5×40/67.5×160 = Rs. 9,007.41
Halsey Premium system:
Earnings = Time taken × time rate + 50% of time saved×time rate
= 40*160+1/2(67.5-40)*Rs.16 = Rs. 8,600

© The Institute of Chartered Accountants of Nepal 28


Suggested Answer Paper Group II

b)
(i) Annual Cost Statement of three vehicles
Rs. '
Diesel 3,369,600
(Refer to working note I)
(134,784 kms / 4 km) × Rs. 100
Oil and sundries 336,960
(134,784 kms/100 kms) × Rs. 250
Maintenance 39,696
(Refer to working note 2)
{(134,784 kms × 0.25P) + Rs. 6,000}
Drivers' salary 720,000
(Rs. 20,000 × 12 months) × 3 trucks
Licence and taxes 15,000
Insurance 5,000
Depreciation 870,000
(Rs. 2,900,000/10 years) × 3 trucks
General overhead 110,840
Total annual cost 5,467,096
(ii) Cost per km. run
Cost per kilometer run = Total annual cost of vehicles
(Refer to working note 1) Total kilometre travelled annually
=Rs. 5,467,096
134,784 Kms
= Rs. 40.56
(iii) Freight rate per tonne km (to yield a profit of 10% on freight)
Cost per tonne km. = Total annual cost of three vehicles
(Refer to working note 1) Total effective tonnes kms per annum
= Rs. 5,467,096
525,312 kms
= Rs. 10.4073
Freight rate per tonne km. = Rs. 11.5637 (Rs. 10.4073/ 9 × 10)
Working notes:
1.Total kilometre travelled and tonnes kilometre (load carried) by three trucks in one
year
Truck One way No. of trips Total distance Load carried Total effective
number distance in covered in km per trip / day tonnes km
kms per day in tonnes
1 16 4 128 6 384
2 40 2 160 9 720
3 30 3 180 8 720
Total 468 1824

Total kilometre travelled by three trucks in one year = 134,784


(468 kms × 24 days × 12 months)
Total effective tonnes kilometre of load carried by three trucks during one year
= 525,312
© The Institute of Chartered Accountants of Nepal 29
Suggested Answer Paper Group II

(1,824 tonnes km × 24 days × 12 months)


2. Fixed and variable component of maintenance cost:
Variable maintenance cost per km = Difference in maintenance cost
Difference in distance travelled
= Rs. 46,050 – Rs.45,175
160,200 kms – 156,700 kms
= Rs. 0.25
Fixed maintenance cost = Total maintenance cost–Variable maintenance cost
= Rs. 46,050 – (160,200 kms × 0.25)
= Rs. 6,000
c) Time and motions study:
It is the study of time taken and motions (movements) performed by workers while
performing their jobs at the place of their work. Time and motion study has played a
significant role in controlling and reducing labour cost. Time Study is concerned with
the determination of standard time required by a person of average ability to perform a
job. Motion study, on the other hand, is concerned with determining the proper method
of performing a job so that there are no wasteful movements, hiring the worker
unnecessarily. However, both the studies are conducted simultaneously. Since
materials, tools, equipment and general arrangement of work, all have vital bearing on
the method and time required for its completion. Therefore, their study would be
incomplete and would not yield its full benefit without a proper consideration of these
factors.
Time and motion study is important to management because of the following features:
i) Improved methods, layout, and design of work ensures effective use of men,
material and resources.
ii) Unnecessary and wasteful methods are pin-pointed with a view to either improving
them or eliminating them altogether. This leads to reduction in the work content of
an operation, economy in human efforts and reduction of fatigue.
iii) Highest possible level of efficiency is achieved in all respect.
iv) Provides information for setting labour standards - a step towards labour cost
control and cost reduction.
v) Useful for fixing wage rates and introducing effective incentive scheme.
4.
a) ABC Ltd. produces three joint products X, Y and Z. The products are processed further.
Pre-separation costs are apportioned on the basis of weight of output of each joint
product. The following data are provided for month just concluded:
Cost incurred up to separation point is Rs. 10,000.
Product X Product Y Product Z
Output (in litre) 100 70 80
Rs. Rs. Rs.
Cost incurred after separation point 2,000 1,200 800

© The Institute of Chartered Accountants of Nepal 30


Suggested Answer Paper Group II

Selling price per litre:


After further processing 50 80 60
At separation point (estimated) 25 70 45
You are required to: (5 marks)
i) Prepare a statement showing profit or loss made by each product using the present
method of apportionment of pre-separation cost, and
ii) Advice the management whether, on purely financial consideration, the three
products are to be processed further.
b) A company sold in two period 7,000 units and 9,000 units of its produce and has
incurred a loss of Rs. 10,000 and earned a profit of Rs. 10,000 respectively. The selling
price per unit is Rs. 100.
Calculate the amount of fixed cost and units sold at breakeven. (5 marks)
c) A Ltd. manufactures a product X which requires two raw materials A and B in a ratio
of 1:4. The sales department has estimated a demand of 500,000 units for the product
for the year. To produce one unit of finished product, 4 units of material A is required.
Stock position at the beginning of the year is as below:
Product X 12,000 units
Material A 24,000 units
Material B 52,000 units
To place an order the company has to spend Rs. 15,000. The Company is financing its
working capital using a bank cash credit @13% p.a.
Product X is sold at Rs. 1,040 per unit. Material A and B are purchased at Rs. 150 and
Rs. 200 respectively
Required: (5 marks)
Compute economic order quantity (EOQ):
i) If purchase order for the both materials is placed separately
ii) If purchase order for the both materials is not placed separately.
Answer:
a)
i) Statement of profit for three joint products
Product X Product Y Product Z Total
Rs. Rs. Rs. Rs.
Sales 5,000 5,600 4,800 15,400
Less:
Pre Separation Costs 4,000 2,800 3,200 10,000
Post Separation Cost 2,000 1,200 800 4,000
Profit/(Loss) (1,000) 1,600 800 1,400
ii) Decision whether to further process the product or not:

Product Incremental Incremental Costs Incremental


Revenue Profit/(Loss)
Rs. Rs. Rs.

© The Institute of Chartered Accountants of Nepal 31


Suggested Answer Paper Group II

X (Rs.25x100) 2,500 2,000 500


Y (Rs.10x 70) 700 1,200 (500)
Z (Rs. 15x80) 1,200 800 400
Total 400
Product X and Z should be further processed. Y should be sold at point of separation.
W.N.
Pre-seperation cost (Rs.)
X: 10,000/(100+70+80) ×100 = 4,000
Y: 10,000/250×70 = 2,800
Y = 10,000/250×80
=3,200
b)
Calculation of PV Ratio
PV Ratio= Change in Profit
Change in Sales
10,000-(-10,000)
900,000-700,000
20,000
200,000
Pv ratio 10%
Fixed Cost Sales * PV Ratio- Profit
=900,000*10%-10,000
80,000.00
BEP FC/ Contribution per unit
= 80,000/(100*10%)
= 8,000 Units

Working Note:
Sale 70,000 units 9,000 units
Profit(loss) (Rs. 10,000) Rs. 10,000
SP (Rs.) 100 100
Sales (Rs.) 700,000 900,000

c) Annual production of Product X


= Annual demand - Opening stock = 500,000-12,000 = 488,000 units
Annual requirement for raw materials
= Annual Production*Material Per unit-Opening stock of Mat.
Material A = 488,000*4 units-24000 units = 1,928,000 units

© The Institute of Chartered Accountants of Nepal 32


Suggested Answer Paper Group II

Material B = 488,000*16 units-52,000 units = 7,756,000 units


i) Computation of EOQ when purchase order for both the materials is placed separately
�2×Annual regd×ordering cost
EOQ = Carrying cost per unit per annum

Material A =
�2 × 1,928,000 × 15,000
150 × 13%

√5,784,000,000
19.5
= 54,462.40 units
= 54,462 units

Material B =
�2 × 7,756,000 × 15,000
200 × 13%
= 94,600 units
ii) Computation of EOQ when purchase order for the both materials is not placed
separately.
�2×(1,928,000+7,756,000)×15,000
Material A & B= Rs. 190×13%
=108,542 units
Rs. 190 (Note) = 108,452 units
Material A = (108,452*1,928,000)/96,84,000 = 21,592 units
Material B = (108,452*7,756,000)/9,684,000 = 86,860 units
Note: Carrying cost per unit per annum = (Rs. 150*1,928,000)+(Rs.
200*7,756,000)/(1,928,000+7,756,000) = Rs. 190
5.
a) Elfa Engineering Ltd. produces hinge for cases of a machine. It has three production
departments and two service departments. Currently overheads are allocated on the
basis of direct labour hours. The management is considering changing the basis of
overhead allocation from a single overhead absorption rate to departmental overhead
rate. The estimated annual overheads for the five departments are as under:
Amount in 000
Production Departments Service Department

Particulars A B C Inspection Maintenance

Direct Material Rs. 7,770 200 750

Direct Labour Rs. 1,200 385 480

Indirect Material Rs. 30 75

Other Variable
Overheads Rs. 250 50 70 30 15

Fixed Overhead Rs. 500 45 115 150 210

© The Institute of Chartered Accountants of Nepal 33


Suggested Answer Paper Group II

Total Departmental
Expenses Rs. 9,720 680 1,415 210 300

Maximum
Production Capacity
Units 20,000 25,000 30,000

Direct Labour
Hours 24,000 9,600 12,000

Machine Hours 8,500 1,200 1,500

Use of Service
Departments:

Maintenance -
Labour Hours 630 273 147

Inspection-
Inspection Hours 1,000 500 1,500

The brief of work undertaken in each of the production department is as follows:


A: The metals are cut into shape and necessary machine work is done to form hinges
and axles are produced.
B: The hinge and axles produced by department A are assembled by labour.
C: The assembled hinge are painted and packed in boxes by labour as a ready produce.
Required: (7 marks)
i) Single overhead absorption rate.
ii) Most appropriate departmental overhead absorption rate.

b) Define Zero Base Budgeting (ZBB). What are the steps involved in preparation of ZBB.
(4 marks)
c) What is breakeven chart and what are the general conclusions we get from the chart.
(4 marks)
Answer:
a)
Computation of Blanket Overhead Rate Amt. in 000
Production Department A B C Total
Variable Overhead 250.00 50.00 70.00 370.00
Fixed Overhead 500.00 45.00 115.00 660.00
Service Department Inspection Maintenance
Indirect Material 30.00 75.00 105.00
Other Variable Overheads 30.00 15.00 45.00
Fixed Overhead 150.00 210.00 360.00
Total Overhead in 000 A 1,540.00
Total Direct Labour Hours B 45,600
Blanket overhead rate A/B 33.77

© The Institute of Chartered Accountants of Nepal 34


Suggested Answer Paper Group II

As given in the question, department A is machine oriented hence machine hours can
be most appropriate base for determining overhead rates. Similarly department B and
C are labour oriented hence labour hours can be most appropriate base for determining
overhead rates.
Computation of Departmental Overhead Rates Amt in 000
PRODUCTION SERVICE
Production Department A B C Inspection Maintenance
Indirect Material - - - 30.00 75.00
Other Variable Overheads 250.00 50.00 70.00 30.00 15.00
Fixed Overhead 500.00 45.00 115.00 150.00 210.00
Total 750.00 95.00 185.00 210.00 300.00
Allocation of Service
Department Overhead
Maintenance -Labour Hours 180.00 78.00 42.00 - (300.00)
Inspection- Inspection Hours 70.00 35.00 105.00 (210.00)
Total Overhead in Lakh Rs. 1,000.00 208.00 332.00
Machine
BASE Hours Labour Hours
Hours 8,500 9,600 12,000
Departmental Rate 117.65 21.67 27.67

Working Note:
Allocation of Service department overhead (Rs. '000):
A: 300/105×630 = 180
B: 300/1,050×273 = 78
C: 300/1,050 ×147 = 42
b) Zero Base Budgeting is a method of budgeting starting from scratch or zero level.
Proposals for the coming period should be based on merit and not related to past
performance. Budgets prepared by conventional methods are the incremental type of
budget based on actual performance in the past periods. In the zero base budget, the
results of the past year is not accepted as a basis, since the past may conceal
inefficiencies. Zero Base Budget is mainly prepared by taking the following steps.
(i) Identification of decision units
(ii) Preparation of decision packages.
(iii) Ranking of decision packages using cost benefit analysis.
(iv) Allotment of available funds according to the priority determined by ranking each
decision package is a self-contained module explaining the need for a certain activity,
its costs, its benefits consequences if the packages is not accepted etc. The ranking of
package based on cost benefit analysis by the difficult levels of management starring
from the bottom upward ensures allotment of funds to relatively more important and
essential activities.
c) The breakeven chart is a graphical representation of cost-volume profit relationship. It
depicts the following:

© The Institute of Chartered Accountants of Nepal 35


Suggested Answer Paper Group II

i) The profitability of undertaking at different levels of output.


ii) The point at which neither profit is made nor loss is incurred. This is known as
breakeven point.
iii) The relationship between marginal cost, fixed expenses and the contribution.
iv) The margin of safety represents the difference between the total sales value and the
sales value at the break-even point.
General conclusions from break-even chart:
i) The location of BEP is determined by the magnitude of fixed costs.
ii) The location of BEP measures the margin of safety of the business.
iii) The magnitude of the angle of incidence is determined by the magnitude of the
variable cost and hence the need to control the variable costs. This will help to maintain
the rate at which profit grows.
iv) For assessing the efficiency and stability of the business, both the location of BEP
and angle of incidence has to be studied.
v) Fixed cost play a dominant role in the profit structure and hence business with low
fixed costs can weather a slump more successfully than those with large fixed cost.
6. Write short notes: (4×2.5=10 marks)
a) Escalation clause
b) Cost audit
c) Limitations of uniform costing
d) Cost reduction
Answer:
a) This clause is usually provided in the contracts as a safeguard against any likely
changes in the price or utilization of material and labour. If during the period of
execution of a contract, the prices of materials or labour rise beyond a certain limit, the
contract price will be increased by an agreed amount. Inclusion of such a term in a
contract deed is known as an ‘escalation clause’.
An escalation clause usually relates to change in price of inputs, it may also be extended
to increased consumption or utilization of quantities of materials, labour, etc. (where it
is beyond the control of the contractor). In such a situation the contractor has to satisfy
the contractee that the increased utilization is not due to his inefficiency.
b) Cost Audit is defined as the verification of cost accounts and a check on the adherence
of cost accounting plan. It in fact comprises of:
(i) The verification of cost accounting records for assuring the accuracy of cost
accounts, cost reports, cost statements, cost data and costing techniques.
(ii) Examining cost accounting records to ensure that they adhere to the cost accounting
principles plan, procedures and objectives.
In other words, the cost auditor ensures that the cost accounting plan is in accordance
with the objectives established by the management and in conformity with the
appropriate system of cost accounting.

© The Institute of Chartered Accountants of Nepal 36


Suggested Answer Paper Group II

The purpose of cost audit can be broadly classified as (i) Protective and (ii)
Constructive.
Protective Purpose:
It examines that there is no undue wastage and losses and the costing system brings out
the correct and realistic cost of production and processing
Constructive Purpose:
It provides management with information useful in regulating production, choosing
economical methods of operation, reducing operations cost and formulating plans etc.
c) 1. Sometimes it is not possible to adopt uniform standards, methods and procedures of
costing in different firms due to differing circumstances in which they operate. Hence,
the adoption of uniform costing becomes difficult in such firms.
2. Disclosure of cost information and other data is an essential requirement of a uniform
costing system. Many firms do not wish to share such information with their
competitors in the same industry.
3. Small firms in an industry believe that uniform costing system is only meant for big
and medium size firms, because they cannot afford it.
4. It reduces monopolistic trend in the business, due to which prices may be increased
artificially and supplies withheld.
d) Cost reduction may be defined as the achievement of real and permanent reduction in
the unit cost of goods manufactured or services rendered without impairing their
suitability for the use intended or diminution in the quality of the product. Cost
reduction, therefore, should not be confused with cost saving and cost control. Cost
saving should be a temporary affair and may be at the cost of quality. Cost reduction
implies the retention of essential characteristics and quality of the product and thus it
must be confined to permanent and genuine savings in the costs of manufacture,
administration, distribution and selling, brought about by elimination of wasteful and
inessential elements from the design of the product and from the techniques and
practices carried out in connection therewith. In other words, the essential
characteristics and quality of the products are retained through improved methods and
techniques used and thereby a permanent reduction in unit cost is achieved. The
definition of cost reduction does not, however, include reduction in expenditure arising
from reduction or similar government action or the effect of price agreements. The
three fold assumption involved in the definition of cost reduction may be summarized
as under:
i) There is saving in unit cost.
ii) Such saving is of permanent nature.
iii) The quality and utility of goods and services remain unaffected, if not improved.

© The Institute of Chartered Accountants of Nepal 37


Suggested Answer Paper Group II

Paper 6: Business Communication and Marketing


All questions are compulsory.
Section -'A'
1. Read the following case and analyze it on the basis of the following questions.
(6+6+8=20 marks)
Anita met Nikita at the beginning of the school year. Anita was drawn to Nikita because
she seemed confident and positive. Over several months, the two of them became good
friends, sharing high and low points about school, family, and dates.
Two months ago, Nikita started dating Rabi, a man who dropped out of college after two
years and who now works as a waiter. At first, Nikita seemed happy with Rabi, but then
she started changing. She’s become less extroverted and a lot less positive.
Often, when Anita suggests doing something together, Nikita says she can’t because Rabi
might come over or call, and he doesn’t like her not to be available to him. When Anita
sees them together, she notices that he doesn’t treat her with respect and often criticizes
her harshly. For example, when Nikita said something to Rabi when he was on his cell
phone, he shouted, “Don’t talk to me. I’m on the phone.” Later, when Nikita dropped some
papers, Rabi said harshly, “You are so clumsy!”
Anita is concerned that Nikita may be in a relationship that is verbally and physically
abusive. Anita thinks that Rabi is damaging Nikita’s self-concept, and she wants to help.
Anita: I’m just worried about you. I don’t like the way he treats you.
Nikita: Because he called me clumsy? I am clumsy, and besides, if I do something stupid,
I can’t expect him not to notice.
Anita: But he doesn’t show any respect for you at all.
Nikita: Well, he’s a guy. He says what he’s thinking. I know a lot of people’s boyfriends
like that. Besides, I don’t think there’s anything wrong with Rabi. I think I just have to stop
doing things that make him mad.
Issues:
a) If you were Nikita’s friend, what responsibilities would you have, if any, for helping
her?
b) If you were Rabi’s friend, what might you say to alter his behaviors?
c) Does communication differ according to gender? Discuss with reference to the case
above.
Answer:

a) If I were Nikita’s friend, then I would listen to her and talk about her love positively.
It is necessary to have empathetic understanding with others so that they can open
up fully. It builds and sustains the friendship too. Then, one can take a rather
objective view and evaluate the terms of equality in the relationship. It also

© The Institute of Chartered Accountants of Nepal 38


Suggested Answer Paper Group II

becomes important to discuss how the nondiscriminatory and non-abusive language


is important to establish equal terms of reference.
b) If I were Rabi’s friend, I would let him know he is a bit rough and insensitive fellow.
Since Nikita is now head over heels in love with him, later on she might not tolerate
such insensitive and abusive behavior. Therefore, as a friend to Rabi, it becomes
one’s responsibility to make him aware about his behavior and language. I would
suggest him how kind and gentle language promotes the relationship in love, and
establishes strong and long lasting relationship.
c) I do agree. Females are much more emotional and soft hearted than males. They
are very sensitive in the relationship. The case clearly shows that Nikita seems very
careful in the relationship; she believes that she should provide most favorable
environment to Rabi so that he enjoys her company and won’t leave her. On the
contrary, Rabi seems very insensitive and shows his male chauvinistic behavior. It
proves that communication is gender specific and for females mutual talk, and
spending time together are the secrets of building friendship, whereas males seem
to give less priority to communication in relationship.

2. Prepare a job application letter in response to the advertisement given below: (10 marks)
Assistant Product Manager: We are currently seeking an assistant product manager for
our fast-paced organization. S/he must possess a sound knowledge of marketing related
activities such as making marketing plan; executing, monitoring, & evaluating it. S/he
should be very good at computer use and must have very good interpersonal
communication skills. Please send your application to Ms. Courtney L. Donahue,
director of Del Rio Enterprises.
Answer:
May 23, 2019
Ms. Courtney L. Donahue
Director, Human Resources
Del Rio Enterprises
Denver, CO 82511
Dear Ms. Donahue:
I am applying for the post of assistant product manager realizing my education and training
closely parallel your needs. According to your advertisement, the job includes “assisting in the
coordination of a wide range of marketing programs as well as analyzing sales results and
tracking marketing budgets.” A recent internship at Ventana Corporation introduced me to
similar tasks. Assisting the marketing manager enabled me to analyze the promotion, budget,
and overall sales success of two products Ventana was evaluating. My ten-page report
examined the nature of the current market, the products’ life cycles, and their sales/profit
return. In addition to this research, I helped formulate a product-merchandising plan and
answered consumers’ questions at a local trade show.
Intensive course work in marketing and management, as well as proficiency in computer
spreadsheets and databases, has given me the kind of marketing and computer training that Del
Rio probably demands in a product manager. Moreover, my recent retail sales experience and

© The Institute of Chartered Accountants of Nepal 39


Suggested Answer Paper Group II

participation in campus organizations have helped me develop the kind of customer service
and interpersonal skills necessary for an effective product manager.
After you have examined the enclosed résumé for details of my qualifications, I would be
happy to answer questions. Please call me at 9851133333 to arrange an interview at your
convenience so that we may discuss how my marketing experience, computer training, and
interpersonal skills could contribute to Del Rio Enterprises.
Sincerely,
Kendrahawkins
Kendra A. Hawkins
3. Answer these questions. (2×5=10 marks)
a) You are a research consultant and a company has sent you to a fieldwork to study on
the responses of their dealers and consumers at different places of the country. You are
at Bandipur now and collecting views of the respondents. Write an e-mail to the
company director reporting the major highlights of your recent study.
b) Discuss the secrets of involving actively and efficiently in a meeting.
Answer:

a)
To: ashoksinha@gmail.com
From: ratan20@gmail.com
Date: November 15, 2019
Subject: fieldwork briefing
Dear Dr. Sinha:
I’m now at Bandipur and news is so good here. The responses of both dealers and
consumers are really wonderful. Almost all the dealers here have sold our brand, and
they are really happy from its sale. It has monopoly in the footwear market. The
dealers have nothing to say about the quality and price of our product. But they have
one complain about the company’s current practice on delivery and payment system.
The consumers of every age group are found satisfied from our products. But most
of the young ladies are looking for more varieties of color and design. I hope I will
return soon and present my final report to you.

Regards,
Ratan Subba
b) Meetings are an opportunity for you to showcase your abilities and boost your career.
To get the most out of the meetings you attend, try these techniques:
• Arrive early. You show respect and look well organized by arriving a little early.
• Come prepared. Bring the agenda and any distributed materials. Study the topics
and be ready with questions, comments, and good ideas.

© The Institute of Chartered Accountants of Nepal 40


Suggested Answer Paper Group II

• Have a positive attitude. Use positive body language; speak energetically.


• Contribute respectfully. Wait your turn to speak; raise your hand to be recognized.
• Wait for others to finish. Show respect and good manners by not interrupting.
• Keep your voice calm and pleasant, yet energetic. Avoid showing anger as this
focuses attention on your behavior rather than on your ideas.
• Give credit to others. Gain allies and enhance your credibility by recognizing
others in front of peers and superiors.
• Put the cell phone and laptop away. Focus your attention on the meeting, not on
answering e-mails or working on your computer.
• Help summarize. Assist the meeting leader by reviewing points you have noted.
• Express your views IN the meeting. Build trust by not holding post-meeting
“sidebars” that involve criticism and judgments.
• Follow up. Send the signal that you are efficient and caring by completing the
actions assigned to you.
4. Write short notes on: (4×2.5=10 marks)
a) Pros and cons of workforce diversity
b) Chronological resume
c) Group decision making process
d) Communicating ethically and responsibly
Answer:

a) Having a diverse workforce can benefit consumers, work teams, and business
organizations. However, diversity can also cause divisiveness among identity
groups. Business communicators should be aware of and sensitive to differences in
the communication techniques of men and women. To promote harmony and
communication in diverse workplaces, many organizations develop diversity
training programs. You must understand and accept the value of differences. Don’t
expect conformity, and create zero tolerance for bias and prejudice. Learn about
your cultural self, make fewer assumptions, and seek common ground when
disagreements arise.
b) A chronological resume is perhaps the simplest resume to write and is often the
format that is most preferred by employers. It is a fact-based resume that allows
employers to quickly skim through and get a feel for your work experience and
qualifications. It is written in the traditional format. Education and Experience are
the most highlighted headings in this resume. Within each section, the most recent
information is presented first. This type of resume is especially effective for
applicants who have progressed up a clearly defined career ladder and want to move
up. It is less effective for those who have career gap in their work history.
c) Groups process information in four stages: orientation, discussion, decision, and
implementation. Members work through these stages as they identify problems,
determine possible outcomes, and implement solutions. Challenges to group
decision making include the tendency of the group to spend too much time
discussing information its members already know; the inability of group members

© The Institute of Chartered Accountants of Nepal 41


Suggested Answer Paper Group II

to express themselves clearly; errors in judgment; and overlooking pertinent and


sometimes important information.
d) Ethical communication is fundamental to thoughtful decision-making and
responsible thinking. It is about developing and nurturing relationships and
building communities. Ethical communication is also accepting responsibility for
the messages we convey to others and the short-term or long-term consequences of
our communication acts.
Developing concise, sensitive messages that focus on the receiver’s point of view
will build and protect goodwill and demand the attention of the receiver.
Communicating ethically and responsibly involves clarity, truthfulness, and
consideration.

Section -'B'

5. Read the following case carefully and answer the questions given below: (4×5=20 marks)
After Nepal embarked on the path of economic liberlisation in early nineties, the
Government of Nepal has adopted various policy measures and established institutional
support mechanism to attract greater flow of foreign investment in the country. KFC
(Kentucky Fried Chicken) is a fast food restaurant chain headquatered in Louisville,
Kentucky, United States which specializes in fried chicken. KFC is the world's largest fried
chicken chain with over 17,000 outlets in 115 countries and territories as of December
2011. Pizza Hut is also an American restaurant chain and international franchise that
offers different styles of pizza along with side dishes including salad, pasta, buffalo wings,
breadsticks and garlic bread. KFC and Pizza Hut entered Nepal in 2009, November 23 as
the first multinational food chain and opened their outlet at Durbar Margh. More than 500
people were served in the first two hours of business the very first day. Devyani
International franchisee of KFC and Pizza Hut also opened delivery at Blue Bird in
Tripureshwor in the year 2012, providing free home delivery service. The international
chain employs some 180 Nepalese.
A dispute between employees and the management has forced KFC and Pizza Hut to close
their outlets for about the month in 2012 when the Government is running Nepal Investment
year 2012/2013 to attract foreign investment in the country. The management has shut the
outlets alleging that the workers manhandled Human Resource Manager. Hotel and
Restaurant Workers Association leader says "the employee had been in the process of
registering their union at the Labour Office in Teku and the management had shut down
the eatery unilaterally to stall the registration of the union, accusing the staff of
manhandling HR manager".
According to KFC and Pizza Hut many of the ingredients are imported from abroad,
including the chicken from Brazil and Thailand and Potatoes from Australia via India.
Nepalese farmers are wondering why KFC and Pizza Hut need to import these while they
are readily available in Nepal. Recently, local poultry farmers seized two Indian containers
laden with chicken-meat for the Kathmandu-based on KFC Restaurant in Dharke. Both the
consignments have documents for customs and quarantine clearance from Birgunj.
However, the local farmers claimed that the chicken was being imported illegally. Devyani
International is an associate company of RJ Corp that has interest in Pepsi, Beer, Milk
Creambell Ice cream and Stem-cell Banking and Education in Nepal and India.
© The Institute of Chartered Accountants of Nepal 42
Suggested Answer Paper Group II

After earthquake of April 2015 KFC was shut down due to trade union dispute but it
reopened on September after four months of uncertainty. Now the business environment
has been improved due to stable government. Many foreign investors are in pipe-line to
invest in future.
a) What are the main issues of the case?
b) Describe the service marketing environment in Nepal?
c) What types of Pizza Hut's marketing strategy is suitable in Nepal?
d) Do you believe the foreign investment will grow in Nepal? Describe.
Answer:
a) The main issues of the case are as follows:
The main issues of the case are related with foreign investment in Nepal which has
created various opportunities and challenges in Nepalese business environment. One
of the issues mentioned in the case is overwhelming response on the launching of
multinational companies like KFC and Pizza Hut which is very encouraging for
investors. Another issue is labor relation problems in such chain stores. Conflicts
between management and employees while registering labor union and adverse
situation because of that reason is one of the main issues of the case. Similarly, use of
imported ingredients by not using local such ingredients, which was raised by the
Nepalese farmers, is another main issue of the case. In the given case, it has already
mentioned that many of the ingredients are imported from abroad such as chicken from
Brazil and Thailand and potatoes from Australia. Thus, celebrating investment year by
government in one hand and unfavorable situation created by labor disputes and
unhappy Nepalese farmers and their protests against import of ingredients which are
easily available in Nepal in another hand are the main issues of the case.
b) Marketing environment is always changing. While taking about service marketing
environment it has both opportunities as well as threats. Based on the above case,
customers are very happy because of the emergence of global chain store like KFC and
Pizza Hut. But it has created challenges for local producers and suppliers. Level of
competition has increased. Local firms have stared to face global competition.
Bargaining power of trade union has increased.
Similarly, while talking about macro environment of service marketing, foreign
investment in service sectors have increased. Many foreign investors are in pipe line to
invest in future. Now we have stable government. Thus we can say with confidence
that business environment has become favorable for investors. Employment
opportunities has also increased in Nepal. Thus in conclusion, growing investment has
created both opportunities and challenges in Nepalese service marketing environment.
1. Political-legal, technological, economical and socio-cultural factors dictate the
marketing environment in the context of Nepal. They are both directly and
indirectly linked to impact business decision and affect their marketing activities.
2. Politically, Nepal has now been moving ahead for flexible foreign investment
policy. After election in 2017, Nepal has got stable government after fifteen years.
Now government amended several legal policies in favour of foreign investors.
These policies influence the marketing directly or indirectly.

© The Institute of Chartered Accountants of Nepal 43


Suggested Answer Paper Group II

3. The economic environment of Nepal is another element contributing to marketing


environment in Nepal has been improved. The government and other international
organizations have predicted around 7 percent growth rate in 2019.
4. Talking about the micro-elements of marketing, they’re those factors which can be
influenced by the business. It includes customers, employees, competitors, media,
shareholders, and suppliers have now conducive environment. Banks, Insurance,
air lines restaurants and hotels have more attraction Foreign Direct Investment.
Trade unions are matured and become professional.
c) Pizza Hut is a multinational chain store. It has its own competitive advantages. Thus,
in my opinion, product line strategy is appropriate while formulating product strategy
as they re offering close related products. Regarding pricing strategy, market skimming
strategy is appropriate. In this strategy, marketers set very high price targeting image
oriented customers. Exclusive distribution strategy is suitable for Pizza Hut because
products offered by them are considered as specialty product by Nepalese customers.
In the context of promotion, pull strategy is suitable for them. Internet marketing and
use of social sites will be added benefits for them. As they are the service providers,
they should not confine their marketing mix to four P’s. they must add people, process
and physical evidence in their marketing program. In conclusion, they should
implement such strategy which must be customer oriented to lead the market in future.
d) In my opinion, foreign investment will grow in future because of the improved business
environment of Nepal. As given case has already mentioned that many foreign
investors are in pipe-line to invest in future. Now we have stable government that is
great opportunity for foreign investors. In my opinion, liberal government policy for
foreign investors, dual citizenship for non-residential Nepalese, expected growth rate
of GDP, cheap labour, strategic location between two giant countries, possibilities of
tourism, easily availability of cheap raw materials, no load shedding situation, impact
of globalization triggers the foreign investment in Nepal.
6. Describe marketing environment in Nepal. Explain the emerging marketing challenges.
(10 marks)
Answer:
Nepalese marketing environment consists of forces that influence exchange relationships.
It affects the performance and outcomes of Nepalese marketing environment. It consists of
political-legal, economic, socio-cultural, technological forces which create opportunities
and challenges for Nepalese marketers.
Nepal is a republican democratic country based on multi-party system. There are more than
dozens political parties. Among them, Nepali Congress, Nepal Communist Party, are major
parties of Nepal. Ideologies of party directly affects Nepalese marketing. Nepali Congress
advocates economic liberalization and privatization where as Nepal Communist Party
advocates public sector promotion. Political instability is the major problem of Nepal.
Pressure groups are emerging in Nepal. Their activities influence marketing. Nepal
government has enacted various law such as consumer protection act, environment
protection act, income tax act, company act etc. Nepal has three tier courts. After local
election deputy mayor has been working as a chairperson of local judicial committee.
Nepal has a mixed economic system. Government’s development activities are based on 5
years development plan. Recently, Nepal Rastra Bank has implemented new monetary

© The Institute of Chartered Accountants of Nepal 44


Suggested Answer Paper Group II

policy, acute trade deficit is Nepal’s evergreen problem of trade sector. Per capita income
is very low in comparison to other countries. Merger is the common phenomena in
Nepalese financial institutions. Nepal is one of the least developed country of the world.
Corruption is very common in each and every sector of the country. Economy of the country
is survived by the foreign employment and remittance. Industrialization is very slow.
Economy is still agro-based. Total population is nearly thirty million. Majority of its
population is below fourteen. Majority of the population are Hindus. Nepal is multi-lingual
country. It has diversified culture. Because of the education and communication, culture is
highly dynamic. Pace of technological change is very show. Research and development is
neglected part of the economy. Nepal is research is natural resources but wastage of
resource is common in Nepal.
Emerging major marketing challenges are as follows:
1. The new digital age:
Use of computer, telecommunications, information, transportation, live satellite
broadcast. Internet created new way to learn about and track customers and create
products & distribute it. Video conferencing. Cell phones, fax machines, CD ROM, pen
drive, blue tooth, interactive T.V. etc. too available. E-Commerce made marketing
possible at home, can receive any goods in 24 hours a day 365 days a year.
2. The Internet:
Internet is web of networks. It allow any time anywhere connections to information,
entertainment and communication. It made close relation with customer & partners.
Internet users are in billions. Business to Business e – commence is higher than
consumer purchases. Social networking, consumer generated media are products of
internet.
3. Rapid globalization:
World becoming increasing smaller, marketers are now connected globally with their
customers. Every product is getting worldwide markets. Coca-Cola in more than 200
countries, MTV in 140. e.g. Bill Blass, US fashion designer selects materials from
Australia, designs prints in Italy, designs the dress in Texas and email drawing to Hong
Kong agent to place order in Chinese factory, finished dress will be air freighted to New
York for further distribution around the cities. Managers must think globally about the
product, competitors and opportunity now
4. The call for more ethics and social responsibility:
Consumerism and environmentalism movements are getting stronger demanding to
take greater responsibility for social and environmental impact of business companies,
actions which is also forced by law. Forward looking companies view socially
responsible actions as an opportunity to do well by doing good, serving long run
interests of their customers and community and getting sympathy.
5. The growth of not for profit marketing:
Colleges, hospitals, museums, churches, religious events, NGOS, INGOS are working
to maintain their decreasing flocks. Ads for recruits in Indian army, energy
conservation, discourage smoking, excessive, drinking, drug use. Governments are
nonprofit institutions. US Government is 24th largest advertiser in USA. Government
of Nepal is the largest advertiser in Nepal.

© The Institute of Chartered Accountants of Nepal 45


Suggested Answer Paper Group II

7.
a) Explain the types of buying behaviour? Describe the factors affecting consumer buying
process.(5 marks)
b) What is industrial product? Explain about ‘installations’ and ‘accessory equipment’
with examples. (5 marks)
Answer:
a) Buying behavior is the decision processes and acts of customers involved in buying
and using products. It is buying behavior of customers. Customers can be individual or
organization. Thus, we can classify buying behavior into two categories, i.e., consumer
buying and organizational buying. Organizational behavior refers to the buying
behavior of organizations that buy products for business use, resell or to make other
products. Organizations consists of business, industries, retailers, government, and
non-government organizations. Similarly, if any customer buy products for personal
use, household use or ultimate consumption, they are ultimate customer. Behavior
shown by customer before purchase, during purchase and after purchase is known as
consumer buying behavior. In another words, behavior shown by customer about
acquisition of product, consumption of product and disposal of product. Consumer
buying behavior varies with the type of buying decision. Based on buying decision
consumer behavior can also classified into: high involvement and low involvement
purchase.
Consumer buying behavior is affected by economic factors, personal factors,
psychological factors, socio-cultural factors. Economic factors which influence
consumer buying includes level of income, availability of liquid assets, savings, debt
and credit availability, attitude toward spending and economic conditions. Similarly,
age, gender, family size, family lifecycle, occupation, education are the personal factors
which affects consumer buying decisions. Again, psychological factors also influence
consumer buying decisions, which includes motivation, perception, learning, belief,
attitude, personality, life style etc. Socio-cultural factors also affect consumer buying
decisions. Socio-cultural factors include reference groups, family, roles and status,
social class, culture, sub-culture and cultural dynamism etc.
b) Industrial products are those products purchased for further processing or for use in
conducting a business. Thus, the distinction between a consumer product and an
industrial product is based on the purpose for which the product is purchased. If a
consumer buys yeti carpet for the use in his home, the carpet is a consumer product. If
the same carpet is purchased for the official use, the carpet is an industrial product.
Raw materials, fabricating material and parts, installations, accessory equipment,
operating supplies, business service, are the examples of industrial products among
them installations and accessory equipment are explained as follows:
Installations
Manufactured products that are an organization’s major, expensive and long-lived
equipment are called installations. The characteristic of installations that differentiates
them from other categories of industrial products is that they directly affect the scale of
operations in an organization in producing goods or services. Sales are usually made
directly from producer to business user; no middlemen are involved, Because
installations are technical in nature, a high caliber will trained sales force is needed to

© The Institute of Chartered Accountants of Nepal 46


Suggested Answer Paper Group II

market installations. Because installations require careful, detailed explanation,


promotion emphasizes personal selling. Buildings, generators, elevators, air bus are the
example of installations.
Accessory Equipment
Tangible products that have substantial value and are used in an organization’s
operations are called accessory equipment. This category of industrial goods neither
becomes an actual part of finished product nor has a significant impact on the
organization’s scale of operations. The life of accessory equipment is shorter than that
of installation. Some examples are point of sale terminals in a retail store, small power
tools and office desks.

8. Write short notes on: (5×2=10 marks)


a) E-commerce
b) Psychological pricing
c) Physical distribution
d) Cost–plus Pricing
e) Specialty product
Answer:
a) E-commerce means that the company or site offers to transact or facilitate the selling
of products and services online. It has given rise in turn to e-purchasing and e-
marketing.
b) Psychological pricing is one of the popular pricing strategies which encourages
emotional buying. It influences buyers to perceive the price favorably. Psychological
pricing includes prestige pricing, odd-even pricing, psychological discounting,
customary pricing and promotional pricing. 99 pricing is one of the examples of
psychological pricing.
c) Physical distribution includes all the activities associated with the supply of finished
product at every step, from the production line to the consumers. Important physical
distribution functions include customer service, order processing, inventory control,
transportation and logistics, and packaging and materials.
d) Adding a standard mark-up to the cost of the product is called cost-plus pricing. This
is the simplest methods of pricing, which involve a fixed cost and variable cost per
units and adding the desired profit margin on the total cost.
e) Specialty products are consumer products and services with unique characteristics or
brand identification for which a significant group of buyers is willing to make a special
purchase effort. Buyers normally do not compare specialty products. They only invest
the time needed to reach dealers carrying the wanted products. Customers are usually
willing to travel great distance to buy specialty product. Dealers do not need convenient
locations, although they must let prospective buyers know their locations. Bata shoes,
sick pack machine, sauna belt, Mercedes car, etc. are examples of specialty product.

© The Institute of Chartered Accountants of Nepal 47


Suggested Answer Paper Group II

Paper 7: Income Tax and VAT


Attempt all questions. Working note should form part of the answer.
1. Royal Industries Private Limited is a private company solely owned by Punjabi family
residing in Kupandole since last 15 years. Royal Industries is the contract manufacturer
of goods for a Chinese company located in Sanxi province of Peoples Republic of China.
The company’s entire production is exported to China through its factory cum office
building in Kathmandu that is owned by the company and partly rented to a restaurant.
Operating result of the company for the income year 2075/76 is provided as follows:

Particulars Rs. Particulars Rs.


To Opening stock: By Export Sales 5,748,520
Raw Material 25,000 By Bad debts recovered 62,500
Work-in-progress 76,292 By Commission received 135,800
Finished goods 56,460 By Gain on sale of business 260,000
assets
To Purchase of raw 3,212,400 By Export incentive from GoN 240,000
materials
To Wages 148,900 By Rent received (after tax) 95,000
To freight inwards 58,960 By Miscellaneous income 15,000
To salaries 650,000 By Exchange gain 10,000
To advance income tax 125,200 By Closing stock
To other expenses not 65,000 Raw Material 43,260
allowable for Income Tax
To Interest expenses 85,000 Work-in-progress 55,190

To Provision for bad debts 15,000 Finished goods 112,325


To Sales promotion expenses 750,000
To R & D Cost 250,000
To Donation expenses 150,000
To Repair expenses 75,000
To Depreciation 150,000
TO Legal expenses 60,000
To Office expenses 90,000
TO Staff Welfare expenses 75,000
To Sundry expenses 175,600
Net Profit 483,783
Total 6,777,595 6,777,595
Additional Information:
a) Sales promotion expenses includes Rs. 500,000 donation given to Prime Minister
Disaster Relief Fund and rest was incurred for Chinese delegates visited for contract
renewal.
b) 40% of bad debts recovered was not allowed for deduction previously.
c) Closing stock of Work-in-progress was overvalued by Rs. 10,000.
d) Exchange rate is calculated on year end effective rate for the company’s USD balance
with Nabil Bank.

© The Institute of Chartered Accountants of Nepal 48


Suggested Answer Paper Group II

e) Raw Material Purchase includes Rs. 100,000 incurred to purchase a machine. Cash
payment of Rs. 80,000 to a supplier was also included in the purchase figure.
f) Salary includes Rs. 60,000 paid to an employee without TDS.
g) Depreciation is calculated at the rate and method provided by the schedule 2 of the
Income Tax Act. However, no additional allowances applicable if any, has not been
considered.
h) 20% of the donation was given to a private school. Rest was given to a tax exempted
entity.
i) Legal expenses include Rs. 5,000 incurred for defending the title of business assets, Rs.
15,000 for income tax penalty.
j) Unabsorbed business loss of last 3 years is Rs. 256,453.
You are required to calculate: (20 marks)
i) Assessable income from business for the Income year 2075/76.
ii) Tax liability
Answer:
Calculation of Assessable Income from Business
Amount (In NRs.)
Particulars Sec Export Sales Other Working Note
Income
Inclusions:
Export Sales 7 (2) 5,748,520

Bad debts recovered 7 (2) 37,500 60% of 62,500;


assuming all export
sales
Commission Received 7 (2) 135,800 Assuming from
foreign party
Gain on Sale of BA 7 (2) 260,000
Export Incentive 7 (2) 240,000 Assuming that the right
to receive the payment
is established during
I.Y. 2075/76
Rent Received 7 (2) 105,555 Grossed up for 10%
TDS
Miscellaneous Income 7 (2) 15,000
Exchange Gain 28 Not included, since it is
revaluation
(translation) gain as per
point (d) of additional
information
Total Inclusions 6,161,820 380,555
Deductions:
Interest Expense 14 80,055.74 4,944.26
Cost of Trading Stock 15 3,197,237 Working Note 2: No
Opening stock+ allocation required in

© The Institute of Chartered Accountants of Nepal 49


Suggested Answer Paper Group II

Purchases (2 adj) other than export sales


-closing stock (1 adj) income
+Wages
+Freight inwards
Depreciation 19 199,950 Assuming the expense
of manufacturing
activities.
It is 133.33% of Rs.
150,000.
Repair and Improvement 16 75,000 Assuming that the
Expense repair expense is within
the limit of 7% of
depreciable basis of
each pool
Other expenses deductible
under Sec. 13:
Salaries Refer working note (3)
for deductibility of
13 expense in case of
612,190.99 37,809.01 failure to withhold tax
Sales promotion Exp. 13 Contribution to PM
Relief fund is
deductible u/s 12Kha
235,458.07 14,541.93 and not u/s 13
Legal Expense 13 Income tax penalty is
not deductible u/s 21
(1) (Kha).
Any other legal
expense is deductible
when it fulfils
42,382.45 2,617.55 conditions of sec. 13
Office Expense 13
84,764.91 5,235.09
Staff Welfare Expense 13
70,637.42 4,362.58
Sundry Expense 13
165,385.75 10,214.25
Deductions before
4,763,062.33 79,724.67
application of Sec. 18
Research and 18 Assuming R&D cost
Development Cost incurred for
manufacturing
activities. For
calculation refer
250,000 Working Note (4)
Advance Income Tax 21 Not deductible due to
Sec. 21 (1) (Kha)
Provision for Bad debts 24 Not deductible as it is
not for fixed obligation

© The Institute of Chartered Accountants of Nepal 50


Suggested Answer Paper Group II

but for probable


obligation
Deductible unrelieved 20 256,453 Assuming that all
losses losses from export
income in the previous
year
Assessable Income from
Business 892,304.67 300,830.33
Less: Reduction u/s
12Kha (on sales ratio) 470,916.14 29,083.86
Less: Reduction u/s 12 32,640.85 2,015.90 Working Note (5)
Taxable Income 388,747.68 269,730.57
Tax Rate 15% 25%
Tax liability 58,312.15 67,432.64 FY 2075/76

Working Notes:
(1) Common cost allocation basis:
The common cost has been allocated on the basis of relative income, i.e. the ratio is
94.18% for export sales income and 5.82% of other income

Cost of Trading stock


Value of opening stock
Opening stock of Raw materials 25,000
Opening Stock of WIP 76,292
Opening Stock of Finished Goods 56,460 157,752
Add: Purchases of Raw materials 3,212,400
Less: Accounting Errors while booking purchases
(a) Purchase of machine (100,000)
Less: Payment over Rs. 50,000 not deductible as a
Result of application of Sec. 21 (1) (e) (80,000) 3,032,400
Less: Value of Closing Stock
Closing stock of raw materials 43,260
Closing stock of WIP 55,190
Less: Adj for valuations (10,000) 45,190
Closing Stock of Finished Goods 112,325 (200,775)
Add: Freight Inwards 58,960
Add: Wages 148,900
Cost of Trading Stock 3,197,237

(2) Inability to withhold tax does not make any expense not deductible. The provisions of
Sec. 90 are applicable in such circumstances whereby the withholding agent is subject
to fees u/s 117 and interest u/s 119.
(3) Calculation of Research and Development Cost
R&D cost is deductible to the extent minimum of following costs:
(a) Actual Cost Incurred during the year, or
(b) 50% of adjusted taxable income from all businesses

© The Institute of Chartered Accountants of Nepal 51


Suggested Answer Paper Group II

Adjusted taxable income is interpreted by Income Tax Manual issued by IRD


as being taxable income which is calculated without deducting expense u/s 12,
and deducting actual incurred expenses u/s 14 (2) and 17, and deductible
expenses under all other provisions of Income Tax Act, 2058.
Particulars Export Other Total
Income Income
Inclusions 6,161,820 380,555

Deductions before Sec. 18 4,763,062.33 79,724.67

Assessable Income 1,398,757.67 300,830.33 1,699,588


Less: Deduction u/s 12Kha 500,000
Less: Unabsorbed deductible losses 256,453
Adjusted Taxable income for Sec. 18 943,135
Therefore, deductible expense is minimum of Rs. 471,567.50 or Rs. 250,000;
whichever is lower

(4) Computation of deductible donation


Particulars Export Other Total
Income Income
Taxable Income before deduction of
donation u/s 12 421,388.53 271,746.47
(ATI is same as this, as there is no
adjustment for Sec. 14 (2), 17 or 18) 693,135
5% of Adjusted Taxable Income 34,656.75
Actual (120,000*.8) 120,000
Maximum 100,000
Eligible 34,656.75
In Export Income (94.18%) 32,640.85

In other income (5.82%) 2,015.90

2. Mr. Rajesh Kumar was employed in a NGO located at remote area (classified as “A” class
area by GON) between Shrawan 2075 to Poush 2075. He joined an INGO located at
Kathmandu immediately after resigning with effect from 1st Magh 2075.
Mr. Kumar has received following salary and benefits during the income year 2075/ 76:
A) Salary and benefit received from NGO
Particulars Amount (Rs.)
Basic Salary (Gross) 100,000 p.m.
Dashain Allowance 100,000
House Rent Allowance 5,000 p.m.
Remote Area Allowance 10,000 p.m.
Leave Encashment 25,000

B) Salary and benefit received from INGO in Kathmandu


Particulars Amount (Rs.)

© The Institute of Chartered Accountants of Nepal 52


Suggested Answer Paper Group II

Basic Salary (Gross) 150,000 p.m.


Advance Salary 1 month
House Rent Allowance 25,000 p.m.
Telephone Allowance 5,000 p.m.
Travelling and Daily Allowance (TADA) 10,000/day
Gym Charge of Mr. Kumar directly paid by INGO. 2,000 p.m.
Other information:
a) Mr. Kumar deposited in Ap pr oved R et i r em ent Fund-C i t i z en In ves t ment
T r u s t in Kathmandu. He deposited Rs. 500,000 in the retirement fund within income
year 2075/76.
b) Mr. Kumar contributed donation of Rs. 250,000 to tax exempt organization.
c) Mr. Kumar had gone outstation for 25 days to monitor INGO activities. TADA
provided to Mr. Kumar is only to pay cost of foods & lodging charges. Mr. Kumar
could only submit bills of Rs. 50,000 incurred by him for food and lodging charges.
d) Life insurance premium paid by Mr. Kumar Rs. 20,000.
e) Mr. Kumar was sick and incurred medical expenses of Rs. 10,000 to hospital run by
GON.
f) TDS deducted and deposited by School for Mr. Kumar salary & benefits withdrawn by
him up to Poush 2075 was Rs. 102,563 and INGO had deposited Rs. 150,000 within
the end of Ashadh 2076.
Required: (10 marks)
i) Calculate taxable income and tax liability of Mr. Kumar for the income year 2075/76.
ii) Does Mr. Kumar require to file income tax return as per the provision of Income Tax
Act, 2058?
iii) Compute amount of TDS need to be deposited as last installment for the salary and
benefit withdrawn during the income year 2075/76.
Answer:
i) Computation of Taxable Income of Mr. Kumar for income year 2075-76
Description Amt. Rs.
A. Salary & benefit received from NGO:
Basic Salary –(100,000 x 6 months) 600,000
Dashain Allowance 100,000
House Rent Allowance–(5,000 x 6 months) 30,000
Remote area allowance–(10,000 x 6 months) 60,000
Leave encashment 25,000
B. Salary & benefit received from INGO in Kathmandu:
Basic Salary –(150,000 x 6 months) 900,000
Advance Salary-(1 month basic pay) 150,000
House Rent Allowance–(25,000 x 6 months) 150,000
Telephone Allowance–(5,000 x 6 months) 30,000
Gym Charges 12,000
Total assessable income 2,057,000

© The Institute of Chartered Accountants of Nepal 53


Suggested Answer Paper Group II

Less: Deduction:
Contribution to approved retirement fund:
Minimum of : (300,000)
1) Maximum of Rs. 300,000
2) 1/3 of Ass. Income (Rs. 2,057,000/3)
3) Actual contribution Rs. 500,000
Adjusted taxable Income (Rs. 2,057,000-Rs. 300,000) 1,757,000
Donation to tax exempt organization:
Minimum of :
1) Maximum of Rs. 100,000
2) 5% of Adj. taxable income (5% of 1,757,000) (87,850)
3) Actual contribution Rs. 250,000
Deduction due to residing in Remote area@ (Rs. 50,000/2) six (25,000)
Life insurance premium (20,000)
Taxable Income 1,624,150
@ Only ½ allowed
Computation of Tax Liability of Mr. Kumar for income year 2075-76 (Assumed Single
Status)
Tax
Particulars
Liability
Taxable income 1,624,150
Social security Tax 1%- up to 350,000 3,500
10%- tax up to next 100,000 10,000
20%- tax up to next 200,000 40,000
30%- tax up to next 974,150 292,200
Tax Payable 345,700
Less: Reduction:
1.Medical tax credit
Minimum of:
1) 15 % of approved medical exp. (15 % of Rs. 10,000) 750
2) Actual eligible medical exp. + unrecovered medical exp. earlier
3) Rs. 750 per annum
Total Tax Payable 344,950
Less: Withholding Tax 252,563
Net Tax payable 92,387
Note: 1) There shall be no tax applicable on travelling and daily allowance (TADA).
2) Leave encashment is treated as normal Employment Income
ii) Mr. Kumar need to file income tax return of income year 2075-76 if he claimed/wants
to claim donation provided to tax exempt organization. However, if he does not
claim/does not want to claim donation from his income tax liability he is not required
to file income tax return for income year 2075-76.
iii) Last installment to be paid is Rs. 92,387, as produce above.
3.
a) KTM Limited purchased raw materials amounted to Rs. 500,000 on credit from XEROX
Limited, a foreign supplier. Supplier could not receive money for long period and
© The Institute of Chartered Accountants of Nepal 54
Suggested Answer Paper Group II

contacted KTM Limited and informed amount receivables from them were written off
in their books of account due to merger with XEROX Limited, another big supplier
company. Further XEROX limited assure that amount receivables will not be asked for
the payment in future. Knowing this Income tax officer add back Rs. 500,000 as income
i.e. amount payable to foreign supplier in the tax assessment of KTM Limited.
Explain actual disposal of a liability as per Income Tax Act, 2058 and do you agree
with the steps taken by IT officer for assessing his income? (5 marks)
b) Mr. Rewat Dhakal was employee posted in foreign diplomatic mission of Nepal in
France since Shrawan 2076. He was paid annual salary of Rs. 600,000 and foreign
allowance of US $ 800 per month. Contribution towards approved retirement fund by
GON was 10% of salary. He also contributed equal amount to the retirement fund.
Calculate his taxable income. Average rate of exchange was Rs. 110 per US $. You are
required to calculate net taxable income from remuneration. (5 marks)
Answer:
a)
As per section 40(2) of the Income Tax Act, 2058, a liability is disposed off from a
person if the person is released from the burden of the liability. There will be actual
disposal of liability if liabilities are settled or cancelled or relieved from and if liability
is merged/amalgamated with another assets or liability.
At the time of purchase and consumption of raw materials is expensed (recognized as
expenses) and subsequently there is no liability for payment for cost of raw materials
purchased because KTM limited get relieved from liability to make payment to foreign
supplier. Hence IT officer is right to add back Rs. 500,000 as taxable income in
assessment of income tax liability of KTM Limited.

b) Taxable income of Mr. Rewat


Particulars Amount (Rs.)
Salary 600,000
Foreign allowance ($800*Rs.110*12) 1,056,000
Contribution to Approved Retirement Fund 60,000
Assessable Income from Employment 1,716,000
Assessable Income 1,716,000
Less: Allowable Reduction
Contribution to Approved Retirement Fund
Lower of Rs. 300,000 or 1/3rd of Assessable Income i.e. Rs.
572,000 or actual Rs. 120,000 (120,000)
Taxable Income 1,596,000
Less: Other Reduction
Foreign allowance (75% of Rs. 1,056,000) (792,000)
Net Taxable Income 804,000

4.
a) As on 1st Baishakh 2075 French international organization "Humanity Foundation"
registered in France approach social welfare council (SWC) and entered General
agreement and project agreement to conduct various social activities in western Nepal
through a local partner. Humanity Foundation set up its country office in Kathmandu
and obtain tax exemption certificate and PAN from Inland Revenue office. The country

© The Institute of Chartered Accountants of Nepal 55


Suggested Answer Paper Group II

director said that the Foundation is registered in France and always resident over there
and it cannot be again resident in Nepal because it is not incorporated in Nepal.
Do you agree with the view of country director and what will be the residential status
of Humanity Foundation in Nepal for financial year 2075/76 as per the provision of
Income Tax Act, 2058? (5 marks)
b) Mr. Ram Prasad, a young sole proprietary wholesaler doing business in rented house
in Putalisadak, Kathmandu. His turnover of the business is continuously going down,
due to this reason he had applied for business visa to one of the Gulf country. He has
received business visa on 15th of Poush 2075. Mr. Ram Prasad is planning to leave the
country as on 1st of Magh 2075 for uncertain period, Income tax officer issued notice
on 27th Poush to file income tax return for part of income year i.e. before leaving Nepal
within current financial year 2075/76.
Required: (5 marks)
i) What are the conditions under which income tax officer can ask for filling income
tax return through written notice with in same financial year or before due date of
filling income return permitted by section 96 of Income Tax Act, 2058?
ii) Can income tax officer issue notice to submit income tax return for part of income
year i.e. before leaving Nepal within current financial year 2075/76?
c) Quantify the amounts in the following payments made to any person as per the section
27 of Income Tax Act, 2058: (5 marks)
i) ABC Ltd. gave a used laptop, whose depreciated value is Rs. 45,000, as a payment
to a raw material supplier, whereas the market value at the time of handover was
Rs. 30,000.
ii) ABC Ltd. paid Rs. 7,500/month for cost of meal, refreshment and entertainment on
behalf of a particular staff to a factory canteen operator. The company only
recovered Rs. 5,000/month from salary and benefit provided to that particular staff.
iii) Mr. Ram, owner of house provided accommodation facility free of cost to Mr.
Pradeep who is not employee. The prevailing market rent of such residence is Rs.
50,000/month.
iv) Mr. Hari provided loan of Rs. 500,000 for a year at the interest rate of 5 % per
annum and paid the interest of Rs. 25,000 at the time of repayment of loan, whereas
prevailing market interest is 10% per annum.
v) ABC Ltd. gave an old motor bike whose depreciated value is zero to an employee
which has market value of Rs. 7,500.
d) Mr. Prakash Sodari resident of Kathmandu provides his detail income for an income
year and request you to compute his tax liability by using both Credit Method and
Expense Method for availing foreign tax credit as permitted by IT Act 2058 and advise
him which one is beneficial to him. Details of business income of Mr. Sodari are as
follows: (5 marks)
i) Assessable income from Nepal 2,500,000 Advance tax paid Rs. 400,000
ii) Assessable income from Germany Rs. 500,000 tax paid Rs. 150,000
iii) Assessable income from Switzerland Rs. 500,000 tax paid Rs. 50,000
Assume Mr. Sodari is married.

© The Institute of Chartered Accountants of Nepal 56


Suggested Answer Paper Group II

Answer:
a)
An INGO is Nepal branch of NGO of foreign country. As such, the INGO in Nepal is
treated as Foreign Permanent Establishment of Foreign NGO. Those foreign NGOs are
usually not resident in Nepal.
As per section 2 (Ka) (Nga), a foreign permanent establishment of non-resident is
always resident in Nepal since it qualifies the definition of permanent establishment in
Nepal by its presence in Nepal for more than 90 days during the year.
Accordingly, French international organization "Humanity Foundation" was
established in Nepal as on 1st Baishak 2075 after entering into agreement with Social
Welfare Council (SWC) is resident in Nepal for the financial year 2075/76 irrespective
of its country of incorporation. A person can be resident in two states/countries as per
the provision of tax laws of respective countries, hence country director view is not
consistence with Nepalese Income Tax Law.
b)
i) As per section 96(5) of IT Act, 2058, Income tax authority can ask through written
notice to a tax payer for filling income tax return with in same financial year or
before due date of filling income return permitted by section 96 as per the provision
of jeopardy assessment under section 100. If any of the following condition are
satisfied, the tax authority may demand for jeopardy assessment:
• The person becoming bankrupt, is wound up or goes into liquidation.
• The person is about to leave Nepal indefinitely
• The person is about to leave the business, or
• The IRD otherwise considers it appropriate
ii) As per section 96(5) of IT Act, 2058, Under happening of any of the condition listed
in i) above, income tax authority (IT Officer) can issue notice to file income tax
return for any part of the income year or for entire income year before due date of
filling income tax return. Mr. Ram Prasad is about to leave the business as well as
planning to leave Nepal for uncertain period, hence IT Officer can issue notice to
tax payer in writing to file income tax return for any part of the income year or for
entire income year before due date of filling income tax return.
c)
Quantification of the amounts in the following payments made to any person as per the
section 27 of IT Act 2058 are as follows:
i) In the case of payments made by one person to another through transfer of assets,
the market value of assets transfer will be the amounts of payments i.e. Rs. 30,000.
ii) Any balance left after deducting the contribution of payee from the amounts paid
by the payer i.e. Rs. 2,500/month (Rs. 7,500- Rs. 5,000) is quantified as payments
made for cost of meal, refreshment and entertainment on behalf of a particular staff.
iii) Mr. Ram owner of house provided accommodation facility free of cost to a Mr.
Pradeep who is not employee. An amount equal to 25% of the market rent of such
residence i.e. Rs. 12,500/month is quantified as the provision of IT Act.
iv) Assuming that Mr. Hari is an employer or has provided loan to any other related
party, Rs. 25,000 need be quantified as interest income, i.e. short interest rate
charged as compared to prevailing market interest rate (Rs. 500,000 x (10%-5%)
will be quantified as per the section 27 (1) (Gha) of IT Act.

© The Institute of Chartered Accountants of Nepal 57


Suggested Answer Paper Group II

v) In the case of payments made by one person to another through transfer of assets,
the market value of assets (Motor bike) transfer will be the amounts of payments
i.e. Rs. 7,500.

d) Option 1:
Computation of Income tax liability of Mr. Sodari after claiming foreign tax credit
under Credit Method (Using tax rate for F.Y. 2076/77 and assuming couple assessment)
Tax Liability before Foreign Tax Credit
Taxable Income Tax Rate Tax (Rs)

Up to Rs. 450,000 0% 0.00

Next Rs. 100,000 10% 10,000.00

Next Rs. 200,000 20% 40,000.00

Next Rs. 1,250,000 30% 375,000.00

Bal. Rs. 1,500,000 36% 540,000.00


Total Tax Liability before foreign tax credit 965,000.00

Less: Foreign tax Credit available (187,850.00)

Tax Liability after claiming foreign tax credit


(Tax Liability using credit method) 777,150.00

Working Note-1:
Computation of average IT Rate and eligible foreign tax credit available to Mr. Sodari
by using Credit Method:
Available Foreign tax
Taxable Credit (3) min of 1 or
Country Income Tax Paid (1) Average tax (2) 2

i) Nepal 2,500,000 400,000.00 - -

ii) Germany 500,000 150,000.00 137,850.00 137,850.00


iii)Switzerland 500,000 50,000.00 137,850.00 50,000.00

Total 3,500,000 600,000.00 171,400.00 187,850.00

Working Note: 2
Average Income Tax Rate:

965,000 x100
Nepal Average Income tax rate = = 27.57%
3,500,000

© The Institute of Chartered Accountants of Nepal 58


Suggested Answer Paper Group II

Option 2:
Computation of Taxable Income of Mr. Sodari under Expense Method
Taxable Income
(Tax paid in foreign country is
Country treated as expense)

0 2,500,000.00
i) Nepal
500,000-150,000 350,000.00
ii) Germany
500,000-50,000 450,000.00
iii) Switzerland
Total taxable income 3,300,000.00

Working Note-3:
Computation of Income tax liability of Mr. Sodari under Expense Method
Taxable Income Tax Rate Tax (Rs)
Up to Rs. 450,000 0% 0.00
Next Rs. 100,000 10% 10,000.00
Next Rs. 200,000 20% 40,000.00
Next Rs. 1,250,000 30% 375,000.00
Balance Rs. 1,300,000 36% 468,000.00
Total Tax Liability (under deduction method) 893,000.00

Conclusion:
Tax liability under credit method is less than that under deduction method, as such,
credit method is preferable.
5. Write short notes of the following with reference to Income Tax Act, 2058.
(4×2.5=10 marks)
a) Recovery of Tax from Agent of Non-resident
b) Departure Prohibition Order
c) Taxpayers' Rights
d) Trustee
Answer:
a) Section 110 of the Income Tax Act states the following with respect to recovery of tax
from agent of non-resident:
1. Where a non-resident person (the tax debtor) fails to pay tax on or before the date
it is payable, the Department may by service of a notice in writing require a person
who is in possession of an asset owned by the tax debtor to pay tax on behalf of the
tax debtor up to the market value of the asset but not exceeding the amount of tax
payable by the tax debtor.
2. Where a person makes a payment to the Department pursuant to a notice under
subsection (1)

© The Institute of Chartered Accountants of Nepal 59


Suggested Answer Paper Group II

(a) The person may recover the payment from the tax debtor;
(b) For the purposes of paragraph (a), the person may retain out of any assets
including money of the tax debtor in or coming into the possession of the person an
amount not exceeding the payment; and
3. The tax debtor or any other person may not make a claim against the person referred
to in subsection (1) (b) with respect to the retention.
b) Section 106 of the Income Tax Act states the following with respect to departure
prohibition order:
(1) Where a person fails to pay tax on or before the date the tax is payable, the
Department may, by notice in writing serve on the concerned office of Nepal
Government, an order to prevent the person from leaving Nepal for a period of 72 hours
after the expiry of time limit specified in a notice served for the purpose of paying tax.
(2) Where an extension to the period referred to in subsection (1) is required, the
Department shall be required to get pre-approval from the concerned appellate court
for the purpose.
(3) Where the person referred to in subsection (1) pays the tax or makes an arrangement
for payment satisfactory to the Department, the Department may by notice in writing
served on the office referred to in subsection (1), withdraw the order.
c) Section 74 of the Income Tax Act states the following with respect to tax payer right:
(1) Tax payer shall abide by the duties in accordance with this Act.
(2) A taxpayer with respect of paying tax under this Act shall have the following
rights:-
(a) right to get respectful behavior;
(b) right to receive tax related information as per the prevailing laws;
(c) right to get the opportunity of submitting proof in own favor in respect of
tax matters;
(d) right to appoint lawyers or auditors for defense; and
(e) right to secrecy in respect of tax matters and keep it inviolable.
Clarification:- For the purpose of this section, taxpayers means a person whom
the tax is imposed on and realized from as referred to in section 3.
d) Trustee: As per Section 2 (Pa) of Income Tax Act 2058, "Trustee" means a natural
person, trust (Guthi) or other body corporate who, individually or jointly with other
natural person, trust (Guthi) or corporate body, holds a property in trust, and this term
includes the following person:
(1) The operator or administrator of the assets of a deceased,
(2) A liquidator, recipient or trustee,
(3) Any person who protects, directs, controls or manages the assets of an incapacitated
person in individual or official capacity,
(4) Any person who manages the assets under a private enterprise or similar other
enterprise, and

© The Institute of Chartered Accountants of Nepal 60


Suggested Answer Paper Group II

(5) Any other person in a position similar to that of the person as referred to in clauses
(1), (2), (3) and (4) above.
6. Hub Trade Pvt. Ltd. is engaged in import and sales of various types of goods from India,
China, Hong Kong and America. During Income Year 2075/76, it imported following
goods from various countries:
India INR 100,000 (1 INR = 1.6 NPR)
China RMB 15,000 (1 RMB = 15 NPR)
Hong Kong HKD 15,000 (1 HKD = 15 NPR)
America USD 25,000 (1 USD = 111.06 NPR)
• All goods are imported through Calcutta Port through Karkarvitta Customs.
• Custom charged on goods by Kakarvitta Custom is 15%, and VAT is imposed at normal
rate except for some goods imported from China which are 45% VAT exempted goods.
• Hub Trade Pvt. Ltd. sells the goods to a wholesaler, who in turn sales it to a distributor
and who in turn sales it to the final consumer.
There is following value additions at each level:
• Hub Trade Pvt. Ltd.: Rs. 350,000 administrative expenses and 10% mark up on sales
• Wholesaler: Rs. 200,000 administrative expense and 10% profit on cost
• Distributor: Rs. 220,000 administrative expenses and 20% mark up on sales
Required: (10 marks)

i) VAT paid by each business at each stage of sales channel


ii) Final Cost to consumer for the goods of VAT attractive goods and VAT exempted
goods.
Answer:

Calculation for First Chain of Business


(Hub Trade Pvt. Ltd.)

Amount
(NRs.)

Foreign Rate of Tax


Imported from: Currency exchange Total Taxable Exempt

India 100000 1.6 160,000.00 160,000.00

Hong Kong 15000 15 225,000.00 225,000.00

America 25000 111.06 2,776,500.00 2,776,500.00

China 15000 15 225,000.00 123,750.00 101,250.00

Total 3,386,500.00 3,285,250.00 101,250.00

Add: 15% Custom 507,975.00 492,787.50 15,187.50

Total Base of VAT Calculation on Import 3,894,475.00 3,778,037.50 116,437.50

© The Institute of Chartered Accountants of Nepal 61


Suggested Answer Paper Group II

VAT 13 % paid on import 491,144.88 491,144.88 Exempt

Add : 350,000 administrative expenses


(Administrative expenses distributed in the
ratio of total cost for taxable and tax exempt
goods, as per question the total
administrative expense is for sale of total of
goods 350,000.00 339,535.66 10,464.34

Cost of sales 4,244,475.00 4,117,573.16 126,901.84

Value Addition (10% margin on sales) (The margin is 10% on sales


price, the cost is 90% of sales) 471,608.33 457,508.13 14,100.20

Sales Price 4,716,083.33 4,575,081.28 141,002.05

VAT on sales 594,760.57 Exempt

Less : VAT paid on purchase/import 491,144.88

Net VAT Payable 103,615.69

Calculation for Second Chain of Business Tax


(Wholesaler) Total Taxable Exempt

Purchase Cost 4,716,083.33 4,575,081.28 141,002.05

VAT on purchase 0.00 594,760.57 Exempt

Add : administrative expenses (200,000


distributed in the ratio of purchase cost) 200,000.00 194,020.38 5,979.62

Total Cost 4,916,083.33 4,769,101.66 146,981.67

Add : Value Addition (10% margin on cost) 491,608.33 476,910.17 14,698.17

Sales Price 5,407,691.67 5,246,011.83 161,679.84

Add: 13% VAT on sales 681,981.54 681,981.54 0.00

Less: VAT paid on purchase 0.00 594,760.57 0

Net VAT Payable 681,981.54 87,220.97 0.00

Calculation for Third Chain of Business Tax


(Distributor) Total Taxable Exempt

Purchase Cost 5,407,691.67 5,246,011.83 161,679.84

VAT on purchase 681,981.54 681,981.54 0.00

Add : administrative expenses (220,000


distributed in the ratio of purchase cost) 220,000.00 213,422.41 6,577.59

Total Cost 5,627,691.67 5,459,434.24 168,257.43

Add : Value Addition (20% margin on sales) 1,406,922.92 1,364,858.56 42,064.36

Sales Price 7,034,614.58 6,824,292.80 210,321.79

Add: 13% VAT on sales 887,158.06 887,158.06 Exempt

Less: VAT paid on purchase 681,981.54 681,981.54

© The Institute of Chartered Accountants of Nepal 62


Suggested Answer Paper Group II

Net VAT Payable 205,176.53 205,176.53

Calculation for Final Consumer (Final Tax


Chain on business) Total Taxable Exempt

Purchase Cost 7,034,614.58 6,824,292.80 210,321.79

VAT on purchase 887,158.06 887,158.06 Exempt

Total Cost 7,921,772.65 7,711,450.86 210,321.79

7.
a) Mr. Ram Prasai a VAT registered person purchased wood/timber from Gairigaun
Community Forest of Jhapa District for business purposes through auction. The
auction price of woods at the time of auction, release order and (cutting)/saw order
was Rs. 2,100,000, Rs. 2,200,000 and Rs. 2,300,000 respectively. The release order
was issued earlier than auction time and cutting order issued by Gairigaun Community
forest.
Required: (5 marks)
i) Explain the applicability of VAT on sale of wood/timber as per the provision of VAT
Act, 2052?
ii) Compute the liability of VAT amount if any under the provision of VAT Act, 2052?
b) X Ltd. was incorporated and registered with VAT. During the month of Kartik 2076 the
company had following transaction:

NPR.
Purchases/Expenses:
175,000 (for taxable
Purchase of raw materials- transaction)
Electricity expenses 25,000.00
Cold drinks 12,000.00
Diesel 20,000.00
LP Gas 7,000.00
Printing and stationery 35,000.00
Services availed from UK 100,000.00
Sales
Taxable sales 500,000.00
Exempted sales 25,000.00
Export sales 10,000.00
All purchases, expenses and sales are exclusive of VAT.
The company had opening balance of VAT credit of
NPR. 1,150,000

You are required to calculate VAT payable or VAT credit available for the month of
Kartik 2076. (5 marks)
Answer:
a)

© The Institute of Chartered Accountants of Nepal 63


Suggested Answer Paper Group II

i) Applicability of Vat on sale of wood/timber as per the provision of VAT Act, 2052:
As per section 12A (1) of VAT Act, 2052, There shall be levied and collected tax
on the amount of royalty of the wood/timber of national forest or the amount of
auction, whichever is higher, at the time of auction, release or (cutting)/saw order,
whichever is earlier.
As per section 12 A (2) If the timber of a personal cultivated, private forest or
community forest for business purpose and despite that royalty is not chargeable,
tax shall be leviable as if it were the timber of national forest.
ii) Computation of VAT liability of Gairigaun Community forest of Jhapa District:
Since, the royalty amount is not given in question, taxable value and VAT liability
cannot be computed. Taxable value would be higher of auction amount or royalty
amount.
b)
VAT-attractive sales Rs. 65,000.00
Exempted Sales 0.00
Export Sales Rs. 0.00
Total VAT payable 65,000.00
Less: VAT credit available (Working Note-1) 39,480.42
VAT payable without adj. opening VAT credit 25,519.59
Add: Opening VAT credit 1,150,000.00
Total VAT credit available for claim in next tax period 1,124,480.42

Working Note-1
VAT credit available
-Purchase of raw materials- Rs. 175,000 x 13 % 22,750.00
(Exclusively used for taxable sales- 100 % credit)
-Electricity Expenses-Rs. 25,000 (VAT Exempt)
-Cold Drinks- Rs. 12,000 x 13% (No credit item)
Diesel (no credit item)
LP Gas (no credit item)
-Printing and Stationeries- Rs. 35,000 x 13 % x 95.33% 4,337.52
(Proportionate credit)
-Services availed from UK- Rs. 100,000 x 13 % x 95.33% 12,392.90
(Reverse VAT)
Total VAT credit available 39,480.42

Working Note: 2
Ratio of sales
Taxable sales 500000 93.46%
Exempted sales 25000 4.67%
Export sales 10000 1.87%
535,000.00

VAT attractive sales proportion 95.33%

© The Institute of Chartered Accountants of Nepal 64


Suggested Answer Paper Group II

8. Write short notes on the basis of VAT Act, 2052: (4×2.5=10 marks)
a) Tax officer
b) Electronic invoice
c) Refund of VAT paid on purchase by foreign tourist.
d) Suspension of transaction of registered person
Answer:
a) As per section 2(na) of Value Added Tax Act, 2052 "Tax Officer" means tax officer,
chief tax officer or chief tax administrator appointed by the Government of Nepal and
the term shall also include any Section Officer, Director or, Deputy Director General
designated by the Government of Nepal authorizing to exercise the power of tax officer
under this Act.
Accordingly, Tax Officer is either appointed or designated by Government of Nepal to
exercise the power mentioned in Vat Act.
b) As per Section 14A (1) of Value Added Tax Act, 2052, Tax payer may after obtaining
prior approval from the Department issue electronic invoice. As per sub-section (2),
notwithstanding anything contained in sub-section (1) above, the department may
publish a notice and compel specific tax payers to issue electronic invoice compulsorily
by using electronic medium and may order such electronic medium should be linked
with Department's central invoice monitoring system. As per sub-section (3), the
Department shall prepare and apply such working procedures relating to safety and
trustworthiness of software or equipment used to generate electronic invoice. Such
working procedures shall be followed by concerned producer, distributor and users.
c) As per section 25 Ka of Value Added Tax Act, 2052, If a foreign tourist visiting Nepal
and returning by air purchases taxable goods of more than Twenty-Five Thousand
Rupees and takes them with him or her, the tax paid on such goods shall be refunded
in accordance with the procedures specified by the Department. Tax paid by tourist on
such purchase shall be refunded to him/her after deducting 3 % as service charges.
d) As per section 30 of Value Added Tax Act, 2052, if any registered person contravenes
any of the offences as mentioned in section 29 twice or more, the Director General may
order a tax officer to suspend the transaction of such registered person for the maximum
period of seven days for each default.

© The Institute of Chartered Accountants of Nepal 65


Suggested Answer Paper Group II

Examiner’s Commentary on Students' Performance in December


2019 Examinations
This commentary has been written to accompany the published questions and answers and is
written based on the observations of evaluators. The aim is to provide constructive guidance
for future candidates, giving insight into what the evaluating team is looking for, and flagging
difficulties encountered by candidates who attempted these questions.
Subject: Financial Management
Question No. 1
Most of the students had not attempted the question or only partially attempted. Treatment of
block of asset concept, idle time and apportionment of general overhead was incorrect. There
was lack of conceptual knowledge. Treatment of opportunity cost and other portion was wrong.
Question No. 2
Students' performance was below average. Students lacked concept of break point. Part (b) was
satisfactorily answered.
Question No. 3
Calculation of capital employed and EBIT was incorrect in maximum cases. Almost all the
candidates were unable to solve the problem of agency.
Question No. 4
Student showed average performance. Treatment of administrative expenses, calculation of
working capital requirement under cash cost basis was wrong. Calculation of S.V. of share at
the end of 8th year was incorrect.
Question No. 5
Student showed average performance. Calculation of stock of portfolio was incorrect. Good
performance in question on leverage but calculation of ROI was wrong. Below average
performance in question on Mutual fund.
Question No. 6
Student performed very poorly in theory questions. Most of the students tried to attempt the
question. However, it seemed that students lacked conceptual clarity.
Question No. 7
Most students have not attempted the question or partially attempted or answered without
having good knowledge in general form only or defined the aspects only without differentiation
as required.

Subject: Cost & Management Accounting


Question No. 1
Students were unable to calculate the cost per unit in process 2. Students omitted to credit the
abnormal loss A/c by the sale proceeds of normal loss. Some student treated flavouring essence
as part of overhead.

© The Institute of Chartered Accountants of Nepal 66


Suggested Answer Paper Group II

Question No. 2
(a) Students committed mistakes in calculating the variable cost of manufacture as well as
budgeted net income. Most of the students did not correctly answer part, that is, statement
of valuable cost per unit.
(b) Some student considered Goodwill as assets without notes.
Question No. 3
(a) Most of the students could not calculate the earnings under Halsey scheme.
(b) Students committed mistake in calculating freight rate. Some student did not correctly
calculate tonne km.
Question No. 4
(a) Most of the students correctly attempted the question.
(b) Students lacked conceptual knowledge.
(c) Carrying cost was not calculated correctly in part (ii). None of the students were able to
calculate the combined EOQ. None of the students scored full marks in this question.
Students were not able to calculate the carrying cost and joint order.
Question No. 5
(a) Most students lacked concept of simple overhead rate. Students committed mistakes in
allocation of overhead and calculating single overhead absorption rate.
(b) The question on ZBB was not answered up to the mark.
(c) Most students did not provide correct answer. They showed lack of conceptual knowledge.
Question No. 6
Level of understanding of costing concept was not up to the mark. Answers to the theory
questions were not specific. Students lacked theoretical knowledge.

Subject: Business Communication


Question No. 1
The question did not address the issue of Business Company. The answers were sastisfactory
in general.
Question No. 2
Students did not seem to have access to current format materials. In letter portion, students'
performance was also good. Personal qualifications and skills could have been incorporated
in the answer. Answers seemed to be sketchy.
Question No. 3
The students were confused with the word report and answered accordingly. Part (a) was a bit
hypothetical so answers were fictional. Students were found writing CV which had wasted their
time and energy.
Actually it was an email writing question. There was lack of conceptual knowledge on writing
email. Some answers were very good whereas others could have elaborated details in question
3(a).

© The Institute of Chartered Accountants of Nepal 67


Suggested Answer Paper Group II

Question No. 4
Lack of conceptual knowledge on resume types and ethics and responsibility in
communication. Most of the answers were too general and sketchy.

Subject: Marketing
Question No. 5
The main issue of question was not properly understood by the students. Very few students did
answer the questions of case. Students were confused in marketing environment and service
marketing environment. Students had less knowledge on the subject matter. They had done less
effort to answer the question properly and they did not take into account the weightage of full
mark of each question.
Question No. 6
Marketing environment in Nepalese context was not described properly. Candidates seemed to
lack conceptual knowledge in second part of question. Particularly, it was Nepalese marketing
environment, which was the critical question, but answers were given in their own words. Most
of the students did not provide the answer properly.
Question No. 7
The definition of installation and accessory equipment were not adequate. Overall, students did
better even though question was confusing. Students had lack of knowledge about the subject
matter and their answers were short and inadequate. There was somewhere confusion.
Question No. 8
Most of the students defined clearly about the points of short notes. However, the answers were
not specific as per the coverage of syllabus.

Subject: Income Tax & VAT


Question No. 1
Students had poor understanding of the concepts regarding business, export and domestic
income segregation. Students had lack of knowledge of Section 11 and assessment of tax with
normal and special rates. All the students had attempted the question but only a few had worked
out the question completely. Majority of the students attempted the question and secured more
than 50% of the marks allocated.
Question No. 2
Students had lack of knowledge of the situation/conditions in which return is required to be
filed. Students needed to emphasize on the requirement of filling of Income Tax Return and
aggregation of income. Students had considered full of the year for tax computation. Majority
of students attempted the question and were unable to answer the IT filling (return) in case of
excess claimed donation in his IT return.
Question No. 3
There was poor understanding of exemption of foreign allowance. Many students had not
answered properly first part (a) of this question. Only few students had tried to solve 3(a).
However, maximum students had solved 3(b).

© The Institute of Chartered Accountants of Nepal 68


Suggested Answer Paper Group II

Question No. 4
There was lack of conceptual clarity. Majority of students had well knowledge in these
questions but were unable to compute tax credit calculation as asked in question 4(d).
Question No. 5
Students' performance was unsatisfactory. There wa lack of knowledge of definitions under
Section 2. Answers should have been brief and to the point. Majority of students gave answer
in general terms as against specific provision of IT Act (terms defined in line with IT Act)
Question No. 6
Students had poor understanding of the concept of partial credit. Majority of students computed
initial part of question only and were unable to compute VAT in subsequent stages of sales
channel. Only few students had the grasp of VAT Act for solving the question. Students'
performance was below the par.
Question No. 7
Student did not do well. There was lack of knowledge about non-taxable items and electricity
expenses.
Question No. 8
Majority of students gave answer in general terms, but not as especially defined/prescribed in
VAT Act.

© The Institute of Chartered Accountants of Nepal 69

You might also like